Level of Difficulty—II - Section 2: LOD Exercises - Part 1: Building skills for reading comprehension

How to prepare for verbal ability and reading comprehension - Sharma Arun, Upadhyay Meenakshi 2014

Level of Difficulty—II
Section 2: LOD Exercises
Part 1: Building skills for reading comprehension

TEST 1

Passage 1

Astrologers habitually prone to goof-ups now have an excuse for why their predictions have been going haywire, the emergence of newer and newer planets that have caused their calculations to go awry. For the international team of astronomers who recently discovered eight new planets, the arrivals are, however, a cause for excitement. Indeed, even as the rest of the world continues to be consumed by a morbid passion for shiny new war machines, deadly chemicals and sinister war tactics, astronomers have been doggedly searching the heavens for more heavenly bodies in the belief that the search will take us closer to a more exalted goal—that of knowing the truth about us and the universe. “Reality is much bigger than it seems… the part we call the universe is the merest tip of the iceberg” one scientist remarked. How true. In the beginning, skeptics wouldn’t accept that the earth actually moves, let alone that it revolves around the sun because of an unshaken belief that the earth was the centre of the universe. We’ve come a long way. Today, scientists have spotted nearly 80 extra-solar planets using sophisticated instruments. What’s more, our universe may not be the only universe in the cosmos; there could well be several parallel universes teeming with many galaxies, solar systems and planets, although none of this may be perceptible to the naked eye. Perhaps sages who say that truth is not easily perceptible, mean just this—what is evidently before us is not the whole truth.

Scientists say that “everything in the tangible universe has its shadowy counterpart in other, parallel universes”. In fact, it is by observing the play of cosmic light and shadow through powerful devices that scientists have been able to ’feel’ shapes or ’see’ shadows that indicate the existence of other heavenly bodies without actually seeing them. The international team of scientists involved in the present discovery conducted their search through telescopes in Australia, Belgium, UK and the US. Two of the newly discovered eight planets are believed to have circular orbits very like the Earth’s, while the others have well-defined elliptical orbits much like Pluto’s. This is significant because a planet with a circular orbit would more likely be hospitable to life forms than would one with an elliptical orbit. In the latter, the planet experiences extreme temperatures depending on whether it is proximate to or distant from the energy-giving star it’s circumambulating. As in the case of other recent discoveries—such as finding traces of microbes in a meteorite—this too strengthens the belief that we’re not alone in the universe. So would we be exchanging inter galactic e-mails soon? Perhaps not as yet, given that our closest neighboring galaxy is millions of light years away. What is within our immediate grasp, though, is exploring the viability of establishing human settlements in space—an endeavor that has assumed urgency what with biological terrorism and the like threatening humankind on earth. As Stephen Hawking recently said, “I don’t think the human race will survive the next thousand years unless we spread into space. There are too many accidents that can befall life on a single planet”.

1.According to the author’s belief, it can be inferred that all of the following are not true except

(a)It is unlikely that the human race can survive the next thousand years.

(b)It seems quite likely that the human race can survive the next thousand years.

(c)Accidents will wipe out the human race from this planet by accidents within the next one thousand years.

(d)Earth is the only planet with life.

(e)The age of human race is one thousand years.

2.According to the passage it can be inferred that:

(a)Finding traces of microbes in a meteorite proves the existence of life on other planets.

(b)Finding traces of microbes in a meteorite converted the belief into truth that we are not alone in the universe.

(c)Finding traces of microbes in a meteorite would make it possible to exchange galactic e-mails in future.

(d)Finding traces of microbes in a meteorite made us think about exploring the viability of establishing human settlements in space.

(e)None of these can be inferred.

3.According to the passage it can be inferred that

(a)A planet with a circular orbit is more likely to be hospitable to life than would one with an elliptical orbit because the latter experiences very low temperatures.

(b)A planet with a circular orbit is more likely to be hospitable to life form than would one with an elliptical orbit since the latter is proximate to the energy-giving star it is circumambulating.

(c)A planet with a circular orbit is more likely to be hospitable to life forms than would one with an elliptical orbit because the latter experiences extreme temperatures.

(d)Both (a) and (c).

(e)(a), (b) & (c).

4.According to the passage, we can infer that

(a)Parallel universes are half truth and half truth is not perceptible

(b)Parallel universes are not easily perceptible because they are not the whole truth.

(c)Parallel universes are not easily perceptible and what we perceive is not the whole truth.

(d)Truth is always easily perceptible.

(e)Parallel universes do not exist.

5.After the discovery of new planets, according to the passage, scientists are

(a)euphoric.

(b)jubilant.

(c)enthusiastic.

(d)elated.

(e)joyous.

Passage 2

In those days, there was no forum in DRDL where issues of general importance could be openly discussed and decisions debated. Scientists, it must be remembered, are basically emotional people. Once they stumble, it is difficult for them to pull themselves together. Setbacks and disappointments have always been and always will be an inherent part of any career, even in science. However, I did not want any of my scientists to face disappointments alone. I also wanted to ensure that none of them set their goals when they were at a low ebb. To avoid such eventualities, a Science Council was created—a sort of panchayat where the community would sit together and take common decisions. Every three months, all scientists—juniors and seniors, veterans and freshers—would sit together and let off steam.

The very first meeting of the council was eventful. After a spell of half-hearted enquiries and expressions of doubt, one senior scientist, MN Rao, shot a straight question: “On what basis did you select these five Pandavas (he meant the Project Directors)?”

I was, in fact, expecting this question. I wanted to tell him I found all these five Pandavas married to the Draupadi of positive thinking. Instead, I told Rao to wait and see. I had chosen them to be in charge of a long-term programme where new storms would arise everyday.

Every tomorrow, I told Rao, will give opportunities to these enthusiastic people—the Garwals, Prahlads, lyer and Saraswats—to gain a fresh perspective on their goals and a strong hold on their commitment.

What makes a productive leader? In my opinion, a productive leader must be very competent in staffing. He should continually introduce new blood into the organization. He must be adept at dealing with problems and new concepts. The problems encountered by an R&D organization typically involve trade-offs among a wide variety of known and unknown parameters. Skill in handling these complex entities is important in achieving high productivity. The leader must be capable of instilling enthusiasm in his team. He should give appropriate credit where it is due; praise publicly, but criticize privately.

One of the most difficult questions came from a young scientist: “How are you going to stop these projects from going the Devil’s way?” I explained to him the philosophy behind IGMDP—it begins with design and ends in deployment. The participation of the production centres and user agencies right from the design stage had been ensured and there was no question of going back till the missile systems had been successfully deployed in the battlefield.

While the process of forming teams and organzing work was going on, I found that the space available at DRDL was grossly inadequate to meet the enhanced requirements of IGMDP. Some of the facilities would have to be located at a nearby site. The missile integration and checkout facility built during the Devil phase consisted only of a 120 sq. meter shed thickly populated with pigeons. Where was the space and the facility to integrate the five missiles which would arrive here shortly? The Environmental Test Facility and the Avionics Laboratory were equally cramped and ill equipped.

I visited the nearby Imarat Kancha area. It used to be the test range for anti-tank missiles developed by DRDL decades ago. The terrain was barren—there were hardly any trees—and dotted with large boulders typical of the Deccan plateau. I felt as if there was some tremendous energy trapped in these stones. I decided to locate the integration and checkout facilities needed for the missile projects here. For the next three years, this became my mission.

6.It can be inferred from the passage that

I.emotions are a drawback for scientists.

II.scientists fail to pull themselves together when they stumble.

III.scientists should be helped by fellow scientists to overcome setbacks.

(a)I, II and III

(b)I and II

(c)II and III

(d)III and I

(e)III only

7.It can be inferred from the passage that goal setting should not be done when one is at a low ebb because

1.disappointments are likely to retard a person’s thinking capacity.

2.setbacks limit a person’s perceptions about his potential.

3.a person on his own can’t think on reasonable terms.

(a)All three

(b)1 and 2

(c)2 and 3

(d)1 only

(e)2 only

8.It can be inferred from the passage that the science council

I.served its purpose.

II.was an excellent platform for interaction.

III.meetings were jeopardized by every member flinging accusations against each other.

(a)II and III

(b)I and III

(c)I and II

(d)I only

(e)II only

9.The author selected the five Pandavas because

I.they had exemplary positive thinking traits.

II.he had faith in their capacity to meet unforeseen challenges.

III.he wanted the Pandavas to grow from the difficulties that could pose ahead.

(a)All three

(b)II and III

(c)I and II

(d)I only

(e)II only

10.According to the passage a productive leader should have the following traits:

I.Should have an open mind, enabling him to tackle problems from new avenues.

II.Should recruit greater number of young people as compared to old people.

III.Should be adept at handling his subordinates.

(a)I and III

(b)I and II

(c)I, II and III

(d)II and III

(e)III only

Passage 3

The 1983 re-organization was done with the objective of renewal; it was indeed a very complex exercise handled deftly by A.V. Ranga Rao and C.R. Swaminathan. We created a team of newly joined young scientists with just one experienced person and gave them the challenge of building the strap down inertial guidance system, an-on board computer and a ram rocket in propulsion system. This exercise was being attempted for the first time in the country and the technology involved was comparable with world-class systems. The guidance technology is centered around the gyro and accelerometer package, and electronics, to process the sensor output. The on-board computer carries the mission computations and flight sequencing. A ram rocket system breathes air to sustain its high velocity for long durations after it is put through a booster rocket. The young teams not only designed these systems but also developed them into operational equipment. Later, Prithvi and then Agni used similar guidance systems, with excellent results. The effort of these young teams made the country self-reliant in the area of protected technologies. It was a good demonstration of the ’renewal factor’. Our intellectual capacity was renewed through contact with enthusiastic young minds and had achieved these outstanding results.

Now, besides the renewal of manpower; emphasis had to be laid on augmenting the strength of project groups. Often, people seek to satisfy their social, egoistic and self actualization needs at their workplaces. A good leader must identify two different sets of environmental features. One, which satisfies a person’s need and the other, which creates dissatisfaction with his work. We have already observed that people look for those characteristics in their work that relate to the values and goals which they consider important as giving meaning to their lives. If a job meets the employees’ need for achievement, recognition, responsibility, growth and advancement, they will work hard to achieve goals.

Once the work is satisfying, a person then looks at the environment and circumstances in the workplace. He observes the policies of the administration, qualities of his leader, security, status and working conditions. Then, he correlates these factors to the interpersonal relations he has with his peers and examines his personal life in the light of these factors. It is the agglomerate of all these aspects that decides the degree and quality of a person’s effort and performance.

The matrix organization evolved in 1983 proved excellent in meeting all these requirements. So, while retaining this structure of the laboratory, we undertook a task-design exercise. The scientists working in technology directorates were made system managers to interact exclusively with one project. An external fabrication wing was formed under P.K. Biswas, a developmental fabrication technologist of long standing, to deal with the public sector undertakings (PSUs) and private sector firms associated with the development of the missile hardware. This reduced pressure on the in-house fabrication facilities and enabled them to concentrate on jobs which could not be undertaken outside, which in fact, occupied all these three shifts.

11.The author’s choice of the team indicated his

I.lack of faith in the older generation to achieve things.

II.bias towards younger scientists.

III.perception of the level of difficulty of the project.

(a)Only I

(b)I and III

(c)II and III

(d)I, II and III

(e)Only II

12.From the passage it can be inferred about the project that:

I.India had attained nothing of the ilk before.

II.project components were developed indigenously.

III.laudable efforts were made by the team members.

(a)III only

(b)II and III

(c)III and I

(d)I, II and III

(e)II and III

13.It can be said about the renewal factor that

(a)It was responsible for the project’s success.

(b)The young were rejuvenated on coming in contact with the experienced.

(c)The old scientists were refocused on their job.

(d)The young intellectually stimulated the experienced.

(e)The young and old gelled together.

14.The following traits of a good leader are evident from the passage:

I.Should be able to identify the contradicting environmental features to which a person reacts.

II.Should keep a tab on a person’s individual need fulfillment.

III.Should well understand the external factors which dissatisfy a person.

(a)All three

(b)I and II

(c)II and III

(d)I and III

(e)Only I

15.From the passage it can be inferred that

I.social, egoistic and self-actualization needs could be catastrophic at the workplace.

II.social, egoistic and self-actualization needs are catastrophic at the workplace.

III.social, egoistic and self-actualization needs should be eliminated.

IV.social, egoistic and self-actualization needs should be taken care of.

(a)I and III

(b)I and IV

(c)II and III

(d)IV only

(e)II and IV

TEST 2

Passage 1

Alcohol doesn’t often get billed as a brain food, but new research suggests that booze offers at least one cerebral benefit. It may reduce aging drinkers’ risk of developing Alzheimer’s disease and other forms of dementia.

Although extreme alcohol consumption kills brain cells, there’s contradictory evidence about whether long-term drinking has permanent effects on cognitive abilities such as reasoning and memory. Prolonged, excessive drinking can lead to the liver disease cirrhosis and may contribute to breast cancer risk, however. Drinking is also responsible for many accidental injuries and deaths.

Nevertheless, alcohol in moderation promotes cardiovascular health by boosting concentrations of good cholesterol and inhibiting the formation of dangerous blood clots. Additional compounds in red wine seem to benefit the heart and blood vessels. Drinking also appears to guard against macular degeneration, an incurable eye disease.

Now, the brain joins the list of organs that seem to benefit from alcohol.

From 1990 to 1999, Monique M.B. Breteler and her colleagues at the Erasmus Medical Centre in Rotterdam, the Netherlands, observed 5,395 individuals aged 55 and older, who didn’t initially show signs of dementia. Of these participants, 1,443 “moderate drinkers” reported having one to three alcohol beverages of some sort each day, while 2,674 said they consumed less than one drink and 165 acknowledged having four or more drinks per day. Another 1,113 participants abstained altogether.

Over an average follow-up period of 6 years, 146 participants developed Alzheimer’s disease and another 51 got some other form of age-related dementia. That put the overall risk for dementia at 3.7 per cent. The risk was about 4 per cent among nondrinkers, light drinkers, and heavy drinkers, but only 2.6 per cent of the moderate drinkers developed dementia.

Once the researchers adjusted their data to account for participants’ sex, age, weight, blood pressure, use of tobacco, and other factors that influence dementia, moderate drinkers showed only 58 per cent the risk of dementia calculated for nondrinkers, Breteler’s team reported.

Moderate drinkers had an even more marked decrease in vascular dementia, a condition in which blockages in blood vessels in the brain cause recurring, minor strokes that gradually erode cognitive ability. The researchers hypothesize that since vascular disorders are linked to dementia in elderly people, alcohol’s benefits to blood vessels might indirectly sustain brain function.

Jean-Marc Orgogozo, a neurological epidemiologist at the University of Bordeaux in France hails the study. He and his colleagues have found that French wine drinkers over the age of 65 have a reduced risk of dementia. The new research supports that finding, shows that beer and hard liquor—not just wine—are protective, and establishes the effect in somewhat younger people, he says.

John R. Copeland, a psychiatrist who’s retired from the University of Liverpool in England, calls the Dutch finding “very interesting but not unexpected.” Although Copeland’s research suggested that heavy, long-term drinking reduces cognitive ability in elderly men, people who show benefits in the new study consumed alcohol in more modest, “therapeutic quantities,” he says.

However, Orgogozo questions exactly what quantity constitutes a happy-hour medium. His own past research suggests three to four drinks per day are required to help ward off dementia. The lower threshold for benefit in the Dutch study may reflect participants’ underreporting of alcohol consumption in a country that, unlike France, attaches a stigma to drinking, Orgogozo says.

1.Which of the following can be inferred from the passage?

I.Sometimes, alcohol is considered as brain food.

II.Alcohol reduces the development of Alzheimer’s disease.

III.Alcohol may be a cure for dementia.

(a)Only II and III

(b)Only III

(c)Only II

(d)I only

(e)None of these

2.According to the passage, alcoholism can lead to all of the following except

(a)macular degeneration

(b)breast cancer

(c)death of brain cells

(d)cognitive problems

(e)liver cirrhosis

3.According to the passage, alcohol may have some beneficial effects on all of the following except,

(a)brain.

(b)liver.

(c)eyes.

(d)cardiovascular health.

(e)None of these.

4.The observations of M.B. Breteler do not include which one of the following?

(a)Moderate drinking reduces the risks of vascular dementia.

(b)Vascular disorders are linked to dementia in elderly people.

(c)Alcohol is highly beneficial for brain function.

(d)Heavy drinking is advised to reduce the risks of dementia and Alzheimer’s disease.

(e)All of these observations have been made by Breteler.

5.According to the passage, the term “therapeutic quantities” is used to indicate that

(a)alcohol acts as a medicine.

(b)moderate drinking is advisable.

(c)moderate drinking has some medicinal values.

(d)small quantities of alcohol is being prescribed as a medicine.

(e)moderate drinking is advisable for everyone.

6.The attitude of the author towards the benefits of alcohol in reducing the risk of dementia in elderly people can be described as:

(a)skeptical.

(b)radical.

(c)positive.

(d)prejudiced.

(e)indifferent.

Passage 2

Is there a plant that eats animals? And if I were to eat that plant, will I become a non-vegetarian, albeit once removed? After all, such a plant is technically a non-vegetarian!

This question is not just coffee-table chatter. There are insect eating plants in nature. The Venus fly trap is an oft-quoted example. Now comes the startling information about a member of the pitcher plant family, which has a particular preference for termites, and eats thousands of them at one go. Drs. Marlis and Dennis Merbach and their associates from Germany report on such a plant in the 3rd January issue of Nature.

Our general opinion about plants is that they are truly ascetic, demanding little from other life forms. Much of what they need comes from whatever there is in the ground below, sunlight, some water and air. They make carbohydrates out of this spartan set of ingredients and store them in their bodies. Animals like us pluck these and feed ourselves.

Many of us humans pride ourselves by declaring that we eat nothing but plants, and that we do not eat meat since that amounts to harming animals. The truth is plants are life forms too; furthermore, such interdependence and “big fish eat small fish” is an inescapable part of existence. Wanton hurting of other life forms is what we should not be practising.

The relation between insects and plants has always been intimate and mutually beneficial. Many plants need insects such as bees to help in propagation. For this purpose, they have put out elaborate structures in their flowers, which entice the bee. The colour attracts the insect while the nectar offers a meal. For its part, the insect carries the pollen across to another plant, helping the latter to propagate through such dispersal. In many instances, this mutualism has become so one-to-one as to be finicky or specific. For example, it is a particular legume plant alone that the insect called psyllid (or the jumping plant lice) will go to and none other: an example of “made for each other”.

It is all very well, as long as it is the flower and the nectar that the insect is interested in. But if the insect were to start eating any other part of the plant, say the leaf or the seed, it has had it! The plant puts up a strong reaction. Its defence or self-protection can be pretty offensive. The plant releases chemicals that can stun or even kill the insect. The neem tree is an example. It synthesizes and stores a chemical called azadirachtin. When the insect takes a bite at the plant, the released chemical kills all further desire in the insect to take any more bites. As a result, the insect dies a slow death of starvation. Azadirachtin is therefore called an insect antifeedant. Other plants use other strategies aimed at the same defensive purpose. For example, why are all fruits two-faced? The pulp in them is entirely tasty and nutritious. But the kernel and seed are at best indigestible (they actually can give you a stomach upset), and at worst poisonous (recall the desperate poor tribals of Orissa who suffered eating mango kernels). The pulp is the enticement, the come-hither. The seed should not be destroyed but simply dispersed, so that the next generation plant can grow.

We know that all plants are not passive bystanders in the theatre of life. Several of them are activists that seek out their food and nutrition beyond air, water, soil and sunlight. There is a whole bunch of tropical plants that are carnivorous. Yes, they gobble up insects for food! The Venus Fly Trap mentioned above is the famous one. Its leaf has two lobes, edged with interlocking “teeth”. On the surface, they have many trigger hairs. When a hapless insect were to disturb these hairs, the leaf snaps shut. Upon this, the plant secretes some digestive juice into the enclosure, which dissolves much of the insect. The food so mashed up into a puree or soup is absorbed by the plant. The chitinous outer layer of the insect is all that is left, which is discarded later. Here is a twist—the leaf of the plant holds its stomach!

What Dr. Merbach and co-workers found in the Southeast Asian country Brunei was an even more striking example of a non-veg plant. Called “Monkey’s Rice Pot” in West Australia, it is technically known as Nepenthes albomarginata. The family Nepenthes has many subspecies. The name is Greek in origin and means banisher of sorrow. One account has it that the plant was used by the ancient Greeks to banish sorrow and induce restful sleep. Obviously, its nectar or some other component has a sedative influence. (Parenthetically, Dr. Dennis Merbach writes to me that he doubts this since not even Alexander the Great came far enough east to find this plant).

The genus Nepenthes is found in Southeast Asia, Seychelles and Madagascar, and Australia, but N. albomarginata is abundant in Peninsular Malaysia, Sumatra, Borneo and Brunei. The German work was done in Brunei. Albomarginata literally means white border, and is so named because the pitcher that it presents at its leaf tips is rimmed with white hair—like protrusions. It is a slender, heat-loving plant that has attractive green-red pitchers rimmed white, and does very well in a stovehouse or heated frames.

N. albomarginata is quite different from its cousins of the pitcher plant genus Nepenthes. Others are not choosy about their prey. They catch any insect that is careless enough to step on their slippery, toothy appendage. Monkey’s Rice Pot, or N. albomarginata is picky—it loves to eat termites! In order to do so, it presents its pitcher to the prey, luring them with its colour and distinctive smell. The white hairs that fringe the rim of the pitcher are edible. Termites seem to love the hair and come to it in hordes. The scientists usually found not one or ten, but thousands of termites trapped in a single pitcher!

All the termites they found in one pitcher belonged to the same species and were in the same state of decomposition. This led the researchers to conclude that a whole battalion was caught over a short period of time. The termites caught and gobbled up were largely from no more than three genera, with one particular genus called Hospitalitermes predominating. It thus seems that these fellows are picky about N. albomarginata, and the latter returns the compliment. In fact, the plant pretty much starves when termites are not around. Over the six-month-lifespan of the pitcher, it gets by with a few dozen ants, beetles or flies (while neighbouring pitcher plants of other provenance, not being so picky, get along much better). For its part, the termite genus mentioned above too gets by usually with live fungi and algae, but upon sensing the plant, it forages in massive columns, and meets its death by the thousands at the teeth of the pitcher plant. Extraordinary, till death do them apart!

It is the white hairs that the termites go for. Pitchers with no hair are ignored. When the researchers placed near a termite marching column, both pitchers with hair and pitchers shaven off, the lead termite sensed the white hair, went back and called his mates for the forage. They came in numbers, started gobbling up the edible hairs and making food pellets out of them to carry home. In the process, they fell into the pitcher and could not escape. The fall-in rate was one every three seconds (could be even faster with a bigger marching band). After an hour, when all the hairs were gone, the pitcher was no longer of any interest to the termites. What it is in the hairs that attract the termite is not clear yet. It could be some volatile molecule, but the researchers could not detect any smell in their study. As of now, it appears that contact happens by chance. To date, N. albomarginata appears to be the only one known plant that offers up its own tissue as bait, and the only one too that specialises on a single prey.

Reading the article, it occurred to me that here is a clean and green way to rid your house of termites—plant a hedge of N. albomarginata around your house, and it will do the rest. Alas, when I raised this point, Dr. Merbach disappointed me with his e-mail message, stating that this group of termites does not feed on wood, since they feed over ground. The wood-eater termites feed underground, while the plant presents its pitchers above. There goes another of my brilliant solutions!

I wonder whether N. albomarginata is seen in India, since there are other members of Nepenthes that grow in our subcontinent. It will also be interesting to check whether some of these are pest-gobblers. One plant, called N. khasiana, is found in the Khasi hills of Assam, but its termite-preference has not been tested so far, to the best of my knowledge. This pitcher plant is an undemanding highland species that grows slowly to a height of about two feet. It is able to tolerate low humidity and temperatures and thus, should be cultivable in other chosen areas of India. And it better be soon, since this plant is already in the endangered list because of encroachment by farmers who have cleared up land in that area for agriculture.

7.According to the passage, which of the following is facing the threat of extinction?

(a)N. albomarginata.

(b)Azadirachtin.

(c)N. Khasiana.

(d)Hospitalitermes.

(e)The pitcher plant.

8.According to the passage, where does the digestion in plants take place?

(a)stem.

(b)leaf.

(c)flower.

(d)roots.

(e)fruits.

9.Accordingly to the passage, azadirachtin

(a)kills the insect.

(b)acts as a poison.

(c)acts as a repellant.

(d)has the effect of starving the insect to death.

(e)induces the death of the insect.

10.It can be inferred from the passage that

(a)tropical plants are omnivorous.

(b)tropical plants are carnivorous.

(c)tropical plants depend upon insects for food.

(d)some tropical plants absorb the insect soup.

(e)Some tropical plants are carnivorous.

11.According to the passage, all of the following can be inferred to be not true except:

(a)N. albomarginata is completely different from the Nepenthes genus.

(b)N. albomarginata is very intelligent.

(c)N. albomarginata catches insects inadvertently while others do it in a planned manner.

(d)Termites love the hair of the pitcher very much.

(e)Plants of the Nepenthes genus eat termites for food.

12.According to the passage, Hospitalitermes

(a)is a termite genus.

(b)is the main food of N. albomarginata.

(c)loves to eat the white hairs that fringe the rim of the pitcher.

(d)is mainly attracted to the N. albomarginata plant.

(e)All of the above.

Passage 3

Global climate change pundits have for long been blowing hot and cold over melting ice caps, rising ocean levels and unusually hot summers on the one hand and receding deserts, shrinking biodiversity and colder winters on the other. Climatologists are, however, unanimous in their opinion that regional variations notwithstanding, the Earth as a whole is becoming warmer—and largely due to the increased human activity. And yet, as a continent, Antarctica would seem to be bucking the trend. Recent reports quoting American scientists from the South Pole say that while temperatures in every other continent have risen over the past century, Antarctica has become appreciably colder over the past 35 years and continues to cool, becoming the only one of Earth’s seven continents to react differently to global warming. The world’s average temperature over the last 100 years has risen by 0.06°C a decade, and the average actually went up to 0.19°C between 1979 and 1998. In the Antarctic, on the other hand, temperatures fell on an average by 0.7°C a decade. Traditional theories of climate change have held that the effects of global warming ought to be magnified at the Poles. Nonetheless, recent research points out that while the Arctic is indeed getting warmer, the Antarctic is definitely getting cooler. This will mean that previous estimates of rising sea levels that included the melting ice caps of both the North and South Poles will have to be suitably revised. So what is the mystery behind the cooling of the White continent?

Since most of the inhabited and industrialised countries are clustered close to the Arctic, polluting emissions waft across to the North Pole, creating a greenhouse effect, warming the air and loosening the ice sheets. Complex interplay of ocean currents appears to have changed temperatures, cooling the southern Ocean around the Antarctic and transforming the Pole’s temperature profile. Antarctica’s harsh desert valleys are turning cooler, setting off a series of ecological consequences in the region. Meanwhile, here’s another contradiction; reports from New Zealand describe how there is a surfeit of global warming—induced break-away icebergs in the Southern Hemisphere.

13.According to the passage, all of the following are not true, except:

(a)Traditional theories failed to calculate the effects of global warming.

(b)Fall in temperatures in the Antarctic is in accordance with the traditional theories.

(c)Effect of global warming is the maximum at the Poles.

(d)Effect of global warming on Antarctica is on unexpected lines.

(e)Effect of global warming on Antarctica is on expected lines.

14.According to the passage, it can be said that

(a)Antarctica has become colder than the other continents of the world.

(b)Antarctica has become colder than the Earth over the last 35 years.

(c)The decade growth in temperature is much higher for Antarctica than the Earth itself.

(d)The average temperature rise for the decade 1979—1998 was more than the average rise in temperature over the last 100 years.

(e)Both the Arctic and Antarctic are getting cooler with global warming.

15.It can be inferred from the passage that:

(a)Our knowledge and our theories about global climate change fails to explain what is happening across the globe.

(b)It is a matter of great contradiction that we failed to understand the full import of global climatic change.

(c)We could not understand the importance of global climatic change.

(d)It was difficult for us to understand the full implication of global climatic change.

(e)The threat of global warming is accentuated due to what is happening in Antarctica.

16.According to the passage, factors affecting the temperature profile of Arctic do not include

(a)ocean currents.

(b)greenhouse effects.

(c)atmospheric pollutions.

(d)loosening of the ice sheets.

(e)Global warming.

17.According to the passage, the most important factor for global warming is

(a)hot summers.

(b)increased human activity.

(c)shrinking biodiversity.

(d)increasing pollution levels.

(e)polluting factories.

TEST 3

Passage 1

It is said that the British ruled India with the help of just two laws, the Revenue Recovery Act and Section 144 of the Criminal Procedure Code, whereas the present governments in India, with hundreds of laws on the statute book, are unable to either recover their revenues or control law and order! It is true that the British Government’s strength was its unfaltering commitment to the collection of revenues and enforcement of the law. It is not, however, my case that the country should now be run the way it was run by the British or that governments should do nothing but collect revenues and maintain law and order. But does one build a house without any mason to lay the foundation and by concentrating only on plans for interior decoration?

In post-Independence Indian public administration, there has always been a controversy, if not a conflict, relating to the roles of specialists and generalists. The former feel that the latter are cornering jobs for which they are not suitable and the latter feel that the former do not have breadth of perception and are blinkered by their narrow specialist knowledge. The IAS, especially, has been in the eye of this storm, so to say. I have heard my colleagues in functional departments say that the IAS was fit only for collecting revenues and maintaining law and order. The implication was that, apart from being unglamorous and pedestrian, these jobs were only for the unintelligent or the unscrupulous!

In the post-Independence era, the IAS too, presented with opportunities in the economic development sector and with the vanishing of land revenue as an important source of revenue for the States, began to regard tax collection and law and order jobs as not being promising enough from the job satisfaction or career points of view. With greater politicisation of the people and the spread of the populist cult, these activities came to be regarded as not merely non-developmental and feudal but almost as anti-people. No wonder that, while premier civil servants distanced themselves more and more from taking them seriously, the politicians seized the opportunity to undermine their importance and effectiveness.

An oversimplified, naïve and misunderstood version of development economics that public finance has nothing in common with private finance and that for a sovereign, especially democratic government, expenditure and income could be independent of each other, has contributed to a dangerously complacent view of poor tax recovery. A government’s sovereignty, alas, does not extend to the laws of arithmetic! A divine faith in the seductive comfort of the Laffer Curve and the emerging philosophy of privatisation and liberalisation have made strict enforcement of any rule or law appear not merely rigid and bureaucratic but almost retrograde and reactionary. (At the same time, the government is accused of not enforcing the law strictly against one’s competitors!)

Today, immunity from payment and prosecution and the impunity with which both can be evaded—even defied—are the most coveted symbols of political importance, and the prime goal of coming to power is to do this on a scale of ever-increasing magnitude and frequency. Immunity from law and impunity of violation have become the modern political equivalents of the ancient sceptre and the crown. Kaleidoscopic coalitions in which the constant goal is to stay in power but the members keep changing almost randomly, have made every politician with at least one other member in his party a potential MLA/MP/Minister! This has made the tax collection and law enforcing agencies diffident and unwilling to stick their necks out. Political scientists who wax eloquent over how the emergence of the concept of coalition is a sign of the maturing of Indian democracy have completely missed, or slurred over, its adverse impact on administration (which includes tax enforcement as well as law and order) and how a coalition is a convenient, indirect and legal way of defection—defecting from principles without defecting from the party!

1.According to the passage, it can be said that:

(a)British rule in India was very effective in collecting revenues.

(b)British rule in India was based on the rule of law.

(c)The existing laws are not sufficient for the recovery of revenues.

(d)Collection of revenues is of supreme importance for the government.

(e)The British were in India only to collect revenues.

2.It can be inferred from the passage that:

(a)IAS is fit only for collecting revenues and maintaining law and order.

(b)IAS is responsible only for collecting revenues and maintaining law and order.

(c)IAS is only for unintelligent people.

(d)Maintaining law and order and collecting revenues are the responsibilities of IAS.

(e)IAS is for the super intelligent.

3.According to the passage, emergence of the concept of coalition politics:

(a)has allowed and encouraged ideological defection.

(b)has made political defection legal.

(c)has strengthened Indian democracy.

(d)is a sure sign of the maturing of Indian democracy.

(e)None of these.

4.According to the passage:

(a)Public finance has nothing in common with private finance.

(b)Sovereign democratic government should keep expenditure and income completely separate.

(c)Development economics has not helped to correct the poor-tax recovery system.

(d)The spirit of the development economics has not been understood.

(e)The spirit of the development economics has been well understood.

Passage 2

Fifty three years since our tricolour fluttered atop Red Fort. Fifty three years of freedom and only a handful of us have succeeded in bagging a Nobel prize. Why has human resource development taken a backseat? Why have we, 1/6th of the world’s population, done so moderately in the international arena? Questions! Questions! Before we try to seek answers, let us look at ways in which we have stood out in the international arena.

Nobel laureates apart, Time magazine has identified a few Indians amongst 100 heroes and icons of the 20th century. Mother Teresa needs no introduction. J. Krishnamurthy is known for his theological ideas. Gandhi is why we are India. And amongst these personalities is a person who has put the Indian system of healing on the global map. Deepak Chopra, “Lord of immortality”, the new age guru and an endocrinologist by training is the poet-prophet of alternative medicine.

The ancient view of medicine was essentially a holistic one, i.e., an intimate interaction of body, mind, environment and spirit. The human being and the whole of society and nature for that matter, was viewed as being intelligent, conscious and ordered. Nature had laws and in order to stay well or to treat illness, one had to work with those laws. The physical world was believed to be underpinned by the mental world—by the spiritual. As these ’worlds,’ move ’inwards’ they become more subtle and difficult to measure. A lot of work has nevertheless gone into developing measurement tools for physical and psychological parameters and through their combination, we are able to test if such relationships exist. This field of study is called mind—body medicine, probably the most comprehensive and reliable scientific examination of holism. Mind—body medicine focuses on the interactions between mind and body and the powerful ways in which emotional, mental, social and spiritual factors can directly affect health. It regards as fundamental, an approach which respects and enhances each person’s capacity for self-knowledge, self-care and emphasizes techniques which are grounded in this approach. These techniques include self-awareness, relaxation, meditation, exercise, diet, biofeedback, visual imagery, self-hypnosis and group support. It explores and integrates the healing practices of other cultures, such as acupuncture and accupressure, meditation and yoga, as well as alternative Western approaches, including herbalism, massages, musculoskeletal manipulation and prayer. It views illness as an opportunity for personal growth and transformation and health care providers as catalysts and guides in this process. Consciousness, being primary, illuminates thoughts, desires and emotions in the mind and these in turn affect behaviour and physiology. Consciousness gives life to mind and body.

Over recent times, there has been an explosion of knowledge in the physical sciences. In medicine, the emphasis has tended to focus on the human body, but it has often ignored a holistic perspective, that is the body’s interaction with mind, emotions, social environment and spirit. This more mechanistic and materialistic way of viewing the human being and illness has often times been at the expense of more traditional holistic models, which placed great emphasis on these more subtle elements of human experience. Mind—body medicine and its kindred fields of study, such as psycho-neuro-immunology and psychooncology, are opening our awareness and require more communication and cooperation across the scientific community than ever before. Thus far we can say that the potential of mind-body medicine for promoting health, both physically and psychologically, seems to be great. Its costs are relatively low and the ’side-effects’ seem to be generally low. It may well be that the next major break-throughs in clinical medicine will involve the re-integration of contemporary physical sciences with traditional wisdom.

5.Which of the following is not a characteristic of mind—body medicine?

(a)It is a field that has been developed by Deepak Chopra.

(b)Deepak Chopra is the only proponent of mind—body medicine.

(c)It emerges out of the ancient view of medicine as a holistic one.

(d)It focuses on the interactions between mind and body and how social, mental, emotional and spiritual factors affect health.

(e)None of these.

6.According to the passage, which of the following is/are vital for the mind?

I.Illuminations

II.Visual imagery

III.Meditation

(a)II and III

(b)Only I

(c)Both I and II

(d)Only II

(e)Only III

7.It can be inferred from the given passage, that:

(a)modern medicine totally rejects the need of harmonizing the human body interaction with the human mind.

(b)Modern medicine does not accept the traditional holistic models.

(c)Modern Medicine is more mechanistic and materialistic in its approach than that of the traditional holistic models.

(d)Traditional holistic models attach greater importance to mind, emotions, social environment and spirituality.

(e)Modern medicine is superior to traditional holistic models.

8.According to the passage, mind—body medicine is the most comprehensive and scientific examination of holism because

I.It developed measurement tools for physical and psychological parameters.

II.It tests the relationships between physical and psychological parameters.

III.In it, the physical world, the mental world and the spiritual world move inwards to become one.

(a)Only III is correct

(b)Only I and II are correct

(c)I, II and III are correct

(d)Only I and III are correct.

(e)None of these

9.Which of the following professions is the most likely one to which the author could belong?

(a)a psychologist

(b)a physician

(c)a sociologist

(d)an economist

(e)a scientist

10.According to the passage, consciousness does not affect

I.thoughts, desires and emotions.

II.behaviour and physiology.

III.mind and body.

(a)II and III only

(b)III only

(c)I and II only

(d)Only I

(e)None of these

TEST 4

Passage 1

THE murder of Fadime Sahindal, a young Kurdish woman, in the Swedish university city of Uppsala was no ordinary affair. Her father shot her in the head, in front of her mother and younger sister, for refusing an arranged marriage with a stranger from her Turkish homeland.

The killing has stunned Swedes. Thousands have joined torchlight vigils in her memory. Thousands more are expected to attend her funeral in Uppsala’s Lutheran cathedral. Miss Sahindal was an outspoken champion for second-generation immigrants seeking their own way of life, often against their more tradition-minded parents’ wishes. Mona Sahlin, Sweden’s immigration minister, called her “a fantastic role model for young women”.

The murder has also touched a raw nerve in Sweden by questioning the country’s ability to integrate its ethnic minorities. Issues such as forced or arranged marriages and the clash between Swedish values and those of immigrants have leapt up the political agenda. The problem is not just Swedish. Denmark too worries about integration, and a row about arranged marriages is now blazing there.

But the two governments have taken different approaches. After Miss Sahindal’s death, Sweden’s ruling Social Democrats said they would give more cash to crisis centres and support groups for young women seeking to avoid arranged marriages or to leave violent partners. They also said they would close a legal loophole that lets foreign girls as young as 15 to marry, when 18 is the threshold for everyone else. The tone of Swedish ministers was one of sympathy for victims of forced marriages.

Not so in Denmark. Foreigners—or at least immigrants—plus their descendants for a couple of generations, make up about 7 per cent of the population. No vast figure, and foreign need not mean brown or black. But the new centre—right coalition was helped to power in November by a promise to curb immigration, and at times, will need votes in parliament from the overtly anti-immigrant People’s Party, which made large gains. The government has chosen to take action at the border. Till now, the foreign spouse of a Danish resident was entitled, automatically, to come in with a residence permit. But under proposals unveiled last month, this—with some exceptions, maybe—would no longer be true, even for Danish citizens, if either person was aged less than 24 (unless, of course, the incomer was a Dane or other EU citizen). Even for a couple both aged 24 or more, the permit would not be automatic; cases would be decided individually.

The Danes are also making it harder for foreigners to come to Denmark to join family members already there. The reason, they say, is that unemployment among young immigrants is already too high. Human-rights campaigners suspect grubbier motives: to keep Denmark’s proportion of brown and black residents below 10 per cent.

1.According to the passage, the murder of Sahindal reflects that

(a)Turks are generally orthodox.

(b)Kurdish are generally orthodox.

(c)Turks are opposed to arranged marriages.

(d)Kurds are opposed to arranged marriages.

(e)Kurds are unorthodox people.

2.According to the passage, the murder of Sahindal raises all of the following issues except:

(a)arranged marriages.

(b)clash of Swedish and immigrant values.

(c)Sweden’s ability to integrate its ethnic minority.

(d)Denmark’s ability to integrate its ethnic minority.

(e)None of these.

3.According to the passage, a large number of Swedes joined torchlight vigils to

(a)express solidarity with the cause for which Sahindal was murdered.

(b)express resentment against the murder.

(c)show displeasure against the murder.

(d)console the family.

(e)express solidarity with the family.

4.According to the passage, the Swedish government is planning to

(a)implement an exhaustive and extensive social reform program in Sweden.

(b)give cash awards to young women seeking to avoid arranged marriages.

(c)punish orthodox Kurdish population.

(d)Implement policies to support victims of forced marriages.

(e)None of these.

5.According to the passage, all of the following are not true, except.

(a)Denmark and Sweden both are facing the problem of integration of ethnic minority.

(b)Denmark and Sweden are planning to work together to tackle the same problem.

(c)Denmark has refused to work together with Sweden.

(d)The problem of ethnic minority is not a big issue in Denmark.

(e)All the above are not true.

6.The author of the passage could be

(a)a political scientist.

(b)a social scientist.

(c)an immigrant.

(d)a psychologist.

(e)a politician.

Passage 2

The year was 1965. I was deputy secretary (budget and planning) in the ministry of defence. In 1963, a top secret committee headed by JRD Tata had been appointed to review the decision to produce the Russian MiG-21 aircraft in India. At that time, there were hopes that the US would agree to license their F-104 fighter aircraft for production in India.

Although these had been supplied to Pakistan, New Delhi and Washington had developed a military aid relationship after the Chinese attack in October 1962.

The sum and substance of the Tata committee report was that India should opt for the US aircraft and scrap the MiG project. However, when defence minister Y B Chavan led a delegation to the US in May 1964 to sign a comprehensive aid agreement, it became clear that US would not release the F-104 aircraft for India. Instead, the US offered the F-5 aircraft which the US air force itself did not have in service. After his return from the US, Chavan went to Moscow in August 1964. At the end of the visit, Chavan not only reconfirmed the earlier MiG-21 production agreement but also placed orders for three extra squadrons.

In 1965, the Public Accounts Committee under the chairmanship of R R Morarka, demanded to see the Tata committee report. As was to be expected, the ministry of defence took the stand that the top secret report could not be shared with the PAC. Since, as deputy secretary, I was the coordinator for dealings with the PAC at my level, the file came to me. I put forward a solution which earned me the distinction of being dubbed a maverick.

I argued that while it may not be possible to share the top secret report with the entire PAC and allow it to be discussed, the PAC chairman’s status was equivalent to that of a cabinet minister; one day, he could become a Union minister. Therefore, it would be quite in order for the defence minister to invite the PAC chairman to his office, give the report to him, get him to read it and then discuss with him how he would like it to be handled. This suggestion of mine was forwarded to Chavan by the joint secretary, albeit with a lot of reservations.

The defence minister accepted my suggestion. He invited Morarka to his room and gave him the slim Tata committee report to read. Morarka read it and he did not press the matter any further. At that time, the Congress party had been in power continuously for 18 years and no opposition had ever tasted office at the Centre.

Some 37 years later, the situation is different. The opposition parties have also been at the helm of the Union government. The present chairman of the PAC has had a long innings as a cabinet minister. Once again, the same kind of issue has arisen with the PAC asking for the CVC’s top secret report on defence deals. Is Chavan’s democratic norm still relevant?

7.According to the passage, Tata Committee was constituted

I.to make a case for the licensed production of F-104 fighter aircraft.

II.to help the government in taking a decision about the production of MIG-21 in India.

III.to suggest to the government how to cancel the proposed plan of production of MIG 21 in India.

(a)I and III only

(b)I and II only

(c)II and III only

(d)All three

(e)I only

8.According to the passage,

(a)The author was of the opinion that the Tata committee report was a top secret report and it was not in national interest to discuss it in PAC.

(b)The author was of the opinion that the Tata committee report may be discussed with the PAC chairman in extraordinary circumstances.

(c)The author was of the opinion that it should be left on the wisdom of the PAC chairman how he would like it to be handled.

(d)Both (b) and (c).

(e)All of a, b and c.

9.It can be inferred from the passage that

(a)US was never interested in selling F-104 aircraft.

(b)US was never in agreement with India to supply F-104 aircraft.

(c)US breached the agreement with India.

(d)India breached the agreement with US.

(e)There was a mutual breach of agreement between India and US.

10.It can be inferred from the passage that the author thinks that

(a)the CVC’s top secret report should be discussed in PAC.

(b)the CVC’s top secret report should not be discussed in PAC.

(c)there is a need for flexibility on the part of the government while dealing with sensitive reports.

(d)Both (a) and (c).

(e)None of these can be inferred.

11.The attitude of the author towards the issue can be best described as

(a)anarchist.

(b)democratic.

(c)autocratic.

(d)moderate.

(e)logical.

Passage 3

“Indian reality,” Panikkar said, “is fast-changing, changing for the worse.” Referring to globalization and the growing power of trans-national corporations, he said: “An empire is forming before our eyes.” He said that sections of society had come to accept the dominance of the market in social relations “as an ideal” and that the situation is ripe for “the making of an uncritical mind, a conformist mindset”. Panikkar said that although the ’empire’ promised modernity and affluence, it actually promoted social obscurantism and cultural backwardness. Panikkar argued tht the forces of communalism and empire complemented each other and had commonly shared interests. He said that the Indian government’s reaction to Afghan war had clearly demonstrated the Indian ruling class’ “uncritical acceptance of the dictates of the empire”.

Panikkar said that activists working to develop a “counter-culture” had to realise that “culture is an area in which social power is exercised”. He emphasized that this action was not a cultural programme, nor was it a performance or a spectacle based on various art forms. Only social activity on a continuous basis, aimed at the “radicalization of civil society”, could develop such a counter-culture, in the process unsettling the existing equilibrium. He urged the People’s Science Movement (PSM) to bring cultural action center-stage.

Panikkar referred to three factors that impede progressive cultural action—the changing relationship between the individual and the outside world, the influence of the market and the spirit of consumerism it enforces, and the rise of religiosity and communalism. These factors create a “myopic vision that is insensitive to social reality”. The rise of the market results in the growing alienation of the individual from society. Consumerism, as the “ideology and the culture of the masses” creates a situation in which people’s material aspirations are largely unrealistic. Panikkar argued that the anxieties generated by this contradiction formed the ground in which religiosity and communalism thrive.

Panikkar suggested two areas for cultural action. In the “creative realm”, he suggested the use of art forms for cultural action. He referred to the Safdar Hashmi Memorial Trust’s (SAHMAT) effective campaign against communalism, using paintings, music, street plays and other methods. However, he said the drawback with this type of cultural action was that it was largely sporadic. Panikkar suggested that cultural action in the constructive realm would be more effective as they would be far more sustained. Action in the constructive realm would enable the formation of cultural communities. Such communities could be either imagined or local. “Imagined communities are those that internalize common values and interests and their members relate to a common origin,” he said. The intervention of Eklavya, a non-governmental organization, in the field of education in Madhya Pradesh was an example of the development of such a community, he said. Local communities constitute the other type of counter-cultural action. They are “local” not because they address local issues, but because they undertake action on issues that are universally valid. Issues such as environmental degradation are prime areas that provide the basis for the development of such a community. Panikkar said such communities offered the possibility of “continuous engagement, a necessary precondition for the creation of social consciousness.” Panikkar said that cultural action was needed to “de-ideologise civil society from the influence of globalisation and the logic of communalism.” Referring to the inherent contradiction between the “interests of globalisation and the ideology of cultural nationalism”, Panikkar said that the contradiction offered local communities the space to initiate counter-cultural action. In developing such a culture there was need to revitalize indigenous cultural resources while ensuring that they were not snared in obscurantism and revivalism, Panikkar added.

12.According to the passage, which of the following about ’local community’ is true?

I.Environmental degradation is the most important area that provides the basis for the development of a local community.

II.They don’t address local issues.

III.Local communities address only those issues that are universally valid.

(a)Only III

(b)Only II

(c)I, II and III

(d)Only I

(e)I and II

13.According to the passage, what made local communities an agent for starting counter-cultural action?

(a)Social consciousness that leads to the formation of such communities.

(b)Contradiction between the interests of globalisaton and the logic of communalism.

(c)Continuous engagement between the interests of globalisaton and the logic of communalism.

(d)Cultural nationalism with the motive for initiating counter cultural action.

(e)Mutual support between the interests of globalisation and the logic of communalism.

14.It can be inferred from the passage that

(a)influence of the market formed the ground in which religiosity and communalism thrive.

(b)consumerism forces people to aspire for largely unrealistic material aspirations.

(c)growing alienation of the individual from the society is generally preceded by consumerism.

(d)Both (b) and (c).

(e)All of a, b and c.

15.According to the passage, counter culture

(a)challenged the dominance of the empire.

(b)aimed at the radicalisation of civil society.

(c)can be developed through continuous social activity.

(d)can be developed without unsettling the existing equilibrium.

(e)is essential to maintain our cultural superiority.

16.The tone of the passage can be best described as

(a)persuasive.

(b)highly critical.

(c)analytical.

(d)exploratory.

(e)descriptive.

TEST 5

Passage 1

One phase of the business cycle is the expansion phase. This phase is a twofold one, including recovery and prosperity. During the recovery period there is ever-growing expansion of existing facilities, and new facilities for production are created. More businesses are created and older ones expanded. Improvements of various kinds are made. There is an ever-increasing optimism about the future of economic growth. Much capital is invested in machinery or “heavy” industry. More labour is employed.

More raw materials are required. As one part of the economy develops, other parts are affected. For example, a great expansion in automobiles results in an expansion of the steel, glass, and rubber industries. Roads are required; thus the cement and machinery industries are stimulated. Demand for labour and materials results in greater prosperity for workers and suppliers of raw materials, including farmers. This increases purchasing power and the volume of goods bought and sold. Thus, prosperity is diffused among the various segments of the population. This prosperity period may continue to rise and rise without an apparent end. However, a time comes when this phase reaches a peak and stops spiraling upwards. This is the end of the expansion phase.

1.Which of the following statements is the best example of the optimism mentioned in the passage as being part of the expansion phase?

(a)Public funds are designated for the construction of new highways designed to stimulate tourism.

(b)Industrial firms allocate monies for the purchase of machine tools.

(c)The prices of agricultural commodities are increased at the producer level.

(d)Full employment is achieved at all levels of the economy.

(e)Taxes are reduced in the economy.

2.It can be inferred from the passage that the author believes that

(a)when consumers lose their confidence in the market, a recession follows.

(b)cyclical ends to business expansion are normal.

(c)luxury goods such as jewellery are unaffected by industrial expansion.

(d)with sound economic policies, prosperity can become a fixed pattern.

(e)business expansion is folled by an economic depression.

3.Which of the following statements would be most likely to begin the paragraph immediately following the passage?

(a)Some industries are, by their very nature, cyclical, having regular phases of expansion and recession.

(b)Inflation is a factor that must be taken into consideration in any discussion of the expansion phase.

(c)The farmer’s role during the expansion phase is of vital importance.

(d)The other phase of the business cycle is called the recession phase.

(e)During the expansion phase, everyone is optimistic.

Passage 2

Revival of sick industries can be effective only when the rehabilitation schemes focus on organisational restructuring, and not just on financial recast. An in-house study by the BIFR reveals that promoters’ non-compliance with revival scheme provisions is a key reason for the failure of rehabilitation programmes. Some other reasons are incompetent and/or dishonest management, state and central government policies and unexpected adverse developments in the external environments. According to RBI, internal factors like project appraisal and management deficiencies account for nearly 56 per cent of the total sick and weak units. Another reason is the limited stake of the promoter. Workforce, contrary to perceptions, is not a significant factor in industrial sickness.

Most of the new class of emerging promoters has no professional experience and their tactics are mostly unprofessional. With the proposed SICA Bill, the role of BIFR should change to that of a facilitator from that of a court. But this is defined very narrowly—the BIFR would be a facilitator only for creditors and promoters of the organisation, and try to work out a solution for the debt repayment. Moreover, the Operating Agencies that formulate the rehabilitation package are financial institutions and banks. They focus only on financial restructuring. What revival schemes need is a competent professional facilitator, and sick companies, transformational restructuring. The present BIFR approach grossly fails to understand how organisations behave and work. Therefore, the high failure rate of the rehabilitation schemes, 54 per cent for under Section 17(2) and 42 per cent for under Section 18(4), till Sept, is not surprising. Further, there has been a decline in the number of references received by the BIFR by 10.63 per cent compounded annually.

Moreover, there are instances of some units being intentionally forced to deteriorate, and the same promoters floating new units in other regions. At present, for the promoters, exit is an easy and desired option. The result is that though at times the BIFR has drawn up the rehabilitation plan, the promoter has not been interested in making the unit viable. All this shows the BIFR has to undergo a drastic change that should stress on the process of change and involve organisational restructuring by the shared learning approach that is characteristic of process based consultancy.

In process consultation, the consultant helps the client to perceive, understand and to act upon process events that occur in the client’s environment. In other words, it addresses the problem solving activities of the client as opposed to the client’s actual problems. The process involves the organisation and the consultant in joint diagnosis of the problem. The process consultant would urge the CEO that he should not leap into any kind of action programme (especially if it deals with changes in structure), until the organisation itself has done a thorough assessment of the strengths and weaknesses of the present structure.

In other words, the need for a change should first emerge from the organisation itself. Since the consultant, being from outside, would not know the people, their tradition, styles and personalities, he would only assist the organisation to become a good diagnostician so that it can solve the problems on its own. This would involve a partnership approach from the unions and the workers in joint diagnosis with the management.

This would in turn, result in lower resistance to change from the unions since they are also involved in the change process. This kind of partnership is crucial in creating the stake of the workforce in the organisation and building up trust that would help turnaround organisations.

4.The revival of sick industries have failed because of which of the following reasons:

I.Rehabilitation schemes focus on organisational restructuring.

II.Rehabilitation schemes focus only on financial recast.

III.Rehabilitation schemes are not effective.

IV.Non-compliance with revival schemes.

(a)Only II & IV

(b)Only I & IV

(c)Only III & IV

(d)Only II.

(e)I and II only.

5.Industrial sickness according to the RBI, is majorly caused by all of the following, except:

(a)The limited stake of the promoter.

(b)Internal factors like project appraisal and management deficiencies.

(c)The attitude of the workforce.

(d)Lack of professional experience of new promoters.

(e)All of the above are causes of industrial sickness.

6.Which of the following is incorrect with respect to the SICA bill:

(a)The SICA bill alters the role of BIFR from a court to a facilitator.

(b)BIFR will be a facilitator only for creditors and promoters of the organisation.

(c)BIFR is the main operating agency.

(d)Financial institutions and banks formulate rehabilitation packages.

(e)BIFR has outlived its power.

7.Revival schemes, according to the passage, should

(a)Focus on organisational restructuring.

(b)Have competent professional facilitators.

(c)Work out the solution for debt repayment.

(d)Both a & b.

(e)All of these.

8.Process based consultancy, according to the passage

(a)Follows the shared learning approach.

(b)Is one in which the consultant helps the client to study the process events.

(c)Improves the problem solving activities rather than solving the actual problems.

(d)All the above.

(e)Only a and b.

9.According to the passage, need for change

(a)is created by the joint diagnosis of the problem by the organisation and the consultant.

(b)is created by the action programmes.

(c)creates awareness of the strength and weakness of the present structure.

(d)should emerge from the organisation itself.

(e)occurs when the organisation fails to reach its goals.

TEST 6

Passage 1

Henry Ford grinned at his wife Clara, while wiping his grease-stained hands on a piece of cloth. His face was aglow with excitement. It seemed to tell his dame, “I have it, dear. Ready to be tested—All that it needs is your signal.” Clara smiled while gently holding the cup with the gasoline, steady at an angle, so that the fuel could flow in at a regular pace. Ford whipped the engine out of its slumber by triggering the mechanism. The engine seemed reluctant to start. But, soon, it spluttered and as the fuel nudged, it roared loudly. The noise was deafening. Yet, to Henry and Clara it was sweet music. As soon as the engine was turned off, Henry ran into the outstretched arms of Clara. The two stood hugging each other, savouring the moment of success. Henry gently tilted Clara’s face up and peered into her eyes. For a long time, they stood, almost mesmerised. Then Clara wriggled out, while gently reprimanding him for his dirty hands and the smears they had left on her cheeks where they had run freely. Did Henry cheer her, saying “These are the signs of my first triumph?” At least, that was his impression. Henry stood and admired the enigma which would revolutionise travel. His mind flashed back to the days of childhood. His father, a farmer at Michigan had eagerly looked forward to the birth of a son. He had his plans. He would help his son learn every thing about farming. He would direct the boy, equip him with the skill to tend plants to eliminate pests and weeds, to plough the field with the help of animals. He saw only one career for his son in fact that lay in farming.

Parental expectations have a distinct slant. Rarely ever do they provide for the natural proclivities of the progeny. By and large, they manage to have their way. In the process they stifle the child’s basic talent. It is only the exceptionally strong and the extremely confident among the offsprings who fly against the set by their parents and streak their path to glory. Henry Ford did just that. He showed even when he was about five, that machines and tools were the playthings with which he could spend hours. Time seemed to stand still for him when he played with them. He would rip open a tool that had a rusted nut; he would tear apart a mechanism, which had a broken axle. He would look around for a suitable replacement. He would oil the joints, get the stains all over his clothes, run into trouble with his mother for spoiling the clothes. But all the reprimands failed to mend his ways. He occasionally went with his father around the hoe and the mechanical plough, the tools of the trade. His father, often exasperated, pleaded, cajoled, shouted and screamed. These had only temporary impact. Henry’s heart was not in farming.

Mechanical contraptions fascinated him. He would pick up a watch, gently prise the lid on the back, up with a thin wedge peer into it, watch the spring pulsating with life, and notice the wheels, which ran into each other, rotating at steady paces. The whole world for the little boy lay there. He often dismantled the watch, checked each part, understood its role in the complex assembly, got them together again after cleansing, dusting and oiling them. Henry offered to repair clocks and watches held by friends and neighbours. A neighbour joked, “Every clock in the village shudders when it sees Henry coming”. At the age of 13, he went with his father to the neighbouring town of Detroit. On the way, he noticed—the steam locomotive. This was in 1876. Henry could not take his eyes off the machine. How smoothly did the locomotive chug along? Could he not try his hand at producing something that would move on roads? Something that won’t need rails to glide along? He pleaded with the locomotive driver to wait for a few minutes. He ran over, waved his hand to the driver of the locomotive, which had come to a halt a little away at the station. The driver glared at the boy—Henry hailed him, politely requested him to explain how the locomotive worked. The driver did not know much. Yet, he decided he would amuse the boy. Henry heard with rapt attention, while the driver explained to him the main parts—the boiler, the pressure created in the boiler by water, heated by coal, the steam pressing the axles to turn the wheels. Henry sought some clarifications. The driver found it rather beyond him to answer the boy. He shooed the boy aside, and got into the engine. Henry rushed back to his father. The two moved on, Henry still wondering about the machine he would produce once he grew up, that would help people move fast on roads.

For making that dream come true, he had to fly against his father’s desire. That pained him. He loved his father, but he was not willing to mortgage his future. His mind was not in farming. He told his father bluntly that he was moving to Detroit to learn the trade of a mechanic. His father was aghast at the suggestion. Had the boy lost his head? The two stood their grounds. Fiery words flew around. The sparks of angry exchanges boomed. Then came the truce. The father gave his son reluctantly, the permission to go. There was a tiny ray of hope that the boy would come back to the sylvan settings, finally settle down at the farm. This hope sustained the old man. It was at Detroit that Henry sharpened his native talent. He would spend hours to understand the intricacies of every machine, which was assigned to him by the small firm where he was employed. Soon, he gained a reputation as a boy who could repair any machine. His reputation was matched by hard work. Henry knew that he was getting the insight into machines. He would soon break off, start work on the ’horse-less carriage’ that he wanted to produce. He was finally earning enough to live in reasonable comfort. He fell in love with Clara Byrant. He was 25. Youthful desires were coursing through him. Yet he did not want to enter wedlock unless he was sure that Clara would share his dream.

The two sat together in a lonely place, while Henry detailed his future plans. He hinted that life would not be a bed of roses. His first love would always be machines. If she could share his enthusiasm, the two together could target for fame, wealth, and recognition. In other words, what he wanted of Clara was a multiple role. Not only should Clara be his wife, but his friend, critic, associate, my Woman Friday. That was not an easy role to play. However, Clara did not bat an eyelid. She responded with warmth to his proposal. The two decided to go through life hand in hand, creating a path that had never been trekked by others. It was hard work for Henry. During the day, he worked at the Edison Illuminating Company. At night, he huddled along with Clara, at the workshop, at the rear of their humble house. Henry would have an assortment of old parts—many of them partly worn out, brought by him from the pile of discards—from which he wanted to fabricate an engine that would work on gasoline. He had read about a gasoline engine developed by Dr Nicolaus A Otto, of Germany. Henry did not miss the cue. If he could develop such an engine, he would have the horse-less carriage. He shared his excitement with Clara.

1.“Every clock in the village shudders when it sees Henry coming.” Why did Henry’s neighbour make this joke?

(a)Mechanical contraptions fascinated Henry.

(b)The whole world of the protagonist lay around machines.

(c)Henry had no interest in farming, as every other villager did.

(d)Henry always damaged all machines.

(e)None of the above.

2.The example of the engine driver is given by the author to elucidate which of the following character traits of Henry Ford?

(a)Inquisitive.

(b)Contemplative.

(c)Challenging.

(d)Perseverant.

(e)Intelligence.

3.The role of Clara in the success of Henry Ford can best be described in the following words:

(a)She was with him as a great support and a great friend.

(b)She was not exactly a part of his success.

(c)She should be given equal accolades for Ford’s success.

(d)Her role was minimal in his success.

(e)Cannot be derived from the passage.

4.“Horse less Carriage”—this was the dream of Henry Ford. He got a major cue from Dr. Nicolaus Otto. This cue was in the form of

(a)Material help which Mr. Otto gave him.

(b)The idea, which he got from Dr. Otto’s discovery.

(c)The confidence, which he got from Dr. Otto’s discovery.

(d)None of the above.

(e)All of the above.

5.“while gently reprimanding him”. What is the attitude of Clara towards Henry?

(a)She was angry with Henry because of the smears she was having on her face.

(b)She was reprimanding him because she did not like the gesture of her husband.

(c)This was a gesture that showed her happiness.

(d)Showed her condescending attitude towards Henry.

(e)None of the above.

6.What is the normal relation between parental expectations and the natural proclivities of the progeny, as per the passage?

(a)They are often on different paths.

(b)Natural proclivities superimpose the parental expectation.

(c)They have no definite relation.

(d)Parental expectations build natural proclivities or, natural proclivities are inherent.

(e)They are mutually supportive.

7.According to the author, the sign of Henry Ford’s first triumph was:

(a)Smears that were running freely on Clara’s cheeks.

(b)The loud noise which was triggered by the start of the engine.

(c)The testing of Ford’s invention.

(d)All of the above.

(e)None of the above.

Passage 2

Many scientists rely on elaborately complex and costly equipment to probe the mysteries confronting humankind. Not Melissa Hines, a behavioural scientist who is hoping to solve one of life’s oldest riddles with a toy box full of police cars, jigsaw puzzles and Barbie dolls. For the past two years, Hines and her colleagues have tried to determine the origins of gender differences by capturing on videotape the squeals of delight, furrows of concentration and myriad decisions that children from 5 to 8 make while playing. Although both sexes play with all the toys available in Hines’ laboratory at the University Of California, the experiments confirmed what parents and more than a few aunts, uncles and nursery school teachers already know. As a group, the boys favour sports cars and fire trucks, while the girls are drawn more often, to dolls and kitchen toys.

But one batch of girls defies expectation and consistently prefers boy toys. These youngsters have a rare genetic abnormality that caused them to produce higher levels of testosterone, among other hormones, during their embryonic development. On the average, they play with the same toys as boys, in the same ways and just as often.

Could it be that the high level of testosterone present in their bodies before birth has left a permanent imprint on their brains, affecting their later behaviour? Or did their parents knowing of their disorder, somehow subtly influence their choices? If the first explanation is true and biology determines the choice, Hines wonders, “Why would you evolve to want to play with a truck?”

Not so long ago, any career-minded researcher would have hesitated to ask such a question. During the feminist revolution of the 1970s, talk of inborn differences in the behaviour of men and women was distinctly unfashionable, even taboo. Men dominated fields like architecture and engineering, it was argued, because of social, not hormonal, pressures. Women had the vast majority of society’s child rearing because few other options were available to them. Once sexism was abolished, so the argument ran, the world would become a perfectly equitable, androgynous place, aside from a few anatomical details.

But biology has a funny way of confounding expectations. Rather than disappear, the evidence for innate sexual differences only began to mount. In medicine, researchers documented about heart disease; what it does to women and that women have a more moderate physiological response to stress. Researchers found subtle neurological differences between the sexes, both in the brain’s structure and in its functioning. In addition, another generation of parents’ best efforts to give baseballs to their daughters and sewing kits to their sons, girls still flocked to dollhouses while boys clambered into tree forts. Perhaps nature is more important than nurture after all. Even professional sceptics have been converted. “When I was younger, I believed that 100% of sex differences were due to the environment.” Says Jerre Levy, professor of psychology at the University of Chicago. Her own toddler toppled that utopian notion.

“My daughter was 15 months old, and I had just dressed her in her teeny little nightie. Some guest arrived, and she came into the room, knowing full well that she looked adorable. She came in with this saucy little walk, cocking her head, blinking her eyes, especially at the men. You never saw such flirtation in your life.” After 20 years spent studying the brain, Levy is convinced: “I am sure there are biologically based differences in our behaviour.”

Now that it is OK to admit the possibility, the search for sexual differences has expanded into nearly every branch of the life science. Anthropologists have debunked Margaret Mead’s work on the extreme variability of gender roles in New Guinea.

Psychologists are untangling the complex interplay between hormones and aggression. But the most provocative, if as yet inconclusive, discoveries of all stem from the pioneering exploration of a tiny 1.4 kg universe: the human brain. In fact, some researchers predict that the confirmation of innate differences in behaviour could lead to an unprecedented understanding of the mind. Some of the findings seem merely curious. For example, more men than women are left-handed, reflecting the dominance of the brain’s right hemisphere. By contrast, more women listen equally with both ears while men favour the right one. Other revelations are bound to provoke more controversies. Psychology tests, for instance, consistently support the notion that men and women perceive the world in subtly different ways. Males excel at rotating three-dimensional objects in their heads and females are better at reading emotions of people in photographs. A growing number of scientists believe the discrepancies reflect functional differences in the brains of men and women. If true, then some misunderstanding between the sexes may have more to do with crossed wiring than crossed tempers.

Most of the gender differences that have been uncovered so far are statistically speaking, quite small. “Even the largest differences in cognitive function are not as large as the differences in male and female height,” Hines notes. “You still see a lot of overlap.” Otherwise, women could never read maps and men would always be left handed. That kind of flexibility within the sexes reveals just how complex a puzzle gender actually is, requiring pieces from biology, sociology and culture. Ironically, researchers are not entirely sure how or even why humans produce sexes in the first place. Why not just one—or even three—as in some species? What is clear is that the two sexes originate with distinct chromosomes. Women bear a double dose of the large X chromosome, while men usually possess a single X and a short, stumpy Y chromosome. In 1990s, British scientists reported they had identified a single gene on the Y chromosome that determines maleness. This master gene turns on a host of other genes to the complex task of turning a foetus into a boy. Without such a signal, all human embryos would develop into girls. “I have all the genes for being male except this one, and my husband has all the genes for being female,” marvels evolutionary psychologist Leda Cosmides, of the University of California at Santa Barbara. “The only differences is which genes got turned on.”

Yet, even this snippet of DNA is not enough to ensure a masculine result. An elevated level of the hormone testosterone is also required during the pregnancy. Where does it come from? The fetuses, own undecided testes. In those rare cases in which the tiny body does not respond to the hormone, a genetically male foetus develops sex organs that look like a clitoris and vagina rather than a penis. Such people look and act female.

The influence of the sex hormones extends into the nervous system. Both—males and females produce androgens, such as testosterone, and estrogens—although in different amounts. Men and women who make no testosterone generally lack a libido. Researchers suspect that an excess of testosterone before birth enables the right hemisphere to dominate the brain, resulting in left-handedness. Since testosterone levels are higher in boys than in girls, that would explain why more boys are left-handed.

Subtle sex-linked preferences have been detected as early as 52 hours after birth. In studies of 72 new-borns, University of Chicago psychologist Martha McClintock and her students found that a toe-fanning reflex was stronger in the left foot for 60% of the males, while all the females favoured their right. However, apart from such reflexes in the hands, legs and feet, the team could find no other differences in the babies responses.

One obvious place to look for gender differences is in the hypothalamus, a lusty little organ perched over the brain stem that, when sufficiently provoked, consumes a person with rage, thirst, hunger or desire. In animals, it performs a sexual function and is somewhat larger in males than in females. But its size need not remain constant. Studies of tropical fish by Stanford University neurobiologist Russell Fernald reveal that certain cells in this tiny region of the brain swell markedly in an individual male whenever he comes to dominate a school. Unfortunately for the piscine pasha, the cells will also shrink if he loses control of his harem to another male.

Many researchers suspect that, in humans too, sexual preferences are controlled by hypothalamus. Based on a study of 41 autopsied brains, Simon Levay of the Salk Institute for Biological Studies in San Diego announced last summer that he had found a region in the hypothalamus that was on average, twice as large in heterosexual men as in either women or homosexual men. Levay’s findings support the idea that varying hormone levels before birth may immutably stamp the developing brain in one erotic direction or another.

These prenuptial fluctuations may also steer boys towards more rambunctious behaviour than girls. June Reinisch, director of the Kinsey Institute for Researches in Sex, Gender and Reproduction at Indiana University, in a pioneering study of eight pairs of brothers and 17 pairs of sister ages 6 to 18, uncovered a complex interplay between hormones and aggression. As a group, the young males gave more belligerent answers than did the females on a multiple-choice test in which they had to imagine their response to stressful situations. But siblings who had been exposed in-utero to synthetic anti-miscarriage hormones that mimic testosterone were the most combative of all. The affected boys proved significantly more aggressive than their unaffected brothers, and the drug-exposed girls were much contentious than their unexposed sisters. Reinisch could not determine, however, whether this childhood aggression would translate into greater ambition or competitiveness in the adult world.

8.According to the passage, which of the following pairs is correct?

I.Leda Cosmides, California University, Los Angeles.

II.Jerre Levy, Kinsey Institute, Indiana University.

III.June Reinisch, Chicago University.

IV.Melissa Hines, California University, Santa Barbara.

(a)I, II, III and IV

(b)I and III only

(c)II and IV only

(d)I and II.

(e)None of these.

9.According to the passage, which of the following statement is correct?

(a)Hines work confirms that boys favour sports cars & fire trucks, while the girls are drawn more often to dolls & kitchen toys.

(b)Levy is sure that there are biologically based differences in our behaviour.

(c)Levy is correct that 100 per cent sex differences were only due to the environment.

(d)Both (a) and (b).

(e)(a), (b) and (c).

10.During the feminist revolution of the 1970s,

(a)men dominated fields like architecture & engineering.

(b)women had the vast majority in society’s child rearing field.

(c)men started showing interest in child rearing activities.

(d)both (a) & (b) above.

(e)None of these.

11.The human brain, according to the passage, weighs-

(a)Less then 1.4 kg.

(b)1.4 kg.

(c)More than 1.4 kg.

(d)2.4 kg.

(e)Cannot be inferred or concluded from the passage.

12.Which of the following statements, according to the passage, is incorrect?

(a)Males excel at rotating three-dimensional objects in their heads.

(b)Females prove better at reading emotions of people in photographs.

(c)More men, than women, are left handed, which shows the dominance of the brain’s right hemisphere.

(d)Men are better at reading maps.

(e)None of these.

13.The psychologist Leda Cosmides, says—“I have all the genes for being male except this one, and my husband has all the genes for being female”. What is she talking about?

I.Women bear double doses of the large X chromosome.

II.Men usually bear a single X and a short stumpy Y chromosome.

III.There is a single gene on the Y chromosome that determines maleness.

IV.A master gene that turns a foetus into a boy, without which, all human embryos would develop into girls.

(a)I & II only

(b)II, III & IV

(c)II & IV only

(d)III & IV only

(e)I, III and IV.

14.According to the researches, as given in the passage, more boys are left-handed because:

(a)Testosterone levels are higher in boys than in girls.

(b)An excess of testosterone before birth enables the right hemisphere to dominate the brain, resulting in left handedness.

(c)Both (a) & (b) of the above.

(d)They are right brained.

(e)None of the above

15.In the studies of 72 new-borns, as done by the psychologist Martha McCIintock of the University of Chicago, it was found that

(a)A toe-fanning reflex was stronger in the left foot for 60% of the males, while all the females favoured their right.

(b)A toe-fanning reflex was stronger in the left foot for 60% of the females, while all the males favoured their right.

(c)Both of the above.

(d)Apart from the toe fanning reflex there were no major differences between boys and girls.

(e)Both a and d.

16.According to the passage, which of the combinations is correct?

I.Hypothalamus is a lusty little organ perched over the brain stem.

II.Hypothalamus, when provoked, doesn’t create any effect on the person with respect to his rage, thirst, hunger or desire.

III.Its size need not remain constant. (of the hypothalamus)

IV.Most probably, sexual preferences are also controlled by the Hypothalamus.

(a)III & IV only

(b)I, III & IV

(c)All of the above

(d)I, II and III

(e)Only I and III

Passage 3

Organisations are made of people. Without people, there can be no organisation. Where people are involved, some learning always takes place. The learning may be good or bad, but it happens all the same. In other words, organisations can and do learn, since their people can and do learn. This ability of organisations takes the shape of strategic and competitive advantage, when you begin to consider that we compete in a world full of knowledge. Not just that, there is so much of knowledge getting added each day that it is almost impossible to compete on any other basis. For sure, financial prudence and soundness helps, but that is useful only if you can compete in the first place. It therefore makes eminent sense for organisations to create an environment where lots and lots of people learn lots and lots of new things all the time. Yes, companies do recognise this, but they do mighty little about actually getting down to making it happen in big and continuous doses.

There is another completely different advantage of competing on learning. Organisational knowledge is the sum of many parts—the sum of many minds working together. This simply cannot be replicated by the competition. Why? The reason is quite simple. It is not possible to replicate the same set of circumstances and the same set of people existing in one company into another company. So, even if a few people leave and join forces with the competition, all is not lost. As a result, when discussions centre around return on investment, there is the invariable war cry for cost cutting. Such debates are common in corporate settings, and the outcome is invariably one-sided. Since the majority of costs relate to people, let’s seize a hatchet and cut the headcount. Few, far too few, senior managers think about the incredible damage they are doing by taking such an approach. No one pauses to ponder over the loss of knowledge, human capital, and loyalty.

This is where the story of the titmouse becomes relevant. Alan Wilson, a zoologist and biochemist at the University of CaIifornia at Berkeley, has been studyjng how animals learn. His research has established that there is a certain behaviour that enables primates and songbirds to share the position at the top of the table of evolved species.

Wilson’s theory for accelerated anatomical evolution describes three characteristics that enable learning:

Innovation: As individuals and as a community, they have the ability to invent new behaviour. They are capable of developing skills that enable them to exploit and take advantage of their environment in newer and better ways.

Social propagation: Skills are propagated and transferred in a proper and established way to the entire community through direct communication, not genetically.

Mobility: Individuals of the species have the ability to move around. They use this ability to a tremendous extent. They flock and move in herds, instead of keeping to themselves like hermits.

To determine whether his theory would hold water, Wilson researched studies done on the British titmouse, a small songbird commonly found in Britain. The study is extremely revealing and goes thus: During the early part of this century, milk was distributed to the doors of British country houses in bottles without tops. The cream would settle at the top of the bottles. Two species of birds—the titmice and red robin, learned to siphon the cream from the bottles and get an enriched diet. This diet was obviously richer than other food the birds had. The digestive systems of these two species underwent a metabolism to cope with the extra nutrition. By the early fifties, the entire titmouse population had learnt how to pierce the aluminium caps and get to the cream. On the other hand, the red robins simply did not learn how to pierce the caps. There was a stray robin here and a stray robin there that had learned how to pierce the cap, but the species as a whole, simply failed to learn. In other words, the knowledge was simply, not passed to all red robins. What was the difference between the two species? Basically, the titmice underwent a remarkably successful process of institutional learning, while the red robins couldn’t do so. This could not be explained as a difference in communicating ability. Both species possess the same range of ability to communicate. The difference lies in the process of social propagation—the manner in which titmice disseminate their skills between members of the community. And here is the difference. The titmice live in pairs (male and female) during spring season. They live thus until their brood grow big enough to fly and feed on their own. By the time summer arrives, the titmice can be seen hopping from one garden to another in flocks. Their propensity to flock is so powerful that the groups remain practically intact, roving the countryside. This movement pattern lasts through the summer.

On the other hand, red robins are highly territorial birds. They care for their young ones but have no ability to move as a community. They guard their turf jealously, and the only real communication that takes place is antagonistic and adversarial. Wilson concluded that birds that flock learn much faster. Moreover, everyone is able to learn. This greatly enhances their chances of survival, and speeds their evolution. The lessons for organisational learning from this study are profound, to put it mildly. How many organisations can you find where the communication is not adversarial? Territorial behaviour and turf guarding are staple diets in corporate corridors. Most organisational structures, in fact, encourage this behaviour, albeit unknowingly. The results are the same, just think, organisational charts have boxes in which people are placed. They then have functions and divisions to make things worse. This is the perfect setting for the proliferation of bureaucracy and empire-building. No wonder Hammer and Champy describe this as “The Humpty Dumpty School of Organisational Management” in their book, “Re-engineering the Corporation”. Mobility comes from moving people across functions and divisions. That may be the easier of the two criteria. The tough one is social transmission. As long as corporations are organised around functional concepts, social transmission will take place in an antagonistic manner. Here lies the key, then: CEOs who continue to organise their structures by function are doomed to head unlearning organisations. Hey, even the humble titmouse has figured that out. What is preventing the lofty man from emulating the titmouse?

17.What is the relationship between people, organisa-tion and learning?

(a)Organisations are made up of people and where people are involved, even if they are bad, some learning takes place.

(b)Organisational learning is brought about by many minds of many people working together.

(c)Organisations are made up of people and the involvement of people evolves learning and this learning can lead to competitive advantage.

(d)All of the above.

(e)Both (b) and (c) but not (a).

18.The author talks of creating an environment of learning. Why is there a need for creating this type of environment?

(a)The world in which the organisations exist is full of knowledge, so it is necessary to imbibe this knowledge.

(b)The learning ability takes the shape of competitiveness and an environment is needed to combat this competitiveness.

(c)An environment of learning is needed because in an organisational set-up, there are many minds working together.

(d)All the above.

(e)(a) and (c) but not (b).

19.Which one of the following is not an advantage of competing on learning in an organisational set up?

(a)The leaving and movement of a few people doesn’t create much difference.

(b)Knowledge is added each day and this knowledge becomes the parameter on which one competes.

(c)Competition brings financial prudence and soundness.

(d)Everyone in the organisation is able to learn and thus, exchange his or her knowledge.

(e)Both (c) and (d).

20.“Let’s seize the hatchet and cut the head count”. What is the author trying to express?

(a)Organisations stop all fresh recruitment to reduce costs.

(b)Organisations reduce the work force to cut costs.

(c)Both (a) and (b).

(d)Organisations focus on targets and cost cutting.

(e)None of the above.

21.What is the relation between Alan Wilson’s study and organisational learning?

(a)Alan Wilson’s experiment illustrates the process through which all organisational learning takes place.

(b)There is no direct relationship between Alan Wilson’s study and organizational learning.

(c)His study gives lots of lessons on organizational learning through a simple example of titmouse and red robins.

(d)Two of the above.

(e)All of the above.

22.“Titmice underwent a remarkably successful process of institutional learning, while the red robin could not do so” because:

(a)Of difference in their communicating ability

(b)Titmice move in groups.

(c)There is a difference in the method they use to propagate their skills.

(d)Two of the above.

(e)All of the above.

23.What is “the Humpty Dumpty School of Organisa-tional Management”?

(a)An institute where organisational management is being taken care of.

(b)It is a remark on the adversarial communication in an organisation, which leads to bureaucracy and empire building.

(c)Organisations encourage bureaucratic and empire-building tempers by these forms of structure of the “Humpty Dumpty School of Organisation Management”.

(d)“The Humpty Dumpty School” is a hypothetical business school.

(e)A hilarious description of a management school.

24.Which one of the following is not a part of Wilson’s theory of accelerated anatomical evolution on primates and songbirds?

(a)Their ability to invent new behaviour and capability of developing skills.

(b)Their ability of propagating their skills.

(c)Their staying in flock and possessing great mobility.

(d)All of these.

(e)None of the above.

25.What is the lesson which can be derived from the passage?

(a)Company heads should not organise structures by function.

(b)Company heads should form learning organisations.

(c)Company heads should learn from the example of Titmouse and Red robins.

(d)All of the above.

(e)None of these.

26.What does the passage advise CEOs to do?

(a)Emulate the titmouse and apply its principles.

(b)Acquire the ideas of organisational management.

(c)They should learn something about institutional learning.

(d)They should become innovative and mobile.

(e)Learn from successful experiments of other organisations.

Passage 4

TWO new books on Leonardo daVinci have covers that are almost identical. Both authors have pored over Leonardo’s notebooks, and claim to take us inside the mind of the Renaissance giant. Yet one book, a surprisingly short one, paints Leonardo as a genius, whereas the other, a doorstep of a volume, presents him uncut, looking something of a fallen angel. Which Leonardo you choose depends on whether you prefer your heroes on or off their pedestal.

Martin Kemp, an eminent Oxford art historian and Leonardo scholar, has condensed what he calls Leonardo’s “strange career” as an engineer and musician into a series of key moments. Writing his book in the Tuscan villa that was once home to a smiling housewife named Lisa, thought to be the model for the most famous painting in the world, Mr. Kemp warms to the ambience of the place before launching into the essential facts about the man. Yet, after an auspicious beginning, the book reads like a gallery guide to Leonardo, and this may be because Mr Kemp is organising a Europe-wide exhibition of Leonardo in 2006, called the Universal Leonardo Project. His is a convenient handbook for the show or for any of the 24 paintings he attributes to the artist, and the book is also worth buying for Mr Kemp’s handy timeline and illustrated list of Leonardo’s at the back. The prose, however, is more efficient than uplifting.

Charles Nicholl’s long biography of the master is more gratifying to read, yet it ties itself in knots trying to follow every lead that Leonardo, his contemporaries and a legion of scholars have left behind. The author’s goal is to show not the genius but, rather the man, and he does his best to drag Leonardo down to earth. He begins with an anecdote about a note of Leonardo’s jotted in the margins of an exposition on geometry: he is stopping work, the note explains, because his soup is getting cold. More details follow about Leonardo’s animal-loving vegetarianism and about his inability to get a job done on time. A Freudian analysis of Leonardo’s paintings of the Holy Family attempts to expose the artist’s problems with father figures (Joseph is always absent, you see). And you learn more perhaps than you might wish about the homoerotic impulses in Leonardo’s angels. That Leonardo, like Michelangelo and Botticelli, was homosexual is not news, but the lurid details of his love life may surprise some.

Such information would be more enlightening if it informed an analysis of how Leonardo became the great creative thinker we now consider him to be. Yet, the only clues about the development of Leonardo’s exceptional mind come from the fact that he was the illegitimate son of a notary; as such, he was not allowed to follow in the family business and was therefore, spared a rigid education.

Instead, he was sent with his mother to live on a Tuscan farm. There, a deep love for nature was fostered, while his mind was able to develop, unfettered to an unusual degree. Leonardo was then apprenticed to a Florentine artist, Verrocchio, at a time when many artists were interested in the fashionable new technique of perspective and oil painting.

According to both books, Leonardo’s fascination with engineering came from watching the construction of Brunelle schi’s vast dome over the cathedral in Florence. Yet, other young artists who worked in Medici Florence had also turned their hands to everything, from painting to architecture and interior decoration. So what was special about Leonardo?

First, as Giorgio Vasari said in the 16th century, Leonardo had a “heretical” state of mind. So great was his curiosity about how things worked that he would believe only what had been proved empirically before his eyes. The clandestine dissections that got him in trouble with the pope flowed logically from a desire to learn how the body worked, the better to be able to render it in art.

Second, Leonardo was obsessed with birds, and claimed that his first memory was of being visited by a red kite in his cradle. Mr Nicholl constructs intricate theories about this, finding hidden bird patterns in the folds of the skirt of the “Madonna of the Rocks”, among other places.

Mr Nicholl’s book brings the reader no closer to the nature of Leonardo’s genius, though a better understanding of the man, warts and all, does emerge—a subtler appreciation of a man, for instance, who devised war machines for the violent Cesare Borgia, yet also bought caged birds to set them free. Leonardo’s love of birds and his desire to “conquer the resistance of the air” is the lasting and original detail of this book. Metaphorically, Leonardo flew, because he was for ever asking why.

27.On the basis of which of the following does the author want us to decide our choice for a particular book?

(a)The desire to read Leonardo as a good character or a bad one.

(b)Which one of the writers was more effective.

(c)The contents of the book.

(d)Our personal preferences.

(e)Both a and d.

28.Which of these bring about the exact definition of Leonardo Da Vinci’s character, according to Martin Kemp?

(a)Leonardo had a strange career.

(b)Leonardo’s brilliance lay in his penchant for a variety of subjects.

(c)He is a genius in paintings.

(d)He is the subject of an exhibition—Universal Leonardo Project.

(e)None of these.

29.Which of these brings about the exact definition of Leonardo da Vinci’s character, according to Charles Nicholl?

(a)He was a man who made mistakes and had weaknesses.

(b)He was a down-to-earth person.

(c)He was a perverted person, indulging more in narcissism.

(d)He was a rebel.

(e)None of the above.

30.According to the paragraph, homosexuality is not present in which of the following?

I.Botticeli

II.Michelangelo

III.Leonardo da Vinci

(a)only I

(b)I & II

(c)II & III

(d)only II

(e)none of these.

31.Which of the following according to the passage, is not correct?

(a)Leonardo believed only what was empirically proved.

(b)Leonardo served as a trainee to Verrochio.

(c)Leonardo was the son of a notary.

(d)Leonardo had a historical bent of mind.

(e)none of above.

TEST 7

Passage 1

JOHN CORNWELL, author of ’A New Life of Pope John Paul II,’ would have made a fine devil’s advocate when the pope’s name is one day advanced for sainthood. Unfortunately, he will not be chosen, for John Paul II himself, some two decades ago, scrapped the custom of having a devout Catholic question the virtues of a candidate for beatification or canonisation. The old job of devil’s advocate is now, in effect, performed by committee.

Devil’s advocates were supposed to be fair-minded, and in the past, Mr. Cornwell, a prolific writer on Catholic matters, has at times been anything but fair. As he admits, “Hitler’s Pope” (1999), his biography of Pope Pius XII, lacked balance. “I would now argue,” he says, “in the light of the debates and evidence following ’Hitler’s Pope’, that Pius XII had so little scope of action that it is impossible to judge the motives for his silence during the war, while Rome was under the heel of Mussolini and later occupied by the Germans.”

Chastened by this experience, Mr. Cornwell is now a better biographer. In this ’Life of John Paul II’ he celebrates his subject’s achievements as well as deplores the mistakes. The pope’s heroism is affirmed. As a young would-be priest in occupied Poland, Karol Wojtyla was not intimidated by Nazi efforts to liquidate the Catholic clergy. A priest under Communism, he was again courageous. When the Soviet system imploded, “few would dispute that the inexorable and bloodless process had been initiated by the Polish pope.”

Unfortunately, as Mr. Cornwell sees it, the siege mentality that enabled Polish Catholics to survive persecution has carried over into John Paul II’s papacy. The pope presents himself as a reformist in the spirit of the Second Vatican Council, and indeed, he reveals himself as such in many things—in “liturgy, focus on scripture, out-reach to the world, compassion for the poor and disenfranchised.” Overall, though, John Paul II is an authoritarian rather than a collegial pope, Mr. Cornwell says. He has centralised power in the Vatican where, in his dotage, it is increasingly exercised by reactionary cardinals.

Here the biographer almost gives way to despair. The astonishing feature of John Paul’s campaign against condoms has been the lack of public dissent by the bishops, even though many bishops privately disagree with the pope. The same, the author complains, goes for the pope’s stands on married clergy, homosexuality and women priests, and for his willingness to meet such sinners as George Bush (whose re-election chances were perhaps boosted) and Tariq Aziz, then Saddam Hussein’s deputy (thereby validating a deeply nasty regime). And by resolutely strengthening the centre during his papacy, Mr. Cornwell says, John Paul II has demoralized the periphery into sullen silence.

In referring to the “periphery”, Mr. Cornwell overreaches himself. He is really interested only in those of the world’s billion Catholics who are liberal westerners like himself. John Allen, an admirably objective American journalist, has a broader perspective. The pope he notes, has to ponder not just how something will play in Peoria, but also in Pretoria, Beijing and Sao-Paulo. Westerners, especially Americans, he notes, often want to do things in their own way, and see opposition from Rome as a form of oppression. But from Rome’s point of view it often seems the reverse—it is saving the rest of the church from being involuntarily “Americanised”.

Like John Paul II, members of the Curia, the Vatican bureaucracy, strive to “think in centuries”. They believe that the Catholic church will still be around when Communism and Nazism are footnotes in history books and when George Bush and Tariq Aziz and even John Paul II are forgotten. They accept reform, but usually only after thinking about it long and hard. Mr. Cornwell’s despair is premature.

1.Cornwell’s account of Pope John Paul II can best be described as

(a)negative.

(b)balanced.

(c)flattering.

(d)sympathetic.

(e)critical.

2.According to the passage, what is the original name of the Pope John Paul II?

(a)Karol Wojtyla

(b)John Paul

(c)John Allen

(d)John Paul Allen.

(e)Can’t be determined

3.Which of the following, in the light of the information given in the passage is not a negative side of the Pope?

(a)his stand on homosexuality.

(b)his meeting with Aziz.

(c)his stand on married clergy.

(d)his validation of women priests.

(e)none of the above.

4.According to the passage, what is true from among the following viewpoints of Cornwell?

(a)There are elements of heroism in the Pope.

(b)The Pope is not an authoritarian.

(c)The Pope should not be concerned with every country of the world.

(d)The Pope does not believe in the future.

(e)The Pope does not deserve to continue.

5.According to John Allen, why is there an instinct of opposition towards Americans, from Rome?

(a)They do not want the values of the Americans to erode those of the church.

(b)Because the Americans do not like Rome’s interference in world affairs.

(c)Both a & b.

(d)Because they feel that Americans do not follow Christian values.

(e)None of these.

Passage 2

When is a grid not a grid? It depends upon whom you ask. According to many in the computer industry, grid computing which roughly means the harnessing of the collective processing power of many computers in different places, is here today, and is already widespread. Yet according to others, grid computing, while promising, is still years away from becoming a reality. Who is right?

The problem is that “grid” has been co-opted as a buzzword and applied to a number of entirely different things. The term “grid computing” was originally coined by Ian Foster of America’s Argonne National Laboratory in the late 1990s. He meant to draw an analogy between the supply of computing power and the supply of electricity, which is delivered along a wire, when you need it and with no need to worry about where it came from.

In 2002, Dr Foster drew up his own three-part definition of grid computing. A grid, he proposed, should co-ordinate computing resources that are not centrally controlled, rely on open standards, and provide more reliability than stand- alone machines. Alas for Dr Foster, his checklist immediately raised hackles within the computer industry, since much existing “grid computing” software fails to meet these criteria. Linking many small computers together to create a more powerful machine, for example, is not new, and is usually called network. For marketing purposes, however, some firms like to call it grid instead.

Similarly, grid is often confused, sometimes deliberately, for marketing reasons, with equally nebulous terms, such as utility computing, on-demand computing, autonomic computing and data-centre virtualisation. Behind all this terminology is the idea of continuously and automatically adjusting the configuration of a corporate data-centre to meet the demands made on it. But Andrew Chien, a grid pioneer at the University of California at San Diego, notes that though useful, such approaches generally eschew the harder part of the grid vision, which requires automated sharing of computing resources between different organisations, not just within one firm.

A well-known example of the sharing of computing resources across the internet is SETI@home, in which over half a million people help to sift radio-telescope readings for evidence of extra- terrestrial life using a glorified screen-saver running on their PCs. Other similar projects, such as IBM’s new World Community Grid, conduct medical research. But David Anderson, the director of SETI@home, rejects the grid label, preferring the term “public resource computing”. Others call it “internet computing” or “cycle scavenging”. While it is grid-like in some respects, this approach is very task-specific and is centrally controlled—so it is not truly grid.

Some firms, such as United Devices, sell proprietary software for cycle scavenging within a single company. Idle PCs can, for example, run drug-design software in a pharmaceuticals company or evaluate a derivatives portfolio for a financial-services firm. Early adopters of this technology claim impressive benefits. Yet since all the resources are controlled by a single organisation, purists argue that this is at best an “intragrid’, just as an intranet is a private, internal version of the internet.

What of those deliberately decentralised systems, peer-to-peer file-sharing networks? Some of them, at least, operate using open standards, and they are certainly robust: repeated attempts to close them down have failed. But they do not count as grid computing either, since they are mostly storage and distribution systems, and do not perform general purpose data-processing.

Grid computing is not entirely fictional, however: scientists have been building grids on a national or even global scale for several years. A good example is the LHC computing Grid, which links large clusters and storage systems in 87 computer centres around the world, for the benefit of particle physicists. Another example is TeraGrid, an American effort to link nine large supercomputing centres for scientific use. Even within the academic arena, though, convergence towards common standards is slow, partly because each grid project tends to reinvent the wheel. To tackle this problem, the European Union launched a major initiative called EGee this year, to provide a common grid infrastructure for scientists; America has a similar initiative.

The hope is that such projects will provide the first glimpse of “the grid”, a single global computing grid that will do for data processing what the world wide web did for online publishing. Wolfgang Gentzsch, a former grid guru at Sun Microsystems, who is now director of MCNC, North Carolina’s statewide grid initiative, says the term “grid” really refers to this ultimate goal, towards which today’s systems are merely stepping stones. But it would, he admits, be more accurate to refer to them as “grid-like” or using “grid technology”.

Constructing a single, global grid will mean solving difficult security, privacy and billing problems. Scientists have a tradition of sharing their results and resources, but others do not. Yet the hurdles are not so much technological as political, economic and terminological. The dream of a single grid, akin to the web in its simplicity and pervasiveness, still seems a long way off—as does agreement about what “grid” really means.

6.According to the passage, which of the following definition of clustering is correct?

(a)Harnessing of the collective processing power of many computers in different places.

(b)Linking of small computers.

(c)Maintenance of grids.

(d)Maintenance of small computers.

(e)None of the above.

7.According to the passage, a grid should not be mistaken as

(a)utility computing.

(b)on-demand computing.

(c)data centre virtualisation.

(d)Autonomic computing.

(e)All of the above.

8.According to the passage, which of the following benefit particle physicists?

(a)LHC computing grid.

(b)United Devices.

(c)IBM.

(d)Argon National laboratory.

(e)SETI @ home.

9.What is the author’s assertion in the passage?

(a)Grid computing’s biggest problem is that nobody knows for sure, what it is.

(b)Grid computing is the next big thing in computers.

(c)In the present scenario, it will be extremely difficult to put grid communication in practice.

(d)Grid computing is an unknown thing.

(e)None of the above.

10.Which of the following in the context of the passage, is correct?

(a)Grid computing is not very far away from reality.

(b)The New World Community Grid conducts research on unidentified terrestrial subjects.

(c)EGEE provides common grid infrastructure for scientists.

(d)SETI @ home is a project for explaining the definition of grid computing to people.

(e)None of the above.

Passage 3

TOM WOLFE’S new novel about a young student, “I am Charlotte Simmons”, is a depressing read for any parent. Four years at an Ivy League university costs as much as a house in parts of the heartland—about $120,000 for tuition alone. But what do you get for your money? A ticket to “Animal House”.

In Mr. Wolfe’s fictional university, the pleasures of the body take absolute precedence over the life of the mind. Students “hook up” (ie, sleep around) with indiscriminate zeal. Brainless jocks rule the roost, while impoverished nerds are reduced to ghost-writing their essays for them. The university administration is utterly indifferent to anything except the dogmas of political correctness (men and women are forced to share the same bathrooms in the name of gender equality). The Bacchanalia takes place to the soundtrack of hate-fuelled gangsta rap.

Mr. Wolfe clearly exaggerates for effect (that’s kinda, like, what satirists do, as one of his students might have explained). But on one subject he is guilty of understatement: diversity. He fires off a few predictable arrows at “diversoids”—students who are chosen on the basis of their race or gender. But he fails to expose the full absurdity of the diversity industry.

Academia is simultaneously both, the part of America that is most obsessed with diversity, and the least diverse part of the country. On the one hand, colleges bend over backwards to hire minority professors and recruit minority students, aided by an ever-burgeoning bureaucracy of “diversity officers”. Yet, when it comes to politics, they are not just indifferent to diversity, but downright allergic to it.

Evidence of the atypical uniformity of American universities grows by the week. The Centre for Responsive Politics notes that this year, two universities—the University of California and Harvard—occupied first and second place in the list of donations to the Kerry campaign by employee groups, ahead of Time Warner, Goldman Sachs, Microsoft et al. Employees at both universities gave 19 times as much to John Kerry as to George Bush. Meanwhile, a new national survey of more than 1,000 academics by Daniel Klein, of Santa Clara University, shows that Democrats outnumber Republicans by at least seven to one in the humanities and social sciences. And things are likely to get less balanced, because younger professors are more liberal. For instance, at Berkeley and Stanford, where Democrats overall outnumber Republicans by a mere nine to one, the ratio rises above 30 to one among assistants and associate professors.

“So what”, you might say, particularly if you happen to be an American liberal academic. Yet the current situation makes a mockery of the very legal opinion that underpins the diversity fad. In 1978, Justice Lewis Powell argued that diversity is vital to a university’s educational mission, to promote the atmosphere of “speculation, experiment and creation” that is essential to their identities. The more diverse the body, the more robust the exchange of ideas. Why apply that argument so rigorously to, say, sexual orientation, where you have campus groups that proudly call themselves GLBTQ (gay, lesbian, bisexual, transgendered and questioning), but ignore it when it comes to political beliefs?

This is profoundly unhealthy per se. Debating chambers are becoming echo chambers. Students hear only one side of the story on everything from abortion (good) to the rise of the West (bad). It is notable that the surveys show far more conservatives in the more rigorous disciplines such as economics than in the vaguer 1960s “ologies”. Yet, as George Will pointed out in the Washington Post this week, this monotheism is also limiting universities’ ability to influence the wider intellectual culture. In John Kennedy’s day, there were so many profs in Washington that it was said the waters of the Charles flowed into the Potomac. These days, academia is marginalised in the capital—unless, of course, you count all the Straussian conservative intellectuals in think-tanks who left academia because they thought it was rigged against them.

Bias in universities is hard to correct because it is usually not overt: it has to do with prejudice about which topics are worth studying and what values are worth holding. Stephen Balch, the president of the conservative National Association of Scholars, argues that university faculties suffer from the same political problems as the “small republics” described in Federalist 10: a motivated majority within the faculty finds it easy to monopolise decision-making and squeeze out minorities.

The question is what to do about it. The most radical solution comes from David Horowitz, a conservative provocateur: force universities to endorse an Academic Bill of Rights, guaranteeing conservatives a fairer deal. Bills modelled on this idea are working their way through Republican state legislatures, most notably Colorado’s. But even some conservatives are nervous about politicians interfering in self-governing institutions.

Mr. Balch prefers an appropriately Madisonian solution to his Madisonian problem: a voluntary system of checks and balances to preserve the influence of minorities and promote intellectual competition. This might include a system of proportional voting that would give dissenters on a faculty more power, or the establishment of special programmes to promote views that are under-represented by the faculties.

The likelihood of much changing in universities in the near future is slim. The Republican business elite doesn’t give a fig about silly academic fads in the humanities so long as American universities remain on the cutting edge of science and technology. As for the university establishment, leftists are hardly likely to relinquish their grip on one of the few bits of America where they remain in the ascendant. And that is a tragedy not just for America’s universities but also for liberal thought.

11.In the passage, Tim Wolfe’s novel, “I am Charlotte Simmons” could be at best described as

(a)hated.

(b)a mockery of the present American educational pattern.

(c)a funny book.

(d)a book dealing with diversity.

(e)a satire about educational institutions.

12.Which of the following, in the light of the information given in the passage, is not an explanation of diversity?

(a)Students coming from diverse regions.

(b)Students of the two genders.

(c)Getting admission on the basis of race/gender.

(d)Both (a) & (b).

(e)(a), (b) and (c).

13.The Madisonian solution according to Mr. Balch in the passage, speaks about

(a)a system of checks and balances.

(b)preservation of the minority influence.

(c)promotion of intellectual competition.

(d)all of the above.

(e)a system of proportional voting.

14.Which of the following is incorrect according to the passage?

(a)Political parties get donations from Universities.

(b)Diversity enhances speculation, experiment and creation.

(c)Tom Wolfe’s novel “I am Charlotte Simmons” is a great book for anybody.

(d)Universities are likely to change fact due to new legislation.

(e)None of the above.

15.Which of the following best describes the style of the author?

(a)Empirical.

(b)Theoretical.

(c)Prosaic.

(d)Practical.

(e)Unplanned and diverse.

Passage 4

For most people, the family is the most sacred part of private life, and therefore, the bit they are keenest to keep the state away from. That is why the idea that the British government has a child-care strategy sounds so immediately repellent. Surely the state already encroaches far too far in our lives: do we really want it playing with our babies? Yet in this area, it is worth curbing a natural aversion to government interventionism: child-rearing is part of the state’s business.

Children are the focus of much of the British govemment’s current hyperactivity. It has targeted its anti-poverty measures at families with children and is now extending pre-school education and state-supported child care for working parents. It is also spending heavily on Sure Start, an expensive scheme focused on helping poor children.

The government has got into the nannying business for both good and bad reasons. The first bad reason is its determination to envelop everybody in the warm embrace of a Scandinavian-style welfare system. Its child-care policies are the sharpest illustration of the plan to extend tax funded benefits up as well as down the income scale by providing pre-school education for all and also, increasingly, keeping schools open after hours to offer child care to hard-pressed parents. In the background lies the idea that by providing the well-off as well as the poor with tax-funded benefits which they will want to hang on to, Labour will make it that much harder for a future Tory government to roll back the state.

The second bad reason is to get women out to work. The government has been pushing them in that direction for years, and providing child care will give them an extra shove. The government has a clear interest in getting mothers back to work—they will boost economic growth and tax revenue—but families, not governments, should decide whether mothers trade their time with their children for cash.

The only good reason for the state to intervene in child-rearing is that the youngsters themselves may benefit. If intervention substantially improves the prospects of the children concerned—and the gain must be substantial, to override the presumption that parents make the best parents—then that is justification in itself. Moreover, in that case, there would be further benefits for society at large. If children are troublesome, they make life hard for those they are taught with. If they grow up criminal, they will steal the cars of those who live near them. If they grow up ill-educated, they will contribute less to the economy.

Do pre-school programmes actually work? The answer is nuanced. While high-quality, part-time pre-school education (for three- and four-year-olds) seems to help children cope with school, dumping babies in nurseries all day long before they are one is also pretty clearly bad for them. So the government needs to temper its enthusiasm for universal child care and working mothers.

And what about targeting children whose parents are too poor, too reckless or too drugged to give them a decent up-bringing? Unfortunately, such schemes are not the panaceas they were once thought to be. Head Start, America’s programme for pre-school education for three- and four-year-old disadvantaged children has consumed $66 billion over four decades, and nobody really knows whether it has done any good. That is why the General Accounting Office has commissioned a large-scale study to answer the question. Sure Start, the version the British government is pushing, looks more promising—it starts earlier, is more intensive and involves mothers, which evidence suggests are all important—but as it rolls out, the British government needs to pay closer attention than Americas had to, whether this is a good way to spend money.

Those who fear letting governments further into their family life are rightly suspicious: governments are not to be trusted. Nor, sadly, are some parents. If the state can act to improve children’s prospects, and hence society’s prospects too, it should. The government’s child-care strategy is not wrong in principle, but ministers also have to show that it will work, and that remains to be seen.

16.Why according to the passage, should the government not interfere in child rearing?

(a)because only mothers can understand babies.

(b)because as it is the government has too much to handle.

(c)family is a holy part of one’s private life.

(d)the government simply does not have resources for it.

(e)none of the above.

17.Which of the following is/are the feature(s) of the child-care scheme that are described by the author as detrimental one(s)?

(a)It engulfs everyone in the welfare system.

(b)It wants women to go and work outside.

(c)Either (a) or (b).

(d)Both (a) and (b).

(e)Neither (a) nor (b).

18.Which of the following in the context of the passage, is correct?

(a)Sure start is a cost friendly scheme to help poor children.

(b)Child care policies will curb benefits from tax funds.

(c)Women must work to encourage economic growth and tax revenue.

(d)Women should leave child rearing responsibilities to the govt.

(e)None of the above.

19.What according to the author, is the way through which the government’s child care programmes be really beneficial?

(a)The people who actually run them should be responsible.

(b)Pumping in more money into programmes such as Sure Start.

(c)Educating parents on the necessity of such programmes.

(d)Involving women to run such programmes.

(e)None of these.

20.Which of the following best describes the author’s stand on child-care strategies by the government?

(a)supportive.

(b)critical.

(c)sarcastic.

(d)philosophical.

(e)Balanced.

TEST 8

Passage 1

Penetrating studies carried out so far suggest conclusively that the human brain is the most unexplored and mystifying territory which would baffle scientists for quite a long time yet. Dr. V. S. Ramachandran, Professor and Director of the Center for Brain and Cognition at the University of California, Santiago, in his recent presentation of the subject at the Apollo Hospital, gave some tantalizing glimpses of the ways in which the brain behaves and responds for dictating behaviour and which he has dealt with in absorbing detail in his ’Phantoms in the Brainwidth’ with his co-author, Sandra Blakeslee. The picture which emerges is of a mocking, teasing presence inside the human head, submitting itself to the exacting demands of Einsteins on the one hand and remaining hopelessly beyond the reach of morons on the other. If, as Dr. Ramachandran has pointed out, India’s achievements have ranged from the realistic to the abstract, it is an indication of the powers locked up in the brain which could throw up glittering images of the cosmic dance of Shiva brought to life in sculptures.

The diverse creativity of the human brain has enriched the world with discoveries spreading from that of the Copernican theory which replaced Earth and the planets by Sun as the center of the universe much to the indignation of the Roman Catholic Church, to the Theory of Relativity. It has led to the flight of imagination from the plays of Shakespeare to the writings of Boris Pasternak who had kept alive the longing for freedom in the Soviet Union even while remaining silent for nearly twenty years. However, it could go haywire and throw up hallucinations that are “more real than reality.” A great deal which yet remains to be known about the brain is about its right and left “hemispheres” with the former having a much broader “searchlight” than the latter. While the left hemisphere is concerned with speech, language and semantics, the right is projected by Dr.Ramachandran as the “intellectual” half for taking care of the “more subtle aspects of language, such as nuances of metaphor, allegory and ambiguity”. Any damage to either of the hemispheres could affect proper brain coordination which would look strange and despairing to a normal person. One of the lessons that are taught in elementary physics is that the image in a mirror is “as far away from the mirror as the object is in front of it.” Dr. Ramachandran mentions the case of mentally ill patients who take this description literally and try to reach the image on the other side of the mirror as the result of the inability to distinguish the real object from its image. It is an instance of a brain suffering from a disturbance to its intricately placed perceptions.

Among the oft-mentioned instances of the strange manifestations of the brain is the still very little understood mathematical genius of Srinivasa Ramanujan. The world would never have known him but for the equations mailed to the Cambridge mathematician G.H. Hardy who was initially inclined to dismiss them as scribblings of a “crackpot”. It was perhaps another case of the brain taking over at the right time when Hardy thought again of the equations and saw that no one else except Ramanujan had the imagination so far to think about them. The equations which could well have remained as just jottings on a piece of paper as they might have to most, were coming alive to Hardy to put him on a trail blazed by Ramanujan. If the brain is a teaser, it could be because it is very demanding on the geniuses who have blazed and would continue to blaze new trails in their chosen disciplines. The brain, which intrigued the caveman with images on the wall thrown up by the sunlight, continues to tease today’s cosmonauts with the beckoning, expanding space. The real wonder here, however, is the brain that comprehends it all.

1.Which of the following sentences is true, according to the passage?

(a)Dr. Ramchandran’s conclusion about the functioning of the brain is abstract.

(b)The abstraction focuses on the life in sculptures.

(c)To draw inference about the exact reaction of the brain towards different emotions is intricate.

(d)Scientists have completely worked out the brain’s intricate processes.

(e)None of these.

2.Which of these incidents show the functioning of the brain in a wide spectrum?

(a)That it leads to the flight of imagination as in the plays of Shakespeare.

(b)It can throw up things as diverse as the theory of relativity and hallucinations ’more real than reality’, at the same time.

(c)That the two hemispheres of the brain scrutinize very minutely, the different sundry affairs.

(d)Both (a) and (b).

(e)None of the above.

3.Which of the following sentences is incorrect, according to the passage?

(a)The brain of mentally ill patients fails to discern the real object from its image.

(b)The brain of mentally ill patients takes the description of Dr. Ramchandran very seriously and tries to realize it literally.

(c)The brain of mentally ill patients, due to its improper perception, has semantic confusion.

(d)All of the above.

(e)Only (b) and (c).

4.What probably could be the reason for the final selection of the equations of Ramanujan?

(a)Mr. G.H. Hardy, to whom the equations were sent, was courteously requested by Ramanujan for his kind consideration.

(b)The second hemisphere of G.H. Hardy’s brain had become active and therefore, perceived the ingenuity of the material, which he had rejected initially.

(c)Later on, the equations were considered in consonance with the methodology of Mr. Hardy.

(d)Both (a) and (b).

(e)None of these.

Passage 2

There are a few instances of diseases that have laid waste, huge tracts of forests throughout India. Caused mainly by pathogens and pests, these diseases are deadly and are capable of wiping out entire forests and plantations, causing immense economic as well as ecological loss.

Meanwhile, forest pathologists and entomologists are grappling with new maladies that are surfacing almost every year. But with meagre resources and just a few experts working on the issue, things are heading virtually towards a cul-de-sac.

Moreover, no assessment has been made so far to quantify the devastation. While large chunks of forests fall prey to maladies, it is also an opportunity for some politicians and timber merchants to cash in on it. Research and documentation on forest disease, particularly on forest pathology, began in India way back in 1929, by pioneering pathologists KD Bagchi and BK Bagchi. Although it has been eight decades since then, not much headway has been made in this direction. The forestry sector today is ailing due to its misplaced priorities, resource crunch, and mismanagement. “Forest management lacks scientific approach”, says Surendra Kumar, director of the Himalayan Forest Research Institute (HFRI), Shimla.

The scientific community involved with forest diseases is today a dispirited lot. With only a few stalwarts left in this field, forest disease is a neglected area of research. Moreover, bureaucracy is increasingly taking over the scientific institutions and scientists in most of these institutes are a marginalized group.

To top it all, there are no institutions dedicated to forest diseases. Although the ministry of environment and forests is the facilitator for such research, it is not paying enough attention to promote scientific research of forest diseases. In fact, government’s lackadaisical approach came to the fore with the Sal borer epidemic in Madhya Pradesh in 1998. While forest bureaucracy slept, the beetles merrily continued to wipe out entire tracts of precious Sal forests. Eventually, with no solution in sight, thousands of valuable trees were hacked. There were also allegations that the Sal tragedy was a chance for the timber mafia in the state to cash in on timber through the legal loophole, with the nexus of politicians.

Today, things haven’t changed one bit. India’s forest department and research institutes have yet to formulate contingency plans to face any assault of similar dimensions.

Forest diseases are elusive. Although experts claim that they know quite a lot about forest diseases, there are still aspects of the maladies that are not completely understood. Says R.S. Bhandari, entomologist in the Forest Research Institute (FRI), Dehradun, “We know about all the important pests and insects, their life cycles and their development. But there are a few diseases which remain an enigma.” According to Jamaluddin, head of the pathology department in the Tropical Forest Research Institute (TFRI), Jabalpur, “Due to micro climatic changes, we are discovering new aspects of the same disease every year. Diseases have also increased manifold.” Another FRI scientist points out that although forest diseases are increasing, there is no study to estimate the economic and ecological damage caused by these pests and pathogens.

Varying with different geophysical regions and climatic conditions, pathogens and pests are essentially responsible for the tree maladies and their mortality. When the pristine, natural and mixed forests existed, forest diseases acted as a natural control measure to check the proliferation of a particular species that could threaten the balance of the ecosystem. Perhaps, this is why forest diseases paled into insignificance in the past. But today, with shrinking forests and increasing monoculture plantations, any outbreak of disease takes on a virulent form.

To top this, changed climatic and forest patterns and environmental pollution have given rise to newer forms of forest diseases. While trees are forced to take an additional load of human induced environmental changes, the introduction of monoculture has substantially increased the problems. Whatever little we know about forest diseases today come primarily through mycology, the study of forest pathogens. Mycology explains that the prime pathological reasons for forest disease are fungi, bacteria and viruses. “Among these, fungi play a major role, while the other two are relatively less significant. There are 150 to 200 major pathological infections in central India. Out of these, only five per cent are bacterial. The rest are fungal,” says Jamaluddin.

Most of these pathogens stay close to a tree, waiting for a chance to infiltrate. Their entry points are small openings or wounds in the tree. However, invasion is not always easy. Like human beings, trees also have antibodies that fight anything alien. In case of invasion from the trunk of a tree, the sapwood acts as shield and secretes enzymes to fight pathogens. But when attacked and conquered, there are tell tale signs in the form of knotty growths or fruit bodies that are extensions of the fungi in the tree.

5.Which of the following is not happening according to the author of the passage?

(a)Prioritizing forest management and weeding out maladies have become a question of concern for the forest professionals.

(b)Research and Documentation work on the forest pathology is being carried out simultaneously to estimate the spoilage.

(c)The scientific community is feeling increasingly dispirited with the enigmatic behaviour of the forest pathogens.

(d)All of these.

(e)Both (a) and (b).

6.With which of the following is the author most likely to agree with?

(a)There needs to be a more coordinated effort towards dealing with forest diseases in India.

(b)There is a likelihood of another forest disease epidemic, similar to the Sal Borer epidemic, spreading in the country.

(c)The ministry responsible should take up a more serious view towards research in forest diseases.

(d)All of these.

(e)Both (a) and (b).

7.Which of these incidents discourages the government to formulate any kind of concrete plans?

(a)The prevalence of malpractices such as the alleged nexus of politicians with some of the forest officials.

(b)The government is not able to work in concomitance with specialists, like entomologists & pathogenists.

(c)India lacks specialists in this area of forestry.

(d)The surfacing of new maladies every year.

(e)None of these.

8.Which of these statements cannot be inferred from the passage?

(a)It is possible that the timber mafias could spread their network with help from vested interests in the political and bureaucratic brass.

(b)There are hardly any committed institutions in India, for the promotion of research in forest diseases in India.

(c)With the variation of different climatic conditions, pests responsible for forest tree degradation, disappear.

(d)Forest disease research has slowed down considerably.

(e)None of these.

9.The discussion on the present condition of forest diseases proves that ______

(a)hitherto, forestry has been a neglected area of research.

(b)a lot more needs to be done by the government for sustaining the ecological balance.

(c)there must be a cooperative endeavour by scientists, government officials, and politicians to weed out the possibilities of forest diseases.

(d)Both (b) and (c).

(e)None of these.

TEST 9

Passage 1

Why do the vital organs of the body slow down on aging? Why do older people experience sleep disorders? A new study holds the molecular machinery of the “master clock” in the brain responsible for such malfunctions during old age. The study conducted by researchers at the University of Virginia and reported in the journal ’Proceedings’ of the National Academy of Sciences, compares the working of the vital organs to a wall full of clocks with a large dominant clock controlling the synchronization of the peripheral clocks. The big clock continues to keep time, but, as it ages, its connecting signal to the smaller clocks weakens. Some of the smaller clocks eventually become desynchronized and some stop running. This weakening of the signal, rather than a problem with the central timekeeper itself, apparently is the cause of alterations in the biological timing system in aging mammals—possibly including humans, according to the researchers. This may explain why older people experience sleep disorders—the signal from the master clock in the brain has weakened, even as it keeps on ticking. This weakened output causes some of the peripheral clocks in other organs to eventually stop oscillating or to fall out of proper sync, causing sleep disruption and malaise, the report says. Lack of sleep can affect more than a person’s level of alertness. In the long term, it can disrupt the body’s metabolism, affect eating cycles, lead to declining cognitive abilities and possibly, a shortened lifespan. Sleep disorders are also associated with Alzheimer’s disease. Gene D. Block, professor of biology and one of the study’s lead researchers, was quoted in the report as saying, “Our new finding demonstrates that the molecular machinery of the master clock continues to function normally. Taken together with our earlier studies, this suggests that there may be an age-related failure of the conversion of the clock’s molecular rhythm into the electrical or humeral signals that the brain uses for communication. These weakened central signals may fail to keep some peripheral clocks appropriately synchronized or, in some cases, even rhythmic.” The scientists studied tissues from the brain and other organs of older mice and measured the activity of a gene that is a part of the biological clock. They found that the central clock in the brain, the suprachiasmatic nucleus, maintained proper periodicity and synchronization. Clocks in some peripheral organs, such as the liver and kidney of older animals, were either improperly synchronized or had lost rhythm entirely. “This new knowledge could eventually lead to new therapies for age-related desynchronization,” Block said. “Arrhythmic or improperly synchronized tissues of old animals could possibly be stimulated by a treatment to oscillate normally.”

1.Which of these factors is held responsible for the slowing down of the vital organs in old age, as laid down in the passage?

(a)Molecular machinery of the master clock and the peripheral clocks in the brain.

(b)Master clock in the brain.

(c)Weakening of the signals from the master clock in the brain.

(d)Arrhythnic oscillations of tissues of old animals.

(e)Sleep disorders.

2.Which of the following sentences is incorrect, according to the passage?

(a)The connecting signals of the big clock fail, as it ages.

(b)Alterations in the biological timing system are restricted to lower mammals only.

(c)The brain uses humeral signals, which are converted from the clock’s molecular rhythm.

(d)Both (b) and (c).

(e)None of these.

3.What was the breakthrough achieved, as a result of the scientist’s effort?

(a)The discovery of the fact that the clocks in some peripheral organs were either improperly synchronized or had lost their rhythm completely.

(b)The knowledge that the oscillation is necessary to maintain a person’s level of alertness.

(c)The observation that the brain uses humeral signals for communication.

(d)Both (a) and (b).

(e)None of these.

4.Which of the following sentences is true, in the context of the passage?

(a)The brain of a guinea pig was used for the tissue experimentation.

(b)Declining cognitive abilities may be traced to sleeplessness.

(c)The big clock is directly synchronized to smaller clocks through tissues.

(d)There is likely to be an age-related failure of the conversion of the master clock’s molecular rhythm into electrical or humeral signals.

(e)Both (b) and (d).

5.Which of the following sentences is incorrect, according to the passage?

(a)There is a similarity in the functioning of the brain and the vital organs of the human body with that of a wall full of clocks and the big clock synchronizing them.

(b)The molecular machinery of the brain is responsible for the entire malfunctioning occurring at older ages.

(c)The study of tissues from the brain and other organs of mice helped scientists discover that even though the suprachiasmatic nucleus maintained time properly, the clocks in the peripheral organs were either out of sync or had lost rhythm entirely.

(d)(b) and (c).

(e)None of these.

6.What appears to be true, in the context of new findings?

(a)Sufficient information about the bipolar disease is necessary to ward off desynchronization.

(b)Age related failure hampers the synchronization of the peripheral clocks in the respective organs.

(c)Alzheimer’s disease and sleeping disorders have a similar impact upon the human mind.

(d)Only (b) and (c).

(e)Only (a) and (b).

7.Alterations in the biological clock of ageing mammals are due to _________________.

(a)desynchronization of smaller clocks.

(b)destabilization of molecular machinery.

(c)arrhythmic functioning of the old tissues.

(d)Both (a) and (c).

(e)None of these.

Passage 2

It wasn’t every day that Patricia Torres raced down the streets of Miami at 70 m.p.h. But then it wasn’t every day that her daughter Nicole Cabezas hallucinated wildly, trying to jump out of the car, pulling off her clothes and ranting that people were following her, so this seemed like a pretty good time to hurry. Nicole, 16, had been having problems for a while now—ever since she was 14—and began closeting herself in her bedroom, incapable of socializing or doing her schoolwork, and contemplating suicide. The past few months had been different, though, with the depression lifting and an odd state of high energy taking its place. Nicole’s thoughts raced; her speech was fragmented. She went without sleep for days at a time and felt none the worse for it. She began to suspect that her friends were using her, but that was understandable, she guessed, since they no doubt envied her profound gifts. “I was the center of the universe,” she says quietly today. “I was the chosen one.” Finally, when the chosen one was struck by violent delusions—the belief that she had telekinetic powers, that she could change the colors of objects at will—Torres decided it was time to take Nicole to the hospital. Emergency-room doctors took one look at the thrashing teenager, strapped her to a gurney and began administering sedatives. She spent two weeks in the hospital as the doctors monitored her shifting moods, adjusted her meds and talked to her and her parents about her descent into madness. Finally, she was released with a therapy plan and a cocktail of drugs. Six months later, doctors at last reached a diagnosis: she was suffering from bipolar disorder. While emotional turmoil is part of being a teenager, Nicole Cabezas is among a growing cohort of kids whose unsteady psyches do not simply rise and fall now and then but whipsaw violently from one extreme to another. Bipolar disorder—once known as manic depression, always known as a ferocious mental illness—seems to be showing up in children at an increasing rate, and that has taken a lot of mental-health professionals by surprise. The illness until recently, was thought of as the rare province of luckless adults—the overachieving businessman given to sullen lows and impulsive highs; the underachieving uncle with the mysterious moods and the drinking problem; the tireless supermom who suddenly takes to her room, pulls the shades and weeps in shadows for months at a time. But bipolar disorder isn’t nearly so selective. As doctors look deeper into the condition and begin to understand its underlying causes, they are coming to the unsettling conclusion that a large number of teens and children are suffering from it as well. The National Depressive and Manic-Depressive Association gathered in Orlando, Fla., last week for its annual meeting, as doctors and therapists face a daunting task. Although the official tally of Americans suffering from bipolar disorder seems to be holding steady—at about 2.3 million, striking men and women equally—the average age of onset has fallen in a single generation from the early 30s to the late teens. And that number doesn’t include kids under 18. Diagnosing the condition at very young ages is new and controversial, but experts estimate that an additional 1 million preteens and children in the U.S. may suffer from the early stages of bipolar disorder. Moreover, when adult bipolars are interviewed, nearly half report that their first manic episode occurred before age 21; 1 in 5 says it occurred in childhood. “We don’t have the exact numbers yet,” says Dr. Robert Hirschfeld, head of the psychiatry department at the University of Texas in Galveston, “except we know it’s there, and it’s under diagnosed.” If he’s right, it’s an important warning sign for parents and doctors, since bipolar disorder is not an illness that can be allowed to go untreated. Victims have an alcoholism and drug-abuse rate triple that of the rest of the population and a suicide rate that may approach 20 per cent. They often suffer for a decade before their condition is diagnosed, and for years more before it is properly treated. “If you don’t catch it early on,” says Dr. Demitri Papolos, research director of the Juvenile Bipolar Research Foundation and co-author of The Bipolar Child (Broadway Books, 1999), “it gets worse, like a tumor.” Heaping this torment on an adult is bad enough; loading it on a child is tragic.

8.What prompted Nicole to act wildly and jump out of the car?

(a)Nicole never wanted to be administered sedatives and therefore, she scuttled off to escape medication.

(b)Nicole was mentally retarded and therefore, she had to be kept under a strong vigil.

(c)Nicole, out of wild hallucination, had the habit of reckless running, which compelled her mother to chase her.

(d)Her hallucinations which put her in an odd state of high energy.

(e)None of these.

9.Which of the following sentences is true, according to the passage?

(a)Patricia’s fragmented speech was conceivable.

(b)Nicole suspected her friends of envying her gifts.

(c)Patricia had telekinetic powers, with which she could change the colors of objects at her will.

(d)Torres was suffering from a bipolar disorder.

(e)Both (b) and (d).

10.Which of the following sentences is incorrect, as laid down in the passage?

(a)Doctors took a little time to diagnose Nicole’s condition.

(b)Bipolar disorder can be treated.

(c)Bipolar disorder took every health practitioner by surprise.

(d)The increasing rate of bipolar disorder in children has taken mental health professionals by surprise.

(e)None of these.

11.Bipolar disorder seems to have affected_________ .

(a)Children

(b)Adults

(c)Both (a) & (b)

(d)Septuagenarians

(e)All of (a), (b) and (d) alike.

12.Which of the following sentences is incorrect, according to the passage?

(a)Bipolar disorder can affect any teenager.

(b)Victims of bipolar disorder are prone to alcoholism and drug abuse.

(c)Bipolar disorder is just manic depression and not a mental illness.

(d)Bipolar disorder is increasingly striking children.

(e)None of these.

13.Which of the following sentences is incorrect, as per the passage?

(a)Nicole was administered alkaloids as first aid, in hospital.

(b)It is tough to diagnose bipolar disorder in its nascent stage.

(c)Support of parents and relatives is a must to restore a patient’s psychic equilibrium.

(d)Both (a) and (c).

(e)None of these.

14.Why is bipolar disorder considered to be serious?

(a)Because the victims can change color at will.

(b)Because the victims are prone to violent delusions.

(c)Because mental health professionals could not conclude as to who could be a victim.

(d)As victims of bipolar disorders have a very high rate of alcoholism, drug abuse and suicide, it is a serious mental illness.

(e)Both (b) and (d).

15.Which of the following sentences can be inferred from the passage?

(a)Since bipolar disorder is not considered to be a mental illness by the health practitioners, it should therefore be allowed to go untreated.

(b)Doctors are intrigued about the selective criteria of the disease.

(c)Impulsive gentlemen are more prone to bipolar disorders.

(d)The increasing rate at which bipolar disorder is afflicting younger people is a cause for concern.

(e)None of these.

Passage 3

You reach Harvard University’s biological anthropology department by climbing five flights of fusty wooden stairs in the Peabody Museum in Cambridge, Mass. It’s an old building, haunted by the remnants of long lost tribes and the ghosts of an era when anthropologists thought nothing of collecting the paraphernalia of ancestor worship, not to mention the bones of the ancestors themselves. But it’s not bones that have brought me to the Peabody today. I’ve made the climb to meet Carole Hooven, a young graduate student in biological anthropology, and Richard Wrangham, one of the world’s leading experts on chimpanzee behavior. They want to show me a collection of what look like sturdy but quite ordinary sticks. These lengths of wood have a special meaning for Hooven and for science, especially the stick that’s shaped like a divining rod with a crook at the end. The last time she saw it, in January 1999 in Uganda’s Kibale forest, it was in the hands of a big male chimp called Imoso who was using it to beat mercilessly, a female named Outamba. As a woman, Hooven felt sick at heart at the violence directed at the smaller chimp. But as a scientist, it exhilarated her. She had never read about anything like this. Trembling, she rushed back to camp to report to Wrangham. He listened in silence and then shook her hand. This was a historic moment. While there are a few scattered accounts of chimps wielding sticks against prey or predators, no one before had ever seen a wild animal repeatedly, unambiguously—and with malice and forethought—use a tool as a weapon against its own kind. Until now, devising tools to inflict pain and death seemed to be something only humans did. To be sure, lots of animals use tools. Sea otters employ rocks to crack open shellfish. Crows in New Caledonia extract insects from foliage with twigs—or even bent wire—held in their beaks. Chimps will use sticks and stones for all sorts of purposes, including flailing and throwing them to impress rivals. But for all the theatrics of these so-called threat displays, no one before now had ever seen an ape use a stick to beat another ape. There are sound evolutionary reasons for this. Predators don’t need weapons; they are weapons. During internecine wars, chimps will rip one another apart with their teeth and hands. Indeed, a good deal of evolutionary ingenuity has gone into the development of greeting and submission rituals to ward off lethal aggression. Which is why Imoso’s crooked club raised so many questions. Where did his behavior come from? Was it something chimps learned from humans, or was it behavior they developed on their own? Whatever the answer, I knew I had to find a way to get to Kibale. For me, seeing is knowing. I wanted to talk to the human witnesses and, if possible, meet the chimps themselves. When an opportunity to go to East Africa arose in 2001, I called Wrangham, who graciously invited me to stop by Kibale and gave me directions. He might have been guiding me to the local Starbucks: “Get to Fort Portal,” he said, as though this was the easiest thing in the world, “and find the cab stand. They all know the way.” As it turned out, that was all I needed to know. I arrived in Kibale one evening just as the sun was setting and introduced myself to Kathi Pieta, a graduate student who ran the research station. Over dinner, she told me a bit about the local chimp community. The so-called Kanyawara group consisted of about 50 chimps, including about 10 adult males and 17 adult females. Imoso was the top dog. Young and very aggressive, he was not very popular with the human observers, and his reputation did not improve with the discovery that he was a wife beater. The best description of the first attack comes from Hooven’s field notes. Imoso had been trying to get at Outamba’s infant Kilimi, but Outamba fended off his efforts. This seemed to enrage Imoso, who began kicking and punching Outamba. To protect her baby, she turned and exposed her back to Imoso’s fists. Here is how Hooven described what happened next: “MS [Imoso] first attacks OU [Outamba] with one stick for about 45 seconds, holding it with his right hand, near the middle. She was hit about 5 times ... he beat her hard. (The stick was brought down on her in a somewhat inefficient way ... MS seemed to start with the stick almost parallel to the body and bring it down in a parallel motion. There was a slight angle to his motion, but not the way a human would do it for maximum impact.)” After resting for a minute, Imoso resumed the beating, this time with two sticks, again held toward the middle. Imoso then began hurting Outamba in a number of creative ways, at one point hanging from the branch above her and stamping on her with his feet. To Hooven, the attack seemed interminable. Toward the end, Outamba’s daughter Tenkere, 2, rushed to her aid, pounding on Imoso’s back with her little fists. But the trouble didn’t stop there. Imoso’s behavior was observed by other chimps in the community, and he may have inspired imitators. In July 2000, Pieta watched as Imoso’s best friend, Johnny, attacked Kilimi, the infant who figured in Imoso’s earlier attack. Outamba turned to help Kilimi, whereupon Johnny turned on her. Immediately Outamba became submissive, but Johnny was not to be appeased. He picked up a big stick and started striking Outamba. “He was definitely trying to hit her,” says Pieta. “It wasn’t just flailing or accidental.” He used an up-and-down motion. The whole attack lasted about three minutes. After the chimps moved on, Pieta retrieved the stick, which now resides at Harvard. The next morning, I arose at 4:45 a.m. and joined Pieta and two trackers in search of Johnny, Imoso and the battered Outamba. After a vigorous walk we got to the area of a fruiting ficus tree near where the chimps had built their nests the night before. There were Johnny, Outamba and a number of other chimps. Imoso was not around. When I asked a tracker named Donor why Imoso had attacked Outamba, his answer was straightforward: “Imoso is just a mean chimp.” That morning, all was peaceful. The principal drama I observed was the struggle of a 3-year-old female chimp whose arms were too short to grab the broad tree trunk. When she finally found a way into the ficus via a nearby sapling, the trackers applauded. The chimps went about their feeding, and then moved off. As they melted into the brush, I asked Pieta which chimp typically made the plan for the day. As one who was familiar with the jockeying for position in the ape community, she laughed and said, “Johnny thinks he does.” In all, the researchers have documented six stick attacks (the most recent seven weeks ago). The behavior is new to science and raises intriguing questions. Why have all the victims been female? And why sticks, why not stones? Imoso could have killed Outamba by slamming her with a heavy rock. That may be precisely why they use sticks, Wrangham and Hooven speculate: to inflict hurt rather than injury. Most of the attacks have been directed at sexually active females. Whereas the males might intend to do real harm to the babies, they have nothing to gain by killing their mates. Brutal as it seems, could it be that the use of sticks signifies restraint? That is one of the mysteries Wrangham and his colleagues are trying to solve, in what they view as a snapshot of the evolutionary process in action. This may be a mirror of how we evolved culturally—by the spread of ideas that moved through our early ancestors in fits and starts. Back in New York City, I experience the familiar sense of relief that comes from returning safely home from an impoverished, disease-ravaged region. Three days later, as I drive my son Alec to nursery school, we hear a radio bulletin announcing that a plane has slammed into the World Trade Center. My son asks, “Is the plane going to be all right, Daddy?” How do I shield a 3-year-old from the enormity of what has just happened? I’m at a loss. I simply say, “I don’t think so.” We humans have ways of killing ourselves that chimps could never imagine.

16.Which of the following sentences is true, according to the passage?

(a)Outamba was physically harassed by Wrangham with sticks.

(b)In Uganda’s Kibale forest resides Imoso, a big male chimp whose riotic deeds were recorded by Hooven.

(c)Kilmi was protected by Outamba against Kathi Pieta.

(d)Imoso’s friend Johnny also learnt his behaviour and beat Outamba with a stick.

(e)Both (b) and (d).

17.Which of the following is incorrect, according to the passage?

(a)Imoso resumed beating with two sticks, injuring Outamba.

(b)Johnny picked up a big stick to attack Imoso.

(c)Imoso’s behavior inspired other chimps.

(d)Tenkere tried to save Outamba from Imoso.

(e)None of these.

18.The sticks found at Kibale are preserved at __________________.

(a)University of Illinois.

(b)Anthropological department USA.

(c)Library of Richard Wrangham.

(d)University of Harvard.

(e)Cannot be inferred.

19.Which of the following weapons is used by the crows of Caledonia to prey upon insects?

(a)Twigs

(b)Sticks

(c)Stones

(d)Foliages

(e)Rocks

20.Which of the following sentences can be inferred from this passage?

(a)Chimpanzees are better equipped to assault tribal members.

(b)Chimpanzees are more dangerous in fights than human beings.

(c)The inspirational values present among chimps are similar to those of human beings.

(d)A sense of fraternity exists among chimpanzees at the best and worst of times.

(e)Human beings are much more dangerous than Chimpanzees.

21.The passage could be described as

(a)Descriptive

(b)Topical

(c)Analytical

(d)Illustrative

(e)Hilarious.

22.Which of the following is correct, according to the passage?

(a)The real cause behind the exhilaration of a scientist named Hooven was the strange behavior of a chimp called Imoso.

(b)Violence directed against smaller chimps by Imoso was the main thrust of the scientist’s inquisition.

(c)Predators themselves, are the manifestation of weapons.

(d)Chimpanzees try to hurt each other with the intention of killing.

(e)None of these.

23.Why did Imoso use only sticks to assault Outamba?

(a)Stones could not have been efficiently used.

(b)The use of sticks was resorted to inflict hurt & not to kill.

(c)Unpredictable behavior of the chimps could not lead the scientist to any conclusion.

(d)He did not know how to use other weapons.

(e)None of these.

24.To which group did all the chimps belong?

(a)Local starbucks

(b)Kanyawara

(c)Kibale

(d)Pieta

(e)None of these

25.Which of these cannot be inferred from the incidents of Kibale?

(a)The behaviour of the chimps can be classified as evolutionary.

(b)It is possible that chimps actually imitate human beings.

(c)The use of sticks signified restraint.

(d)All of the above

(e)None of the above.

TEST 10

Passage 1

Deep in our hearts, we have an intense desire to be loved. Yet, just as deeply, we know no one really understands us, and that we are separated from those around us by differences we don’t fully comprehend. The ’Family of Man’ we long to share and belong to does not exist. We are just a bunch of desperate, lonely orphans. We would give anything to know how to be lovable. Yet, now more than ever, seemingly caring guides want to help us in our quest. A flurry of books and lectures tells us that change is mandatory for survival. While the recipes for harmony sometimes strike chords in our hearts, it is not enough to read the books or say the phrases. Only if the tools offered are actually used and do indeed make our lives better will we know that the message was correct and we understood. The majority of self-help books agree on one thing: Change is necessary for a more fulfilling life. Some say that the answers lie in justifying fears, anger and emotional suffering by holding someone else—usually a parent or spouse—responsible. Often, another race or some other force at work in our life gets the blame for the mess in which we find ourselves. But one thing is for sure: We are not responsible. ’They’ are. How many books, tapes and seminars does it take before the average hurting human being becomes so frustrated that he or she cries, ’The true path must exist because all the wise people say so, but I’ll never find it.’ Life must have more to offer, something most of us are missing. Otherwise, God has a stranger sense of humour than any comedian working today. Here’s the bad news. We can all agree that this thing we call our self, our ego, our personality, is somehow the cause of all our conflicts and negative emotions. This is the cold, hard truth: The self has no idea how to fix itself or it already would have. But no one else can fix our self. We must do it—on our own. You would have probably always suspected this—but no one would prove it because it appears to become a problem with no solution. This realisation is extremely threatening, especially to those who have tried so hard to change in the past and have been unable to do so. But the good news is that you can learn to fix the self by understanding four basic principles: (1) What the self is; (2) How the self works; (3) How the self got broken; and (4) How you can fix the self. Each of us made our self and only we can fix it. Real changes begin to happen as soon as repair begins and, as you become a better mechanic, the changes will be greater and come easier.

1.What is the central idea of the passage?

(a)Flurry of books and lectures guide us to become loveable.

(b)Our personality is the cause of negative emotions and conflicts and knowing how to fix it is crucial.

(c)Change is mandatory for survival.

(d)The root cause for all our troubles are ’others’.

(e)None of these.

2.For what does the author bestow the responsibility to other people?

(a)For justifying human sufferings.

(b)For bringing us into intricate situations.

(c)For not showing the true path.

(d)For inflicting our ’self’ with fears, anger and emotional suffering.

(e)None of these.

3.Who can fix our self?

(a)No one

(b)We

(c)God

(d)Others

(e)None of these

4.What, according to the author is required to make our life better?

(a)Changing ourselves as per the dictates of time.

(b)Renouncing our negative emotions.

(c)Actually using the tools offered to us in self-help books, tapes and seminars.

(d)Immunising ourselves against negative emotions.

(e)None of these.

5.What is the cause behind all our conflicts and negative emotions?

(a)Our self

(b)Our personality

(c)Our ego

(d)All of the above

(e)Others

6.Of what use are tapes, books & seminars for the author?

(a)They are useless as they are meant to detract us from our original path.

(b)The true path can be shown only by these media.

(c)They serve as change agents in our society.

(d)They are of use only to the extent that their principles are put to use by us.

(e)None of these.

7.Why does the author seem to insist on self-help books?

(a)They can bring change in our lives.

(b)They justify tears, anger & emotional sufferings.

(c)They help us to harmonize the chords of our heart.

(d)They give us the tools for change—provided we use them.

(e)None of these.

8.What, according to the author are the inhibiting factors in the way of leading a good life?

(a)Self help books

(b)Our ego and our personality

(c)Conflicts and negative emotions

(d)Selfishness

(e)Cannot be inferred

Passage 2

Scandalised by how Arthur Andersen could destroy thousands of e-mails and paper documents related to its audit of Enron and the energy major’s more than unconventional accounting methods? Don’t know whether to believe it was just a rogue partner acting on his own accord, or whether the lead partner on the Enron account, David Duncan, was just following instructions—a person close to Duncan told The Wall Street Journal that, on October 12, an Andersen lawyer advised the Enron auditors to follow company procedure that allows for the disposal of many documents.

Well, it’s true that past practices in themselves are no indication that they’re still being followed, but it would be instructive to go back, as I did, and read Mark Stevens’ The Big Six, which is one of the best ’’audits’’ of the shenanigans of the world’s top audit firms. Stevens’ book is replete with examples of how the Big Six have fudged, obfuscated and kept their eyes wide shut in order to please clients. Oh yes, as in the Andersen-Enron case, there’s even an example of shredding of possibly vital files. Perhaps that’s a good place to begin. It concerns Touche Ross (which merged with Deloitte Haskins to become Deloitte & Touche later), and its audit of the Beverly Hills Savings & Loan, BHSL—the sudden collapse of various S&Ls, certified as financially sound, was a big scandal in the US in the late ’80s.

Anyway, while reviewing the business of a former vice president of the BHSL, Touche was told all the files ’’were contained in eight cardboard boxes and were (BHSL told Touche)... the complete set of files... except for one box which was accidentally shredded.’’ Touche was initially sceptical about the shredding, but clearly got over these doubts quickly enough since it gave BHSL the all-clear. Later, when Touche was examined by the US Congress, Congressman Wyden was scathingly sarcastic: ’’Is the shredding machine at Beverly Hills big enough to shred an entire box of documents all at once, or do they have to feed the documents page by page?’’

It gets better. When it became clear that BHSL was having a major problem disposing of high-cost property investments, Touche simply decided to change the book-keeping, and instead of showing the apartments as investments, decided to show them as ’’equity-participation loans’’. And once these were shown as loans, BHSL showed it was getting interest and fee income from them. Problem solved, except there was no interest or fee that was actually received. Congressman Dingell later quizzed Touche on the amount of ’’equity’’ in these ’’equity-participation loans’’. What was the amount of the equity, the Congressman asked? I don’t know, replied the Touche partner. And they were the auditors.

Stevens’ most evocative story, of course, is the one about ZZZZ Best, or the carpet-cleaning business begun by Barry J. Minkow. Having built up a respectable business, Minkow decided to go public and, in order to get people interested in buying into his equity, boasted that his firm was in the lucrative insurance-restoration business—that is, he got restoration contracts from insurance firms. Minkow hired Ernst & Whinney (that later merged with Arthur Young to become Ernst & Young) to audit his firm.

Naturally, one of the first things Ernst did was to audit the insurance business. Minkow, to be fair to Ernst, cheated them. He hired an office in Sacramento, bribed the security guard to pretend he was familiar with ZZZZ’s staff, and forced Ernst to do an inspection on a Sunday when other offices were closed. Duped by an impostor, Ernst said Minkow’s business was fine, and repeated the inspections in various other ’’facilities’’. In fact, when the House Committee on Oversight began investigations, Ernst argued they couldn’t be blamed for not being able to detect such an elaborate fraud.

Fair enough, but Ernst didn’t even do basic checks like going to the buildings department in various cities to find out if the buildings that ZZZZ was helping ’’restore’’ had ever had a fire or the kind of water leakages ZZZZ claimed they’d had. Ernst had also signed a confidentiality letter preventing it from disclosing the location of the buildings ZZZZ was restoring to any third party. But, and this is critical, it also said it would ’’not make any follow-up telephone calls to any contractors, insurance companies, the building owners... involved in the restoration project’’. Congressman Ron Wyden asked Ernst how it proposed to do an independent audit with such restrictions on it? Ernst’s behaviour gets curiouser. It appears someone told Ernst the ’’restoration’’ job it had inspected in Sacramento was a fake, but even then the audit firm didn’t feel the need to revisit the Sacramento site. The charge about the restoration being fake, it appears, was withdrawn, but Ernst itself found evidence that ZZZZ had made payments to the individual who made and then withdrew the complaint!

While you’re following every twist and riveting turn in the Andersen saga, be sure to compare them with those catalogued by Stevens. It promises to be both an interesting and frightening exercise.

9.It can be inferred from the passage that

(a)Ernst conducted an independent audit under the influence of bribe.

(b)Ernst attempted to conduct an independent and thorough audit but was prevented from doing so.

(c)Ernst’s audit lacked professionalism.

(d)Ernst’s audit was a complete fraud.

(e)Both c and d.

10.According to the passage, all of the following are not true except—

(a)Arthur Andersen destroyed Enron’s document on it’s own accord.

(b)Andersen and Enron were in close complicity in destroying Enron’s documents.

(c)Andersen was against the concept of selective destruction of documents.

(d)Andersen was not averse to the practice of selective disposal of documents.

(e)Destroying documents is a standard audit practise.

11.The author of the book The Big Six

(a)has given the author the requisite insight to look at the various unprofessional modus operandi adopted by audit firms.

(b)has described the different frauds associated with the different methods of audits adopted by six big audit firms.

(c)has made a critical analysis of some of the top audit firms’ bad practices.

(d)declares Andersen’s audit as the best.

(e)has written a book on audit firms best practises.

12.According to the author, which of the following could have been the main cause behind Arthur Andersen’s dubious shredding of vital files in the case of Enron?

I.A lead partner acting on instructions.

II.Complicity between Arthur Andersen, the audit firm, and Enron.

III.A rogue partner acting on his own.

(a)I & II only

(b)All three

(c)I & III only

(d)III only

(e)II and III only.

13.According to the passage, big audit firms

(a)were often misled and cheated by their clients

(b)used business practices that were aimed to satisfy their clients.

(c)were ignorant about the motives of their clients.

(d)were innocent victims of corporate fraud.

(e)None of these.

◊ Answer Key

Test I

Passage 1

1. (a)

2. (d)

3. (d)

4. (c)

5. (c)

Passage 2

6. (d)

7. (b)

8. (c)

9. (c)

10. (a)

Passage 3

11. (c)

12. (d)

13. (d)

14. (a)

15. (d)

Test II

Passage 1

1. (c)

2. (a)

3. (b)

4. (c)

5. (c)

6. (c)





Passage 2

7. (c)

8. (b)

9. (d)

10. (e)

11. (a)

12. (e)





Passage 3

13. (d)

14. (d)

15. (a)

16. (a)

17. (b)

Test III

Passage 1

1. (d)

2. (d)

3. (a)

4. (d)


Passage 2

5. (a)

6. (c)

7. (c)

8. (b)

9. (b)

10. (e)





Test IV

Passage 1

1. (b)

2. (d)

3. (b)

4. (d)

5. (a)

6. (a)





Passage 2

7. (a)

8. (b)

9. (b)

10. (c)

11. (b)

Passage 3

12. (b)

13. (b)

14. (b)

15. (c)

16. (a)

Test V

Passage 1

1. (b)

2. (b)

3. (d)



Passage 2

4. (d)

5. (c)

6. (c)

7. (d)

8. (d)

9. (d)





Test VI

Passage 1

1. (a)

2. (a)

3. (a)

4. (b)

5. (c)

6. (a)

7. (b)




Passage 2

8. (e)

9. (d)

10. (e)

11. (b)

12. (e)

13. (d)

14. (a)

15. (a)

16. (b)


Passage 3

17. (c)

18. (c)

19. (c)

20. (c)

21. (c)

22. (d)

23. (b)

24. (e)

25. (d)

26. (a)

Passage 4

27. (e)

28. (e)

29. (a)

30. (e)

31. (d)

Test VII

Passage 1

1. (b)

2. (a)

3. (e)

4. (a)

5. (a)

Passage 2

6. (b)

7. (e)

8. (a)

9. (a)

10. (c)

Passage 3

11. (b)

12. (d)

13. (d)

14. (c)

15. (a)

Passage 4

16. (c)

17. (d)

18. (e)

19. (a)

20. (b)

Test VIII

Passage 1

1. (c)

2. (b)

3. (b)

4. (e)


Passage 2

5. (d)

6. (d)

7. (e)

8. (c)

9. (c)

Test IX

Passage 1

1. (c)

2. (b)

3. (a)

4. (e)

5. (b)

6. (b)

7. (c)




Passage 2

8. (d)

9. (b)

10. (c)

11. (c)

12. (c)

13. (a)

14. (d)

15. (d)



Passage 3

16. (b)

17. (b)

18. (d)

19. (a)

20. (e)

21. (d)

22. (c)

23. (b)

24. (e)

25. (d)

Test X

Passage 1

1. (b)

2. (e)

3. (b)

4. (c)

5. (d)

6. (d)

7. (d)

8. (c)



Passage 2

9. (e)

10. (d)

11. (c)

12. (b)

13. (b)

Latest pattern comprehension passages for the cat

TEST 1

Passage 1

Aeschylus was only the first of the great classic Greek dramatists. The generation that followed brought several other talented and powerful tragic playwrights. The most famous of these were Sophocles and Euripides. Both writers did much to expand the traditionally simple expectations of characters in plays. Sophocles’ plays feature incredibly strong-willed, modern characters who struggle with serious moral concerns, and Euripides wrote some of the most chilling psychological dramas of classical Greece.

Comic theatre also had its hero. A generation younger than Sophocles and Euripides, the comic playwright Aristophanes is today regarded as the best of the classic comic playwrights. His sometimes absurd, and sometimes elegant plays demonstrate a great skill in mixing comedy with honest and sincere ideas. Aristophanes’ plays certainly contain many wild and vulgar jokes, yet they also discuss serious concerns of his time, such as politics, art, and education. His work provides modern readers with some of the clearest images of the life and daily affairs of fifth-century Greeks. This literary revolution in the early part of the fifth century was not limited to the stage. The world of philosophy also experienced an awakening. Skills such as oration, logic, and rhetoric were polished, and the study of mathematics and the sciences was encouraged. Philosophers such as Socrates became famous for wandering about the city of Athens, engaging students and scholars in lively academic debates about morality and politics. Socrates’ most famous pupil, Plato, wrote a number of books and dialogues in which he explains the philosophy and thoughts of his teacher, and goes on to develop the ideas further.His work—which deals with a wide range of theoretical concerns, such as politics, ethics, and the importance of friendship—is invaluable for modern scholars who wish to understand the ancient Greek mindset.

Another very well-known philosopher of the time was Aristotle. Aristotle was, in his turn, a pupil of Plato’s, and is best known for his numerous texts dealing with the natural sciences. Interested in almost every conceivable topic, Aristotle left behind texts detailing his thoughts and questions on physics and astronomy, as well as meteorology, sleep patterns, raising animals, and numerous other subjects.

The age of classical Greece drew to a close with the end of the fourth century BC. Yet through its literature, classical Greece still has an enormous effect on modern Western thought and traditions. Almost all Western literature, from histories to romance novels, from thrillers to poetry, is rooted in classical Greek traditions. Many of the themes present in those ancient plays and poems are in fact still popular in modern literature. And even those plays that deal with uniquely modern themes still owe much in the way of technique and style to the golden age of Greece.

Q.1. How did Euripides contribute to the development of tragic theatre?

1.He wrote chilling stories about strong-willed, modern characters.

2.He expanded the expectations of characters in plays.

3.He wrote very disturbing plays.

(a)Only 1 and 2  (b) Only 2

(c)Both 2 and 3  (d) Only 1

Q.2. Which of the following best describes the plays written by Aristophanes?

1.Funny and light-hearted, yet without purpose

2.Serious and intellectual, discussing political ideas of the day

3.Humorous while still putting across serious ideas

(a)Only 1  (b) only 2

(c)Only 3  (d) Both 2 and 3

Q.3. Which of the following best describes the subjects that Aristotle wrote about?

a.Meteorology

b.Sleep disorders

c.Astronomy

d.All kinds of natural sciences and more.

Passage 2

Up at the League, says a friend, there had been one night a brisk conversational discussion, as to what would happen on the Morrow of the Revolution, finally shading off into a vigorous statement by various friends of their views on the future of the fully-developed new society.

Says our friend: Considering the subject, the discussion was good-tempered; for those present being used to public meetings and after-lecture debates, if they did not listen to each others’ opinions (which could scarcely be expected of them), at all events did not always attempt to speak all together, as is the custom of people in ordinary polite society when conversing on a subject which interests them. For the rest, there were six persons present, and consequently six sections of the party were represented, four of which had strong but divergent anarchist opinions. One of the sections, says our friend, a man whom he knows very well indeed, sat almost silent at the beginning of the discussion, but at last got drawn into it, and finished by roaring out very loud, and damning all the rest for fools; after which befell a period of noise, and then a lull, during which the aforesaid section, having said good-night very amicably, took his way home by himself to a western suburb, using the means of travelling which civilisation has forced upon us like a habit. As he sat in that vapour-bath of hurried and discontented humanity, a carriage of the underground railway, he, like others, stewed discontentedly, while in self-reproachful mood he turned over the many excellent and conclusive arguments which, though they lay at his fingers’ ends, he had forgotten in the just past discussion. But this frame of mind he was so used to, that it didn’t last him long, and after a brief discomfort, caused by disgust with himself for having lost his temper(which he was also well used to), he found himself musing on the subject-matter of discussion, but still discontentedly and unhappily. “If I could but see a day of it,” he said to himself; “if I could but see it!”

Q.1. The man in the carriage was:

1.Upset with himself at having lost his temper.

2.Angry because he had forgotten to mention so many important points which would have provided buttress to his argument.

3.Was thinking about the topic of discussion

(a)Both 1 and 2  (b) 1, 2 and 3

(c)Both 2 and 3  (d) Only 3

Q.2. The person in the discussion who damned everyone :

1.Parted happily.

2.Parted after saying a friendly goodbye

3.Parted after cursing and walked away angrily.

(a)Only 1  (b) Only 2

(c)Only 3  (d) Only 1 and 2.

Passage 3

What is an Assembly? It is a structure that brings the people of an entire community into a systematic relationship with one another for the purpose of solving individual and community problems. It is the organization of a community in such a manner that if a great many people wanted to do a particular thing, they could do it. It is a means—through its formal structure and logical, systematic organization—of connecting masses of people; a means, through such systematic organization, of establishing a channel of communication between the Assembly’s elected leadership and every last man and woman in the community; a means by which large numbers of people may engage in a common effort.

The Assembly follows certain rules of logic: in order for masses of people to enter into collective decision-making they must be organized prior to any decisions having been made. This means that for an anti-poverty effort to succeed, structure must be established prior to the creation of any program; otherwise it is likely that programs will be designed and controlled by persons outside the community.

We go into a community, and we divide it into Conferences of manageable size. Each Conference meets on a regular basis, and each sends a Representative to a central decision-making body called the Assembly. Within the Assembly, each county or city is organized like a country, but instead of unwieldy Congressional districts there are districts of fifty people—or in cities, fifty households. These districts are called Conferences.

The representational dimension of the Assembly enables it to function community-wide. Each Conference elects one Representative to the Assembly. Thus, if there are five thousand adults (in cities, five thousand households) in a given community, there will be one hundred Representatives for one hundred groups of fifty people (in cities, fifty households). If there are ten thousand adults (or households), there will be two hundred Representatives, and so forth. Representatives meet at a community-wide assembly to discuss problems.

Communication is the key to success. Each Representative is in touch with fifty members through seven committee-persons, each of whom in turn communicates with six other members (see diagram). And it works! What we’ve done is transform entire communities.

The conference provides a setting in which citizens can bring problems to the group for discussion and creative problem-solving. Problems range from disparities in government services to the need for job training to how to file for Social Security benefits to school problems confronted by a member’s children. Community members can fill out a problem sheet to address individual concerns. The sheet is given to their representative. If the representative cannot solve the problem, it is passed on to the Executive Council, which is made up of a dozen chairpersons assigned to areas such as employment, social services, housing, etc.

The assembly is the heart of the NASP approach to community transformation. It grapples with issues affecting the county as a whole. Its actions often result in new and expanded educational programs, low-income housing projects, improved county facilities such as all-weather roads and modern sanitation systems, and enhanced health, welfare, and recreational services.

Q.1. Which of the following questions would you ask the author of this piece?

1.Why is the conference a systemic organisation ?

2.Why is the assembly a systematic organisation?

3.Why does the action of the conference affect the county as a whole?

(a)Only 1  (b) Only 2

(c)Only 3  (d) Both 1 and 2

Q.2. Which of the following is correct according to the passage?

1.The conference aims at creative problem solving.

2.The conference transforms the communities.

III.The communities are the districts of the assembly.

(a)Only 1  (b) Only 2

(c)Only 3  (d) Both 1 and 2

Q.3. Which of the following logics applies to the working of the Assembly?

(a)The masses of people can solve the problem once they have identified the problem, picked out the process of problem solving and then organized themselves into groups.

(b)Before a collective decision is made, the masses of people have to be organized into a structure.

(c)The programs are controlled by people from outside the community.

(d)The assembly grapples with the problems of the community as a whole.

TEST 2

Passage 1

It was the best of times,

It was the worst of times,

It was the age of wisdom,

It was the age of foolishness,

It was the epoch of belief,

It was the epoch of incredulity,

It was the season of Light,

It was the season of Darkness,

It was the spring of hope,

It was the winter of despair,

We had everything before us,

We had nothing before us,

We were all going direct to Heaven,

We were all going direct the other way--

In short, the period was so far like the present period, that some of its noisiest authorities insisted on its being received, for good or for evil, in the superlative degree of comparison only.

There were a king with a large jaw and a queen with a plain face, on the throne of England; there were a king with a large jaw and a queen with a fair face, on the throne of France. In both countries it was clearer than crystal to the lords of the state preserves of loaves and fishes, that things in general were settled for ever.

It was the year of Our Lord one thousand seven hundred and seventy-five.

Spiritual revelations were conceded to England at that favoured period, as at this. Mrs. Southcott had recently attained her five-and-twentieth blessed birthday, of whom a prophetic private in the Life Guards had heralded the sublime appearance by announcing that arrangements were made for the swallowing up of London and Westminster. Even the Cock-lane ghost had been laid only a round dozen of years, after rapping out its messages, as the spirits of this very year last past (supernaturally deficient in originality) rapped out theirs. Mere messages in the earthly order of events had lately come to the English Crown and People, from a congress of British subjects in America: which, strange to relate, have proved more important to the human race than any communications yet received through any of the chickens of the Cock-lane brood.

France, less favoured on the whole as to matters spiritual than her sister of the shield and trident, rolled with exceeding smoothness down hill, making paper money and spending it. Under the guidance of her Christian pastors, she entertained herself, besides, with such humane achievements as sentencing a youth to have his hands cut off, his tongue torn out with pincers, and his body burned alive, because he had not kneeled down in the rain to do honour to a dirty procession of monks which passed within his view, at a distance of some fifty or sixty yards. It is likely enough that, rooted in the woods of France and Norway, there were growing trees, when that sufferer was put to death, already marked by the Woodman, Fate, to come down and be sawn into boards, to make a certain movable framework with a sack and a knife in it, terrible in history. It is likely enough that in the rough outhouses of some tillers of the heavy lands adjacent to Paris, there were sheltered from the weather that very day, rude carts, bespattered with rustic mire, snuffed about by pigs, and roosted in by poultry, which the Farmer, Death, had already set apart to be his tumbrils of the Revolution. But that Woodman and that Farmer, though they work unceasingly, work silently, and no one heard them as they went about with muffled tread: the rather, for as much as to entertain any suspicion that they were awake, was to be atheistical and traitorous.

Q.1. What is the tone of the passage?

(a)Argumentative  (b) Nostalgic

(c)Ironical   (d) Aesthetical

Q.2. Which one of the following options is correct according to the passage?

1.The author tries to use opposing pairs to highlight how they were equal in their struggle

2.The author tries to show how Britain was superior to France.

3.The author tries to show how it was going to be the end of the world with things so mismatched as they had never been before.

(a)Only 1   (b) Only 2

(c)Only 3   (d) None of these

Q.3. The ’humane achievements’ mentioned in the passage refers to :

1.The humane values which were upheld in those times.

2.A sarcastic comment on the system which had become totally ’inhuman’ as opposed to ’humane’.

3.The punishments and trials given to people was most humane and would be counted as an achievement.

(a)Only 1  (b) Only 2

(c)Only 3  (d) Both 2 and 3

Passage 2

Egoism can be held as a descriptive or a normative theory and Hobbes seems to have held both versions. As an empirical theory about human motivation generally termed psychological egoism, it holds that persons are always motivated by and act on the basis of what they perceive to be their self-interest and their good. As an ethical or normative theory it holds that all persons have a basic obligation to act to promote their own interests even to the detriment of others. Although Hobbes states in some of his writings that not all helping others is irrational, he does claim that sacrificing one’s life is always irrational. Virtues, for Hobbes, have no intrinsic value but only as a means to power.

An essential component of Hobbes theory of human nature is his belief in human equality. This equality is a physical and intellectual equality in that, as Hobbes puts it, the weakest, under the right circumstances, can kill the strongest.Since all persons are equal in this sense, all have an endless desire for power to satisfy their desires, and given a scarcity of power and the goods necessary to satisfy desires, competition for power and conflict are inevitable. In addition to the competition for power, the other major causes of conflict are fear of death and injury and the desire for glory. Given such an analysis of human nature a condition without government has some clear implications for Hobbes.

The condition without a common power or authority Hobbes terms the “state of nature.” Hobbes does not claim that there was ever such a state throughout the world, but he does believe it is still in existence among the independent nations of the world who have no power over them. He also believed that native Americans were still in such a circumstance.

Given the infinite desire for power, the state of nature is, for Hobbes, necessarily a state of constant war and threat of war “of every man against every man.” In such a condition, there is no industry, agriculture, private property or civilization at all. Hobbes reinforces his claim that war is the natural state of mankind in some ordinary observations of human behavior.

He notes that all take precautions against robbery and violence even in society by locking their doors and other measures for self-protection.

The state of nature Hobbes describes is far starker than in mostcontract theories being a state without any morality or justice and permeated by a constant power struggle. In this harsh state of nature there are no moral or legal limits to anything and every person has a right to everything even another’s life. Life is reduced to such a primitive, violent and crude level that Hobbes calls it “solitary, poor, nasty, brutish and short.”

Human beings are, according to Hobbes, rational creatures with an inborn desire for life and pleasure and, as such, seek a way to escape the amoral, lawless and chaotic state of nature they find themselves in. At this point in his argument Hobbes introduces the idea of the “laws of nature” as immutable laws which exist in the state of nature and which are not fundamental moral rules necessary for social order as understood by Aquinas and Locke among others, but rather precepts of prudential reason promoting survival and motivating humans to move to a more secure and comfortable life. The laws of nature give humanity what Hobbes calls the “natural right” to self-preservation. By “right” Hobbes does not mean a moral or legal notion but simply the natural liberty to act in one’s own enlightened interest.

For Hobbes, the desire for power is a basic human drive since power is the necessary means for happiness and getting what humans desire.

Consequently, the more power one has the more likely one will achieve one’s desires. Our desires are, for Hobbes, endless and so is the human need for power, which only ends in death.

Hobbes explains that individuals in the state of nature are in a state of war and so see the need to escape that turbulent state by forming some kind of central power or government. They decide to “confer all their power and strength upon one man or upon one assembly” and so make that person or group the authority to act for each person of the society. Later Hobbes qualifies this by noting that no one can or should give up the right to survival; this then is the only limit to the contract. The actual words of the contract or covenant “I authorize and give up my right of governing myself to this man or to this assembly of men on this condition, that you give up your right to him and authorize all his actions in like manner.” However, this agreement need not be unanimous but only a majority decision. If some refuse to consent to the contract, they would be destroyed since they would then be in a state of nature with respect to the others. This agreement creates a “sovereign,” a common political power, and what Hobbes calls a “commonwealth” which he also calls the “leviathan.” A sovereign created by agreement Hobbes calls sovereignty by “institution.”

However, a sovereign can also be created by conquest where one commonwealth overpowers another. For Hobbes, a contract is nothing without the power of enforcement or “covenants without the sword are but words.” Words cannot keep a contract effective but two ways are effective, the fear of the consequences of breaking the contract and the “glory” of not breaking it or needing to break it. Hence, the sovereign must have all the necessary power to enforce the contract and the people keep their contract strictly out of fear of punishment and death.

Q.1. Which of the following courses of action would Hobbes be most likely to disagree with?

(a)A soldier defending the borders of his country from the belligerent neighbour’s surprise attack has to decide whether to take on an individual maneuver on the attackers. He knows his actions would buy his country’s forces a lot of time and possibly save many lives. He is torn between going forward and retreating as he thinks of his wife and newborn daughter. He decides to withdraw and convene with the others as he rationalizes that there must be some other way out.

(b)On the way to an important meeting, a man witnesses a severe accident between a two-wheeler and a car. Both parties are severely injured and there is a large crowd gathering at the scene of the accident. He hears someone call out to him help transport the injured victims to the nearby hospital. The man pretends not have heard and drives away quickly, thinking there would be several others to come to the rescue of the victims and he could not afford to be late to the meeting.

(c)A mother takes the blame for an act of theft on the behalf of her son in order to prevent him from being punished, as she believes he is too young and was under the bad influence of his friends.

(d)A country concludes an entente with its long-standing bitter rival neighbouring country due to mutual strategic advantages that the alliances offers them.

Q.2. According to the passage, which of the following are not characteristics of human beings?

(a)A strong urge to acquire might, glory and precedence over others.

(b)The desire to escape conflict, chaos and the fear of death.

(c)A predilection for selfless action and sacrifice.

(d)The capacity for critical reasoning.

Q.3. According to Hobbes’ formulation of the pre-societal, what are the main disadvantages of the state of nature?

1.No sense of security and a fear for one’s life

2.Absence of law and order and endless power struggle

3.Equality of all with every other member of the state of nature

4.There is great possibility of progress and development of mankind

(a)1, 2 and 3  (b) Only 2

(c)Both 3 and 4  (d) Both 1 and 2

Q.4. In the context of the passage, the word ’covenant’ means which of the following:

(a)A decree  (b) An order

(c)A pact   (d) An undertaking

Passage 3

Back in 1991, India was the world’s greatest aid recipient, a patently uncompetitive giant begging for alms. Economic reforms from 1991 onwards gradually made India highly competitive, enabled it to touch 9% GDP growth, and to be called a potential superpower.

Those days are gone. GDP growth has halved to 4.5%. India has become uncompetitive in several ways. Worse, the Indian political class has stopped even trying to compete globally. It focuses on subsidies, reservations and special measures for sundry vote banks, regardless of the implications for competitiveness. This will ultimately lead to bankruptcy, not inclusive growth (as claimed by Congress spin-masters).

When India went bust in 1991, drastic measures over several years were needed to restore competitiveness. The rupee was devalued hugely, industrial licensing and MRTP clearance were abolished, trade was liberalized, import duties were gradually slashed from 300% to around 10% by 2005. Public sector monopolies like telecom and oil refining were opened up to the private sector, enabling the creation of the world’s fastest growing and cheapest telecom service, and the world’s biggest export-oriented refineries. Liberal foreign investment converted India into a global small car hub, many multinationals like GE came in and made India an R&D hub. A liberalized private sector created the software/BPO revolution.

Merchandise exports rose from 5% of GDP to 15%. Service exports rose even faster, and India became a world leader in software. The rupee strengthened from Rs 50 per dollar in 2002 to Rs 40 per dollar by 2008. India’s rising competitiveness made it a potential superpower. That’s suddenly gone. In the last three years, GDP growth has plummeted, industry and exports have stagnated, the current account deficit has widened dramatically, and the rupee has crashed. A year ago this finally persuaded the government to embark on some reforms, but these have failed to revive the old dynamism.

Why? Because the new measures do not change the political mind-set of constantly ignoring competitiveness in formulating new policies and regulations. Politicians take competitiveness for granted, and just want to divide up the spoils of growth. Alas, neglect of competitiveness has meant a collapse of growth, and hence of the spoils too.

India certainly needs new rules to safeguard the environment and tribals. But in formulating new rules and regulations, did anybody check whether our rules compared with those in competing countries? Alas no.We need a fairer land acquisition policy. But our new law mandates a social impact assessment and expert group clearance before any acquisition. Do competing nations have such time-consuming rules? Do they insist on 80% of land-owners consenting to acquisitions? Do they give workers, suppliers and other non-owners of land veto powers? Do they put the burden of rehabilitation on industries or the government?

Activist courts and NGOs have worsened matters, despite good intentions. The green tribunal has banned sand mining without environmental clearance. The result is a huge shortage of legal sand, which simply encourages illegal mining. A similar outcome is evident after ill-formulated bans on rock quarrying. It now takes 12 years to open a new coal mine. Not just illegal but even legal iron ore mining has been banned in some states. Do other Asian competitors have bans of this sort? If not, how does it affect India’s competitiveness?

Neither the courts, NGOs nor the politicians seem to care. A profusion of new rules and regulations are constantly churned out without any cost-benefit exercise to judge the impact on competitiveness. The latest Doing Business 2013 report of the World Bank says India has slipped from 131st to 134th position in ease of doing business. It stands 177th in ease of starting a business, 183rd in getting a construction permit; and 186th in enforcing contracts. Yet this damning expose of our uncompetitiveness produces no political will to change. We have a deep structural problem that is not even recognized, let alone redressed. Will India have to go bust again to concentrate the minds of politicians?

Q.1. Identify the false statement:

(a)India’s rules and regulations are inefficient in comparison to other countries and have led to time wastage, extra costs and put extra burden on the government.

(b)India has consistently been ranked as one of the more favorable nations in the world in terms of doing business and starting a business

(c)India is headed towards a crisis similar to that of the years preceding 1991

(d)India’s political class allows irrelevant political concerns to come in the way of good economic decisions

Q. 2. According to the author, competitiveness is enhanced by:

(a)Reducing the requirement for industrial licensing and clearances

(b)By promoting the creation of public sector monopolies

(c)By creating newer fair and comprehensive policies

(d)By creating effective filters for foreign investment into the country

Q.3. If you were to ask the author a question on his views on the role of competitiveness in a nation’s growth, which of the following would you choose?

(a)In your opinion, is it unnecessary for the incumbent government to consider and protect the interests of the underprivileged while formulating policies?

(b)Should India revert to the pre-liberalization conditions, given that despite the reforms of 1991, it is still not experiencing the kind of growth that had been envisioned and promised?

(c)What are the key structural changes that should come about in the India system so as to enhance competitiveness and speed up development?

(d)Are over-enthusiastic activism and petitioning solely responsible for the current bleak economic scenario?

TEST 3

Passage 1

After intense negotiations between Iran and the world powers (chiefly among them the United States), November 24 saw a historic breakthrough. In a six-month interim agreement, Tehran has committed itself to a substantial freezing of its nuclear program in return for “modest relief” — according to US President Barack Obama — in sanctions. The agreement will be a first step towards achieving a comprehensive solution, with which the peaceful nature of Iran’s nuclear program will be ensured while all sanctions against the country would be lifted.

Now, there has been much speculation over the degree in which the decade-long transatlantic Iran strategy of coercive diplomacy was responsible for reaching this diplomatic victory. Was it the permanent threats of war or the increasingly crippling sanctions which, in the eyes of many Western observers, that led Iran to “give in”?

Arguably, it rather was a shift away from that policy of threats and pressure, and towards serious diplomacy aiming at a reconciliation of interests (especially during the month of November), which rendered the deal possible. But yes, without any doubt the sanctions did have an impact.

The sanctions have severely deepened Iran’s economic malaise, considerably harmed a variety of social groups, while part of the power elite quite comfortably adjusted to the situation. Consequently, the power gap separating the state and (civil) society was even boosted.

Yet, the immense damage that sanctions have done to society does not bear much relevance for policy-makers. However, what has gone largely unnoticed by supporters of the sanctions policy is the realpolitik fact that, contrary to their stated goal, the escalation of sanctions was accompanied by an increase in Iran’s nuclear program. When Obama entered the White House, there were not even 1,000 centrifuges spinning in Iran; today, the figure stands at almost 19,000.

The reason for this is that the west views sanctions through the cost-benefit lens, according to which it can only be a matter of time until the sanctioned party will give in. In contrast, Tehran sees sanctions as an illegitimate form of coercion, which ought to be resisted, for the alternative would be nothing less than capitulation.

Nonetheless, many commentators sardonically insist on praising the sanctions’ alleged effectiveness for aiding diplomacy. This is not only a sign of analytical short-sightedness, but also constitutes the not-so-covert attempt to shed a positive light on the coercive diplomacy that was pursued so far.

In reality, Iran’s willingness to offer concessions is rooted within a wider context.

Firstly, Iran already demonstrated its readiness to compromise over the last three years, which the Obama administration did not dare to accept due to domestic political issues (i.e. his re-election.

Secondly, and this is likely to have been crucial for achieving the agreement in Geneva, Iran’s current foreign policy is primarily not a result of pressure through sanctions. Instead, it is embedded into a specific foreign-policy school of thought which is characterized by realism and a policy of détente.

Notably, with Hassan Rouhani’s election, the “defensive realist” school of thought reasserted power, which had already been leading during Akbar Hashemi Rafsanjani’s and Mohammad Khatami’s administrations. Their prime objective was a policy of détente and rapprochement, especially with the West but also with neighboring Arab states — namely Iran’s geopolitical adversary, Saudi Arabia.

In contrast to the “offensive realists” who were taking the lead under the Ahmadinejad administration, “defensive realists” do not view foreign policy as a zero-sum game but instead as an arena where win-win situations ought to be explored — especially with the United States. Another pivotal difference between these schools of thought is their estimation of US power.

While “offensive realists” see the superpower’s power-projection capabilities rapidly declining, the “defensive” camp rightly acknowledges that even a US in relative decline can inflict substantial damage on weaker countries like Iran. The historically unprecedented Iran sanctions regime is a prime example for the veracity of the latter view.

Ultimately, the nuclear agreement in its core has to be seen as an American-Iranian one, which expresses the will of both sides to secure their interests in a rapidly changing regional landscape. To what extent this will affect Washington’s traditional regional allies in Tel Aviv and Riyadh will be highly interesting to watch.

Q.1. Which of the following is the closest meaning of ’realpolitik’?

(a)A kind of politico-economic system

(b)The actual consequences of political maneuvers

(c)A political strategy focused on practical and material factors

(d)The idea that politics and reality are interlinked and should not be focused upon ideological things.

Q.2. What according to the author is the real reason for Iran ’giving in’?

(a)The inability to contain the debilitating effects of economic coercion

(b)The overall current direction of its general policy being that of reconciliation and peace-making and strategic reasoning

(c)A new flavor in foreign policy of appeasement and capitulation

(d)A failure of top political machinery in protecting the nation’s pride

Q.3. What is the key point of divergence between the offensive realists and the defensive realists?

(a)The former argues for a state’s active pursuit of power and the latter that states are best served by signaling restraint rather than aggression.

(b)Offensive realists don’t believe in peaceful resolution of conflict while defensive realists do.

(c)Offensive realism is a strategy of the power elite while the latter belongs to civil society.

(d)Offensive realists believe in the effectiveness of economic sanctions while the latter condemn them.

Q.4. The author would most likely agree with which of the following?

(a)Iran’s nuclear deal with the USA is a sign of its weakness and the effectiveness of the American economic sanctions.

(b)The USA has emerged triumphant in the long tussle between Tehran and Washington DC.

(c)Iran has displayed shortsightedness in pandering to the demands of the USA and compromised its own interests in the process.

(d)Iran has key political insight and has displayed restraint that will yield invaluable future gains.

Passage 2

Manto was in Bombay when he heard the news about Pakistan: the British, quitting India at last, had decided in their haste to let the Muslim-majority regions in the east and west become a separate country. To a man like Manto—a Muslim who grew up in Sikhism’s holiest city, who derived his livelihood from the cosmopolitan film business and had many Hindu friends—this religious designation of a landmass must have seemed unreal. “Where were they going to Inter the bones that had been stripped of the flesh of religion by vultures and birds of prey?” Manto would ask later, and not wholly rhetorically, recalling the plunder and flag-waving and killing that had broken out all around him. “Now that we were free, had subjection ceased to exist? Who would be our slaves? Thousands of Hindus and Muslims were dying all around us. Why were they dying?”

The answers were distressingly near. Even in Bombay, now a part of “secular” India, “the communal atmosphere was becoming more vicious by the day.” At the Bombay Talkies film studio where Manto was employed, Hindu staffers began sending anonymous letters to the management, demanding the removal of Muslims from high posts and threatening “everything from murder to the destruction of the studio.” Manto, suddenly a Muslim among Hindus, panicked. He “stopped going to Bombay Talkies.” He “began to drink heavily” and “all day long” would “lie on [his] sofa in a sort of daze.”

Much has been made of Manto’s decision in that difficult hour to flee to Pakistan. His friend Ismat Chughtai, herself a Muslim writer and self-avowed progressive who stayed on in India, saw in it a proof of Manto’s opportunism. She quotes him as saying that “the future looks beautiful in Pakistan” because Muslim migrants could now “get the houses of people who’ve fled from there.” But once in Lahore, now Pakistan, Manto was inconsolable. “Despite my best efforts,” he writes, “I could not dissociate India from Pakistan and Pakistan from India.” What was such a writer—one who saw maddening continuities where borders had been drawn—to do?

He could begin by putting down his most immediate impressions of the partition violence. “Siyah Hashiye” (“Black Margins”), published in October, 1948, is a collection of “sketches,” some as short as a paragraph: in “Jelly,” a murdered ice vendor’s blood has merged on the road with ice melted from his pushcart, and a child who drives past it in a horse-carriage mistakes it for jelly. In “warning,” a rich man is dragged out of his house by a mob and kicked to the ground. The mob is about to kill him when the man gets up, removes the dust from his clothes “with great dignity” and “wagging a finger at the rioters” says, “You can kill me, but I am warning you, don’t you dare touch my money!” In “The Garland,” another mob, this one in Lahore, frantically descends on the statue of a Hindu philanthropist (Hindus began to flee Lahore as soon as it was “given” to Pakistan). One of the rioters is about to place a garland of shoes around the statue’s neck when he is shot by police and taken to the hospital—a hospital built by the same Hindu philanthropist. And this is what happens in “modesty”:

The rioters brought the train to a stop. Those who belonged to the other religion were methodically picked out and slaughtered. After it was all over, those who remained were treated to a feast of milk, custard pies and fresh fruit.

Before the train moved off, the leader of the assassins made a small farewell speech: “Dear brothers and sisters, since we were not sure about the time of your train’s arrival, regretfully we were not able to offer you anything better than this most modest hospitality. We would have liked to have done more.”

That is all there is: murder—methodical and quick—followed by a feast and an ingratiating speech. Note the withholding of tags: we don’t know the location of the massacre or the religion of the killers. All we have is a spurt of base instincts. (Including false modesty, present even in carnage.) Though “Modesty” is unrelated in plot or sequence to the other sketches in “Siyah Hashiye,” it fits in the whole—like a snapshot in a slide show. Taken one at a time, these sketches enact the speed, randomness, and anonymity of the partition violence; cumulatively they expose the partition—the coming-into-being of two flag-waving nation-states—as little more than an orgy of loot and blood. Manto’s eye for irony here is sharp as ever. And the fragmented feel of his “sketches” represents a formal breakthrough—Manto has blasted away the conventions of the short story and arrived at a jagged approximation of the partition trauma.

But the timing of “Siyah Hashiye” was off. The literary scene in Pakistan had already broken down into factions: on one side were the progressive writers, now taking cues from the Soviet Union and calling for a socialist reform of the new state, which was the project of “a few reactionaries, capitalists and feudal landlords”3; and on the other side were “liberal” writers who disdained “Art for the sake of the Party” and were more inclined to peddle the idea of a fragile and imperiled new state in need of “nation-building.” In such a fight, of what use were Manto’s writings, which offered Pakistan neither a Communist cure nor a patriotic facelift?

He was punished. The progressives called “Siyah Hashiye” reactionary; Manto responded by saying that he didn’t “care at all about ’progressivism.’” His next story about the partition, titled “Thanda Gosht” (“Cold Flesh”), was even more unmindful of the literary temperature: Ishwar Singh, a Sikh recently returned from a round of partition killing, can’t bring himself to make love to his mistress. The mistress suspects him of infidelity—why else can’t he get it up? In a fit of jealousy she stabs Ishwar Singh with his own dagger. Bleeding to death, Ishwar Singh admits that he tried to rape an unconscious Muslim girl, only to find her already dead (the “cold flesh” of the title). While it makes some literary stabs (death as contagion, etc.), “Thanda Gosht” is, ultimately, a racy little thing, relying on that last shocking detail for effect—and may well live up to a charge of sensationalism.

But in Pakistan the charge brought against “Thanda Gosht” was much more serious. A local bureaucrat named Chaudhry Muhammad Hussain, “one of Manto’s old tormentors,” took the story to mean that “we Muslims are so utterly without a sense of honor that Sikhs can rape even our dead daughters.”

Manto was back in the dock.

A protracted court trial in Lahore had him worrying about the punishment, which came to three years in jail and a fine of three hundred rupees. Manto appealed the verdict. Eventually, after a lot of worrying (and drinking), he was let off the hook by a judge who wore a conspicuously Islamic beard. (“If I sentence Saadat Hassan Manto,” the judge had said with a Manto-esque smile, “he will go around telling everyone that he was sentenced by a man with a beard.”)

Manto now found himself in the singularly unfortunate position of being branded a “progressive” by the state and a “reactionary” by the progressives. In an essay written at this time and addressed to his readers, Manto admits to feeling “a strange melancholy.” He had “rebelled against the great upheaval that the partition of the country caused” and recovered from that “sea of blood” some “pearls of regret.” (He means his partition stories.) But these pearls had found no place in Pakistan. Manto the writer had suffered, but so had Manto the man: “My present life is full of hardship. After working day and night, I barely make enough to fulfill my daily needs. The fear that keeps gnawing at me is that were I to die suddenly who will look after my wife and three little daughters?”

Q.1. What does the sentence, ’Manto, suddenly a Muslim among Hindus, panicked.” mean?’

(a)Manto had developed strong vitriolic feelings towards Hindus

(b)Manto did not identify with his Hindu counterparts anymore

(c)Manto had not changed, but the circumstances around him had changed throwing his religion into sharp relief

(d)Manto suddenly saw the clear distinction that existed between Hinduism and Islam

Q.2. Which of the following can be said to be true of the bureaucrat’s misinterpration of the premise of ’Thanda Ghost’?

1.Manto was a progressive and therefore everything he wrote was reactionary

2.The bureaucrat believed Manto was trying to dishonor Islam and bring shame to the community

3.The bureaucrat deliberately missed the real point of the story which aimed to highlight the human elements of the partition and unfairly charged Manto of attacking Islam

4.The bureaucrat followed Manto’s work and looked for opportunities to disparage him

(a)1 and 4  (b) 1, 2, 3 and 4

(c)2, 3 and 4  (d) 3 and 4

Q.3. With reference to the passage, what did Manto’s writing not offer the reader?

(a)Irony and romanticism

(b)Sensationalism

(c)Satire

(d)Political solutions

Q.4. According to the passage, it can be inferred that:

(a)Manto was apathetic to the fate of Pakistan

(b)The writings of Manto reflected his anguish at the partition as well as his helplessness

(c)Manto gladly left India for Pakistan

(d)Manto’s work deserved the criticism leveled at it from all ends

Passage 3

Scientists have found the oldest DNA evidence of humans’ biological history. However, instead of clarifying human evolution, the finding is adding new mysteries, NYT reports. Analysis of a DNA test of a 400,000-year-old human thighbone found in a cave in northern Spain known as the “Pit of Bones,” shows an unexpected evolutionary twist, German scientists report.

The fossil found in Spain, had previously seemed to many experts to belong to a forerunner of Neanderthals and as the early hominins looked a little like Neanderthals, researchers expected their DNA to share a common ancestor. When Dr. Meyer and his colleagues drilled into the femur, they found ancient human DNA inside, just as they had hoped. However, after analyzing mitochondrial DNA extracted from the bone, it was found that early Europeans share a common ancestor with another mysterious species—eastern Eurasian sister group to the Neanderthals, the Denisovans. “Our expectation was that it would be a very early Neanderthal,” Dr. Meyer said.

First discovered in Siberia in 2010, the Denisovans were a genetically different group of people who are known only by a pinkie bone and a tooth. “Everybody had a hard time believing it at first,” Dr. Meyer said. “So we generated more and more data to nail it down.” Until now, Denisovans were known only from DNA retrieved from 80,000-year-old remains in Siberia, 4,000 miles east of where the new DNA was found.

“Right now, we’ve basically generated a big question mark,” said Matthias Meyer, a geneticist at the Max Planck Institute for Evolutionary Anthropology in Leipzig, Germany, and a co-author of the new study.

The mismatch between the anatomical and genetic evidence surprised the scientists, who are now rethinking human evolution over the past few hundred thousand years.

Finding such ancient human DNA was a major advance, said David Reich, a geneticist at Harvard Medical School. “That’s an amazing, game-changing thing,” he said.

“This would not have been possible even a year ago,” said Juan Luis Arsuaga, a paleoanthropologist at Universidad Complutense de Madrid and a co-author of the paper.

Since the 1970s, Spanish scientists have excavated 28 nearly complete skeletons of humans from the cave. Based on the anatomy of the fossils, Dr. Arsuaga has argued that they belonged to ancestors of Neanderthals, which lived in western Asia and Europe from about 200,000 to 30,000 years ago but the new finding give a totally different picture of human evolution especially as Denisovans were believed to be limited to East Asia, and they were not thought to look so Neanderthal-like.

As The New York Times writes, based on previously discovered ancient DNA and fossil evidence, scientists generally agreed that humans’ direct ancestors shared a common ancestor with Neanderthals and Denisovans that lived about half a million years ago in Africa. In 2006, a team of French and Belgian researchers obtained a fragment of Neanderthal DNA dating back 100,000 years, which until now was the oldest human DNA ever found, Science Direct says.

Around the same time as that discovery, Russian scientists sent the Max Planck team 80,000-year-old Denisovans’ fossils they had found in a cave in Siberia. When the German scientists sequenced the entire genome from the finger bone of a girl, it turned out to be neither human nor Neanderthal.

This made the scientists assume that their shared ancestors split off from humans’ lineage and left Africa, then split further into the Denisovans and Neanderthals about 300,000 years ago. The evidence suggested that Neanderthals headed west, toward Europe, and that the Denisovans moved east. Humans’ ancestors, meanwhile, stayed in Africa, giving rise to Homo sapiens about 200,000 years ago. Humans then expanded from Africa into Asia and Europe about 60,000 years ago. They then interbred not only with Neanderthals, but with Denisovans, too. Later, both the Denisovans and Neanderthals became extinct.

Dr. Arsuaga doubts that Denisovans were spread out across so much of the Old World, from Spain to Siberia, so one alternative explanation could be that the humans of Sima de los Huesos were not true Neanderthals, but belonged to the ancestors of both Denisovans and Neanderthals.

It is also possible that the newly discovered DNA was passed to both Neanderthals and Denisovans, but eventually disappeared from Neanderthals, replaced by other variants.

Beth Shapiro, an expert on ancient DNA at the University of California, Santa Cruz, assumes that the humans of Sima de los Huesos belong to another branch of humans, which might have been a species called Homo erectus and have originated about 1.8 million years ago having become extinct within the last few hundred thousand years.

“The more we learn from the DNA extracted from these fossils, the more complicated the story becomes,” Dr. Shapiro said. This new genetic link has baffled experts from around the world. Thus, Professor Chris Stringer from London’s Natural History Museum says it shattered the previous record of 100,000 year old DNA. “With the DNA from the rest of the genome, we really can build up a very full story of these earlier stages of human evolution which so far we haven’t been able to do,” he said.

Q.1. Which of the following can be concluded about the Neanderthal DNA?

(a)They were discovered 80, 000 years ago in Siberia

(b)They could be identified by only a pinkie bone and a tooth and were not very human-like

(c)They have been found to be unrelated to the newly discovered DNA found in the Pit of Bones

(d)It was more concentrated in the West than in the East

Q.2 Identify the true statement:

(a)The Denisovans are the ancestors of Eastern civilizations and the Neanderthals of Western civilizations

(b)The new DNA could have been passed onto both the Denisovans and the Neanderthals

(c)Finding the new DNA was a one-off chance discovery that won’t be possible again

(d)Dr. Arsuaga is of the opinion that the humans of Sima de los Huesos belonged to another branch of humans called the Homo Erectus

Q.3. Which of the following is the most startling revelation as a result of the new finding?

(a)The revelation that human biological history may go back further than 100, 000 years to 400, 000 years.

(b)That the DNA was related to the Denisovans, thus far believed to have been limited to the ’Old World’

(c)That the Denisovans were not as genetically different as was previously believed

(d)That technology that can gauge the age of DNA this old exists

TEST 4

Passage 1

The ’Right to Protect’ (R2P or RtoP) doctrine developed as a direct result of conflict in the former Yugoslavia and Rwanda in the 1990s, beginning with a re-evaluation of both the obligations and role of the international community in times of conflict. In the International Commission on Intervention and State Sovereignty’s (ICISS) groundbreaking report of 2001 entitled ’Responsibility to Protect’, it was argued that states have a duty to protect their population from four types of crimes: genocide, war crimes, ethnic cleansing and crimes against humanity.

When states were either unable or unwilling to meet this responsibility, the responsibility shifted then to the international community. In this sense, R2P consisted of three consistent sets of responsibilities: to prevent, react and rebuild. In 2005, world leaders at the World Summit collectively endorsed R2P and the Summit’s outcome document was later adopted as a resolution of the General Assembly. R2P was further reaffirmed by UN Security Council (UNSC) Resolutions 1674 (2006) and 1894 (2009) respectively. In just a short time, R2P became a concept with significant weight in the international community.

Irrespective of its positive start, R2P had problems moving from a theoretical concept to one firmly recognized in policy. The situation in Dafur was often seen to be a ’test case’ for R2P; one which was widely judged to have failed. In 2007, after the Kenyan elections, some 1,500 people were killed as a result of ethnic and tribal violence, yet through a diplomatic effort understood directly in R2P terms, both sides settled on a power-sharing agreement — demonstrating R2P’s capacity to facilitate the prevention of crimes through peaceful measures.

Moreover, R2P has been applied to circumstances it was not originally intended for, for example the slow response of Myanmar to requests for humanitarian access after Cyclone Nargis and the Russian invasion of Georgia both in 2008. In both cases the international community rejected claims of the application of R2P, further clarifying the limitations of R2P’s remit. In 2009, in the midst of a situation between Sri Lankan government forces and Tamil Tigers (LTTE) in the Wanni region, the UN country team withdrew its entire staff and remained silent on potential violations of both humanitarian law and international human rights law by the state. In an Internal Review Panel of 2012, it was ruled that this outcome was a ’grave failure’ by the UN, in contradiction of its core responsibilities and principles. All of the different interpretations of when R2P comes into effect illustrates that state sovereignty remains a strong international norm that can undermine action.

The different actions of the UNSC in both Libya and Syria shed some light on the current effectiveness of R2P. In 2011, in response to the violence in Libya, the UNSC passed Resolution 1970, under Chapter VII of the UN Charter. With the Qaddafi regime’s failure to comply to UNSC demands, the UNSC authorized Resolution 1973, which consented the use of force in order to protect civilians from danger, enforced a no-fly zone over Libya and implemented an arms embargo. This was the first instance where the UNSC had permitted the use of force against a Member State in the name of protecting civilians.

In Syria, the UNSC initially failed to gain consensus on the conflict. There were several attempts to pass UNSC Resolutions, yet all were in vain because of vetoes by both Russia and China. For other actors, namely Russia, the experience of Libya has compelled it to openly criticize Western pressure on Syria as it might have opened the door to an undesirable regime change. The UNSC then authorized the deployment of the UN Supervision Mission in Syria (UNSMIS) during 2012, where only after a couple of months it failure to influence the conflict resulted in an embarrassing withdrawal.

R2P as a concept offers much potential but remains far from perfect. With the ’success’ in Libya and bigger failure in Syria, it could be said that the end of R2P is quite close, simply because it is so selectively applied and in the current international framework does not allow much flexibility (such as the sheer difficulty of authorizing humanitarian intervention). If it is the end of R2P, one wonders how then intervention will be framed in the future.

Q.1. Which of the following is not a weakness of the R2P?

(a)Due to the wide room for interpretation, it is susceptible to misuse

(b)It falls prey to selective and impartial implementation

(c)Human rights remain a flexible concept and more so internationally

(d)There are not enough policy mechanisms or organizations to implement the R2P

Q.2. With reference to the passage, select the option that involves the application of a component of the R2P:

(a)Nearly all states in the world have maintain active defense forces and a slew of international civil servants responsible for international diplomacy

(b)Some states such as The Peoples’ Republic of China engage in repression and contraction of basic liberties like freedom of expression, that all human beings are entitled to

(c)In many nations of the world women are not yet entitled to the same privileges as men and suffer atrocities at their hands

(d)In some countries, the government is monarchical, and citizens assume the role of passive subjects instead of active participants.

Q.3. With regards to the implementation of R2P, which statement is not justified?

(a)Diplomatic efforts in Kenya in order to bring both the sides to power sharing agreement show the influence of R2P. Although it missed on the timeliness part. Prior implementation could have avoided the genocide that took place just after the elections.

(b)R2P as a concept failed to be implemented in Syria and not in Libya because of the interventions of Russia and China at the right time ,seeing the result from the Libyan case. It does reflect the political grasp of super powers on implementations of UN policies.

(c)Withdrawal of UN forces from Sri Lanka shows the incompetency of the UN organization for not being able to implement its responsibilities properly and not the weakness of the policy of R2P.

(d)Though theoretically R2P is a strong concept, implementation is difficult because of the difference in interpretation among the nations across the world.

Q.4. The author’s tone towards the R2P is.

(a)Critical  (b) Skeptical

(c)Neutral  (d) Specious

Passage 2

JFK has often been criticised for his lack of tangible political success in the US yet his domestic record is far from poor. Civil rights formed one of the cornerstones of Kennedy’s domestic programme as both the President and the Attorney General, Robert Kennedy, strove to improve the living and working conditions of African-Americans in the US. The CEEO (Commission on Equal Employment Opportunity) was created to ensure that all federal government employees were subject to the same employment rights and 40 African-Americans were appointed to senior federal posts under his administration.

Whilst these figures may seem paltry by today’s standards, these actions inspired momentum in the movement for racial equality. On June 11 1963 in his address to the nation, Kennedy became the first president since the much-revered Abraham Lincoln to publically promote the moral necessity for racial equality. In August 1963, despite initial fears about the Civil Rights March on Washington, Kennedy gave it his full backing; Martin Luther King Jr. did the rest.

Kennedy’s New Frontier essentially centred on breaking down long-held beliefs that had made discrimination and exploitation the norm. During his time in office, the minimum wage was increased to $1.25 and coverage was extended to 4.3 million people. At the same time, the Commission on the Status of Women was created in 1961 to ensure greater standards of equality for women — particularly in the work place. Kennedy’s approach to promoting sexual equality as part of his domestic programme was something that even FDR’s New Deal had overlooked.

The Peace Corps, created by Kennedy to expand US influence abroad whilst also providing the American youth with the opportunity to experience different cultures and ways of life, operated under a $375m budget in 2012. This serves to highlight the longevity of JFK’s ideals which would not have stood the test of time had his policies been lacking in substance.

Foreign policy was Kennedy’s biggest strength and asset, despite early failings with the Bay of Pigs debacle and the Vienna Conference. Whilst it was Eisenhower who had planned the invasion of Cuba, it was Kennedy who authorised the invasion and was rightly criticised for its failure. In Vienna in 1961, the young Kennedy was no match for his more experienced Soviet counterpart. Khrushchev left Austria thinking Kennedy to be naïve and weak.

These thoughts were quickly dispelled the following year during the Cuban Missile Crisis when Kennedy displayed strong, decisive leadership in the face of an international crisis. Despite intense pressure from leading military figures within ExComm to go to war over the placement of Soviet missiles in Cuba, Kennedy stuck to his guns and gave the Soviet Union every opportunity to withdraw missiles from Cuba without provoking military action. The naval blockade and the covert methods of negotiation he employed ultimately proved successful as he reached his goal, which in his words was “not the victory of might, but the vindication of right”.

Under the spectre of nuclear war, JFK’s leadership was measured. His behaviour not only eased the fears of the American people but also ensured the immediate withdrawal of Soviet missiles, thus handing his administration a significant boost both at home and abroad. Not only was war avoided, but JFK also enabled the Soviets to pull out without losing face at home by agreeing to withdraw outdated US missiles from Turkey. His actions displayed a deeply held moral belief in the pursuit of peace.

JFK’s style has never been called into question. His approach to politics was fresh and he exuded a personable charm that was distinctly lacking in many of his contemporaries.

Q.1. Which quality of Kennedy from the following, do you think is not true as per the passage?

(a)His hold on the domestic front by the implementation of the movement against Racial Discrimination.

(b)His methods were different from conventional politicians. He was always up against the rival nations & created shadows of war. This inculcated a feeling of fear not only among the Americans, but brought him criticism throughout the world.

(c)His ability to learn from his mistakes and failures allowed him to achieve success in the later stages of his career.

(d)He not only proved his mettle as a successful strategist and a leader, but also a combination of fresh ideas made him stand apart from his contemporaries.

Q.2. The author would agree with which of the following judgments about the style and character of JFK’s politics and personality?

(a)He was a misfit in the world of politics, with little relevant experience and lacked foresight.

(b)He was an asset to American polity and was crucial to the success of many civil rights’ movements and initiatives in the country.

(c)Kennedy was faultless and always authorized the best possible courses of action

(d)Kennedy was considered a political hero and there are no questions as to his contributions to the American political system.

Passage 3

China’s leaders have now agreed to relax the country’s decades-old “one-child” policy. Couples now will be permitted to have two children if one parent is an only child (previously, both parents had to be only children), making the new rule applicable to most of the post-1980’s generation that grew up in urban areas. But, while the potential social consequences are obvious, the likely economic impact is less apparent.

When the one-child policy was implemented in 1979 — in an effort to alleviate social, economic, and environmental pressures following the population boom in the 1950s and 1960s — the fertility rate plummeted, from three children per household in 1970 to 1.2 in 1982. The household saving rate subsequently soared, from 10.4% in 1983 to a staggering 30.5% in 2011. Could the one-child policy have fueled this rise? If so, will the modified policy precipitate a reversal in this trend and, in turn, a consumption boom in the coming decade?

Rising fertility rates can lower the household saving rate in two main ways. First, children demand increased household expenditure — especially on education, which, for only children aged 15-22, accounts for 15-25% of total Chinese household expenditures. Second, with more children to support them in their old age, parents feel less pressure to save for retirement.

An examination of the spending and saving patterns of households with twins born under the one-child policy suggests that these changes could lead to a drop of 8-9 percentage points in China’s household savings rate — to around 22% — in the coming decades. Because the birth of twins is arguably an exogenous occurrence (unrelated to factors like income and education that affect parents’ decision to have multiple children), comparing households with an only child to those with twins can reveal some of the effects that an additional child has on saving.

From 2002 to 2009, the average saving rate for an only-child household was 21.3%, compared to 12.8% for a household with twins — a difference that holds across all income groups. Regression analysis of all urban households in the 1992-2009 period shows that an additional child reduced the saving rate by about 6-7 percentage points. One additional child increased education expenditure (as a percentage of household income) by an average of seven percentage points, food expenditure by 2.5 percentage points, and other spending by roughly 2.7 percentage points.

Intergenerational support — whereby children provide for parents in old age — is not only a social norm; in China, it is stipulated by law. More than half of elderly people’s income derives from family support, which includes financial transfers and in-kind benefits like cohabitation. And the total amount of transfers that parents receive rises with the number of children.

The added security offered by additional offspring means that loosening the one-child policy will probably lead parents to save less on their own, and possibly even to make the kind of risky investments that would be unwise for only-child families. In fact, the one-child policy could be an important reason why, in recent decades, Chinese households have gravitated toward riskless assets. If parents view having a second child as doubling their “safe assets,” they may decide to allocate more of their portfolio toward riskier assets.

It follows that parents of multiple children may also feel secure enough to consume more. On average, the share of household income spent by parents of multiple children on non-child-related consumption in later stages of life is about eight percentage points higher than it is for parents of only children.

The coming demographic shift will also affect national saving. The share of young dependents/borrowers will rise relative to the middle-aged working population. Reducing the proportion of high savers in the economy will tend to lower the aggregate saving rate. The share of elderly people in the population, however, will not change until a generation later.

Though the change to the one-child policy may cause a moderate consumption boom, it could also reverse a positive trend: the acceleration of human-capital accumulation. The typical only child receives significantly more educational investment than the average twin and is 40% more likely to pursue higher education (as opposed to a vocational school). This can likely be explained, in part, by a quantity-quality tradeoff.

Q. 1. Out of the following points which are in sync with the author’s thinking about the new policy:

1.The removal of “one-child policy” will increase the support to the elders in the family, thereby safeguarding their savings and support in the older age.

2.Chinese households will have increased consumption and savings rate will decline, which in turn will make the parents to invest more in assets that are riskier to increase the returns.

3.Rising fertility rates will cause more of social concerns and economic impact will be lesser.

4.The removal of such a policy will soon decrease the average age of China, thus it will add to the national savings.

(a)Only 1 & 3 (b) 1, 2 & 4

(c)2 & 3  (d) 2, 3 & 4

Q.2. Which of the following will NOT be a consequence of the relaxation of the one-child policy:

(a)The Chinese will begin showing less risk-averse behavior and adopt more unconventional investment practices

(b)The quality of education per child is likely to suffer

(c)The economy will contract as household turn towards greater savings

(d)The fertility rate of household will show a marked jump and the share of elderly people in the population will ascend slowly

Q.3. Identify the correct inference of the author’s intent in the passage:

(a)To insinuate that the Chinese consider their offspring as mere assets

(b)To analyse the socio-economic impact of the revocation of the old one-child policy in China

(c)To describe the successes and failures of the one-child policy

(d)To depict the recent changes and patterns in the Chinese society and economy

TEST 5

Passage 1

Implicit in the freedoms we cherish in our democracy is our right to offend. That is the cornerstone of all free thought and its expression. In a country as beautiful and complex as ours, it is our inalienable right to offend that makes us the nation we are. Of course I also recognize the fact that this right attaches to itself many risks, including the risk of being targeted. But as long as these risks are within reasonable, well-defined limits, most people will take them in their stride. I am ready to defend my right to offend in any debate or a court of law. But it’s not fine when mobs come to lynch you. It’s not right, when they vandalize your home or burn your books or art or stop you from showing your film or, what’s becoming more frequent, hire thugs to kill you. Authors, journalists, painters, and now even activists and rationalists are being openly attacked and murdered.

It’s a constant challenge to walk the tightrope; to know exactly where to draw the line when you write, paint, speak. The funny thing is truth has no limits, no frontiers. When you want to say something you strongly believe in, there is no point where you can stop. The truth is always whole. When you draw a line, as discretion suggests, you encourage half-truths and falsehoods being foisted on others, you subvert your conscience. In some cases it’s not even possible to draw a line. A campaigner against corruption can never stop midway through his campaign even though he knows exactly at which point the truth invites danger, extreme danger. Yet India is a brave nation and there are many common people, ordinary citizens with hardly any resources and no one to protect them who are ready to go out on a limb and say it as it is. They are the ones who keep our democracy burning bright.

Every few days you read about a journalist killed. About RTI activists murdered for exposing what is in the public interest. You read about people campaigning for a cause (like Narendra Dabholkar, who fought against superstitions, human sacrifices, babas and tantriks) being gunned down in cold blood. Even before the police can start investigations, the crime is invariably politicised. Issues of religion, caste, community, political affiliation are dragged in only to complicate (read obfuscate) the crime and, before you know it, the story dies because some other, even more ugly crime is committed somewhere else and draws away the headlines and your attention. And when that happens, criminals get away. We are today an attention deficit nation because there’s so much happening everywhere, all pretty awful stuff, that it’s impossible for anyone to stay focused.

Even fame and success can’t protect you. Dr Dabholkar was a renowned rationalist, a man of immaculate credentials. Yet he was gunned down by fanatics who thought he was endangering their trade in cheating poor and gullible people. Husain was our greatest living painter. He was forced into exile in his 80s because zealots refused to let him live and work in peace here. They vandalised his art; hunted down his shows, ransacked them. Yet Husain, as I knew him, was as ardent a Hindu as anyone else. His paintings on the Mahabharata are the stuff legends are made of. A pusillanimous government lacked the will to intervene.

When we deny ourselves the right to offend, we deny ourselves the possibility of change. That’s how societies become brutal, moribund, disgustingly boring. Is this what you want? If the answer is No and you want to stay a free citizen, insist on your right to offend. If enough people do that, change is not just inevitable. It’s assured. And change is what defines a living culture.

Q.1. The author is in agreement with which of the following?

1.The right to offend is worth preserving and an essential component of a democratic set up. It allows one to put his/her point upfront when it’s the need of hour.

2.The absence of the right to offend would lead to a destruction of cultural freedom and society would degenerate into a lackluster, homogenous constant

3.The right to offend should be absolutely limitless to safeguard us against the atrocities that are brought down upon us by the zealots, who will always prevail in the society.

4.The right to offend is responsible for many injustices such as the murder of Narendra Dabholkar and the vandalism against Hussain’s paintings

(a)1   (b) 1, 2 and 3

(c)1 and 2  (d) 3 and 4

Q.2. If you were to paraphrase the word ’pusillanimous’, which of the following would you choose to use:

(a)Complacent  (b) Ineffective

(c)Timid   (d) Sensible

Q.3. Why is the right to offend a right worth preserving?

(a)It empowers a select few in enforcing their opinions on others regardless of their acceptance or agreement

(b)The right to express oneself is essential for individual development and allows for cultural advancement

(c)To offend is to deny another of his/her right

(d)The right to offend is a partial representation of the truth, and the truth must be expressed in public

Passage 2

Simon Murden writes, “The Clash of Civilizations may not have told the whole story of what was happening in the post cold war world, but it told part of it”. The claims he made shaped the contours of the international realm in a manner which was largely ignored in an age dominated by ideological leanings. The post Cold War dynamics have been captured by Huntington by elevating the importance of cultural variables. The fervor of the text is decidedly realist, however Huntington in an interview clarified that he was aware of the views which branded his work as the “latest version of billiard ball realism” but he also asserted that the difference lay in the replacement of nation state as a primary actor to civilizations as the movers’ history. In essence the Cold War paradigm now came to be replaced by a new paradigm with culture occupying the central role. The problem of binaries was evident as the shifting power equations gave way to identities being defined not in ideological terms rather they were now defined in civilizational terms which he believed was, “culture writ large”.

Huntington’s contribution lay in generating a new paradigm. Gideon Rose argued that his work was directed at the major practical and theoretical questions of the day. Herein lay his brilliance. However it has also been argued that his methodology was anecdotal in nature and it catered to few specific situations to arrive at a grand theory. His approach many believe can lead to situation based convenient explanations and arguments. Huntington was aware of this and he acknowledged the need to situating certain issues in time and space. He was never much concerned about the method, as much as he was about arriving at empirical generalizations and this encouraged him to draw comparisons. He identified the emergence of western universalism and the backlash against it which assumed various forms. It ranged from emergence of religious fundamentalism to challenges in the form of assertion of Asian Values which gained salience owing to Japan’s emergence as a major economic power.

The notion of power is central to his work. This is evident in his treatment of civilizational differences as a major source of conflict in the post Cold War era. In this sense he recognized culture as acquiring a political dimension. The issue of identity defined by religion which was obscured by the Cold War obsession with ideologies gained renewed importance. The understanding of the ’other’ was weighed against their ’own constructed identities’. Huntington warns against insistence on equating westernization and modernization. This he argues strengthens indigenizing forces and often initiates a violent opposition to the west.

Samuel Huntington’s book provides a powerful and compelling analysis of a de-ideologized world in which he writes, “The velvet curtain of culture has replaced the iron curtain of ideology as the most significant dividing line in Europe”. On the one hand was Francis Fukuyama who was proclaiming that the collapse of the Soviet Union signaled the end of history as liberal values triumphed, and on the other hand was Huntington who evidently was uncomfortable with such a Universalist claim. Huntington writes that, “western belief in universality of western culture suffers from three problems: it is false; it is immoral; and it is dangerous”.

He recognized the pitfalls of viewing the neo liberal concepts of interdependence largely in terms of the west and acknowledged the need for, “universalism at home and multiculturalism abroad”. The ’Clash of Civilizations thesis’ despite its misinterpretations, became conventional wisdom when much of the conflicts and wars in the world could be articulated in terms of being a conflict between different cultures. The 9/11 incident signaled the potency of cultural differences. The borders of civilizations for Huntington became sacrosanct. To the extent that many have argued, he ignored the cross cultural interactions which shaped these civilizations and the internal schisms characteristic of many of them.

Q.1. Which of the following is NOT a conclusion applicable to Hungtington’s work or style?

(a)Huntington’s work had a distinctively realist flavor with a stand opposite to the neo liberal world’s generalized universality

(b)Huntington employed an anecdotal methodology to arrive at grand theories

(c)Huntington was essentially a universalist who used empirical generalizations to justify his deductions

(d)Huntington was a critic of the neo-liberal school of thought and believed that the curtain of ideology was being replaced by the curtain of culture

Q.2. If you were to ask Samuel Huntington a follow-up question based on your reading of this passage, which of the following would be the LEAST FAVORABLE option?

(a)What are the disadvantages of viewing the world through the Universalist prism?

(b)Can culture be prevented from taking on political dimensions such as in the West, or this inevitable? Again, is it necessary?

(c)In what ways can cross-cultural interactions shape entire civilizations?

(d)Is ideology completely irrelevant in the age of the curtain of culture?

Q.3. In the context of the passage, the best replacement (that would not alter the meaning of the sentence) for the phrase ’internal schisms’ would be:

(a)Internal estrangements

(b)Internal detachments

(c)Internal rifts

(d)Internal dissension

Passage 3

A country under foreign domination seeks escape from the present in dreams of a vanished age, and finds consolation in visions of past greatness. That is a foolish and dangerous pastime in which many of us indulge. An equally questionable practice for us in India is to imagine that we are still spiritually great though we have come down in the world in other respects. Spiritual or any other greatness cannot be founded on lack of freedom and opportunity, or on starvation and misery. Many western writers have encouraged that notion that Indians are other-worldly. I suppose the poor and unfortunate in every country become to some extent other-worldly, unless they become revolutionaries, for this world is evidently not meant for them. So also subject peoples.

As a man grows to maturity he is not entirely engrossed in, or satisfied with, the external objective world. He seeks also some inner meaning some psychological and physical satisfactions. So also with people and civilizations as they mature and grow adult. Every civilization and every people exhibit these parallel streams of an external life and an internal life. Where they meet or keep close to each other, there is an equilibrium and stability. When they diverge conflict arises and the crises that torture the mind and spirit.

Q.1. The passage mentions that “this world is evidently not meant for them”. It refers to people who

1.Seek freedom from foreign domination.

2.Live in starvation and misery.

3.Become revolutionaries

Which of the statements given above is/are correct?

(a)1 and 2

(b)2 only

(c)2 and 3

(d)3 only

Q. 2. Consider the following assumptions :

1.A country under foreign domination cannot indulge in spiritual pursuit.

2.Poverty is an impediment in the spiritual pursuit.

3.Subject peoples may become other-worldly.

With reference to the passage, which of the above assumptions is/are valid?

(a)1 and 2

(b)2 only

(c)2 and 3

(d)3 only

Q.3. The passage thematically centers on

(a)The state of mind of oppressed people

(b)Starvation and misery

(c)The growth of civilization

(d)Body, mind and spirit of people in general.

Q.4. According to the passage, the torture of the mind and spirit is caused

(a)By the impact of foreign domination.

(b)By the desire to escape from foreign domination and find consolation in visions of past greatness.

(c)Due to lack of equilibrium between an external life and an internal life.

(d)Due to one’s inability to be either revolutionary or other-worldly.

TEST 6

Passage 1

The concept of ’creative society’ refers to a phase of development of a society in which a large number of potential contradictions become articulate and active. This is most evident when oppressed social groups get politically mobilised and demand their rights. The upsurge of the peasants and tribals, the movements for regional autonomy and self-determination, the environmental movements, and the women’s movements in the developing countries are signs of emergence of creative society in the contemporary times. The forms of social movements and their intensity may vary from country to country and place to place within a country. But the very presence of movements for social transformation in various spheres of a society indicates the emergence of a creative society in a country.

Q.1. What does the author imply by “creative society”?

1.A society where diverse art forms and literary writings seek incentive.

2.A society where social inequalities are accepted as the norm.

3.A society where a large number of contradictions are recognised.

4.A society where the exploited and the oppressed groups grow conscious of their human rights and upliftment.

Select the correct answer using the codes given below:

(a)1, 2 and 3

(b)4 only

(c)3 and 4

(d)2 and 4

Q. 2. What according to the passage are the manifestations of social movements?

1.Aggressiveness and being incendiary.

2.Instigation by external forces.

3.Quest for social equality and individual freedom.

4.Urge for granting privileges and self-respect to disparaged sections of the society.

Select the correct answer using the codes given below:

(a)1 and 3 only

(b)2 and 4 only

(c)3 and 4 only

(d)1, 2, 3 and 4

Q.3. With reference to the passage, consider the following statements:

1.To be a creative society, it is essential to have a variety of social movements.

2.To be a creative society, it is imperative to have potential contradictions and conflicts.

Which of the statements given above is/are correct?

(a)1 only

(b)2 Only

(c)Both 1 and 2

(d)Neither 1 nor 2

Passage 2

Ecosystems provide people with a variety of goods and services; food, clean water, clean air, flood control, soil stabilization, pollination, climate regulation, spiritual fulfillment and aesthetic enjoyment, to name just a few. Most of these benefits either are irreplaceable or the technology necessary to replace them is prohibitively expensive. For example, potable fresh water can be provided by desalinating sea-water, but only at great cost.

The rapidly expanding human population has greatly modified the Earth’s ecosystems to meet their increased requirements of some of the goods and services, particularly food, fresh water, timbre, fiber and fuel. These modifications save contributed substantially to human well being and economic development. The benefits have not been equally distributed. Some people have actually been harmed by these changes. Moreover, short-term increases in some ecosystem goods and services have come at the cost of the long-term degradation of others. For example, efforts to increase the production of food and fiber have decreased the ability of some ecosystems to provide clean water, regulate flooding and support biodiversity.

Q.1. With reference to the passage, consider the following statements.

Expanding human population has an adverse effect on:

1.Spiritual fulfillment

2.Aesthetic enjoyment

3.Potable fresh water

4.Production of food and fiber

5.Biodiversity

Which of the statements given above are correct?

(a)1, 2 and 3 only

(b)2, 4 and 5 only

(c)3 and 5 only

(d)1, 2, 3, 4 and 5

Q.2. The passage mentions that “some people have actually been harmed by these changes”. What does it imply?

1.The rapid expansion of population has adversely affected some people.

2.Sufficient efforts have not been made to increase the production of food and fiber.

3.In the short term some people may be harmed, but in the long term everyone will benefit from modifications in the Earth’s ecosystems.

Which of the statements given above is/are correct ?

(a)1 only

(b)2

(c)1 and 3

(d)None of the statements given above.

Q.3. With reference to the passage, consider the following statements :

1.It is imperative to modify the Earth’s ecosystems for the well being of mankind.

2.Technology can never replace all the goods and services provided by ecosystems.

Which of the statements given above is/are correct?

(a)1 only

(b)2 only

(c)Both 1 and 2

(d)Neither 1 nor 2

Passage 3

For achieving inclusive growth there is a critical need to rethink the role of the State. The early debate among economists about the size of the Government can be misleading. The need of the hour is to have an enabling Government. India is too large and complex a nation for the State to be able to deliver all that is needed. Asking the Government to produce all the essential goods, create all the necessary jobs, and keep a curb on the prices of all goods is to lead to a large cumbersome bureaucracy and widespread corruption.

The aim must be to stay with the objective of inclusive growth that was laid down by the founding fathers of the nation and also to take a more modern view of what the state can realistically deliver.

This is what leads to the idea of an enabling state, that is, a government that does not try to directly deliver to the citizens everything that they need. Instead, it (1) creates an enabling ethos for the market so that individual enterprise can flourish and citizens can, for the most part, provide for the needs of one another, and (2) steps in to help those who do not manage to do well for themselves, for there will always be individuals, no matter what the system, who need support and help. Hence we need a government that, when it comes to the market, sets effective, incentive-compatible rules and remains on the sidelines with minimal interference, and, at the same time, plays an important role in directly helping the poor by ensuring that they get basic education and health services and receive adequate nutrition and food

Q.1. According to the passage:

1.The objective of inclusive growth was laid down by the founding fathers of the nation.

2.Need of the hour is to have an enabling Government.

3.The Government should engage in maximum interference in market processes.

4.There is a need to change the size of the Government.

Which of the statements given above are correct?

(a)1 and 2 only

(b)2 and 3 only

(c)1 and 4 only

(d)1,2,3 and 4

Q.2. According to the passage, the strategy of inclusive growth can be effected by focusing on

(a)Meeting all the needs of every citizen in the country.

(b)Increasing the regulations over the manufacturing sector.

(c)Controlling the distribution of manufactured goods.

(d)Delivery of the basic services to the deprived sections of the society.

Q.3. What constitutes Government an enabling?

1.A large bureaucracy.

2.Implementation of welfare programmes through representatives.

3.Creating an ethos that helps individual enterprise.

4.Providing resources to those who are underprivileged.

5.Offering direct help to the poor regarding basic services.

Select the correct answer from the codes given below:

(a)1, 2 and 3 only

(b)4 and 5 only

(c)3, 4 and 5 only

(d)1, 2, 3, 4 and 5

Q.4. Why is the State unable to deliver “all that is needed” ?

1.It does not have sufficient bureaucracy.

2.It does not promote inclusive growth.

Select the correct’ answer from the codes given below:

(a)1 only

(b)2 only

(c)Both 1 and 2

(d)Neither 1 nor 2

Q.5. What is the essential message being conveyed by the author of the passage?

(a)The objectives of inclusive growth laid down by the founding fathers of the nation should be remembered.

(b)The Government needs to make available more schools and health services.

(c)The Government needs to establish markets and industries to meet the needs of the poor strata of the society.

(d)There is a need to rethink the role of the State in achieving inclusive growth.

TEST 7

Passage 1

Now India’s children have a right to receive at least eight years of education, the gnawing question is whether it will remain on paper or become a reality. One hardly needs a reminder that this right is different from the others enshrined in the Constitution, that the beneficiary — a six year old child cannot demand it, nor can she or he fight a legal battle when the right is denied or violated. In all cases, it is the adult society which must act on behalf of the child’s right to education when denied, no compensation offered later can be adequate or relevant. This is so because childhood does not last. If a legal battle fought on behalf of a child is eventually won, it may be little use to the boy or girl because the opportunity missed at school during childhood cannot serve the same purpose later in life. This may be painfully true for girls because our society permits them only a short childhood, if at all. The Right to Education (RTE) has become law at a point in India’s history when the ghastly practice of female infanticide has resurfaced in the form of foeticide. This is “symptomatic of a deeper turmoil” in society which is compounding the traditional obstacles to the education of girls. Tenacious prejudice against the intellectual potential of girls runs across our cultural diversity and the system of education has not been able to address it.

Q.1. With reference to the passage, consider the following statements :

1.When children are denied education, adult society does not act on behalf of them.

2.Right to Education as a law cannot be enforced in the country.

Which of the statements given above is/are correct?

(a)1 only

(b)2 only

(c)Both 1 and 2

(d)Neither 1 nor 2

Q.2. According to the passage, what could be the traditional obstacle to the education of girls ?

1.Inability of parents to fight a legal battle when the Right to Education is denied to their children.

2.The traditional way of thinking about girls’ role in society.

3.The prejudice against the intellectual ’potential of girls.

4.Improper system of education.

Select the correct answer from the codes given below:

(a)1 and 2 only

(b)2 , 3 and 4 only

(c)1, 3 and 4 only

(d)1, 2, 3 and 4

Q.3. On the basis of the passage, consider the following statements:

1.Right to Education is a legal right and not a fundamental right.

2.For realising the goal of universal education, the education system in the country must be made identical to that of developed countries.

Which of the statements given above is/are correct ?

(a)1 only

(b)2 only

(c)Both 1 and 2

(d)Neither 1 nor 2

Q.4. Which one of the following statements conveys the key message of the passage?

(a)India has declared that education is compulsory for its children.

(b)Adult society is not keen on implementing the Right to Education.

(c)The Right to Education, particularly of a girl child, needs to be safeguarded.

(d)The system of education should address the issue of Right to Education.

Q.5. Which one of the following statements conveys the inference of the passage?

(a)The society has a tenacious prejudice against the intellectual potential of girls.

(b)Adults’ cannot be relied upon to fight on behalf of children for —their Right to Education.

(c)The legal fight to get education for children is often protracted and prohibitive.

(d)There is no sufficient substitute for education received in childhood.

Passage 2

A species that exerts an influence out of proportion to its abundance in an ecosystem is called a keystone species. The keystone species may influence both the species richness of communities and the flow of energy and materials through ecosystems. The sea star Pisaster ochraceus which lives in rocky intertidal ecosystems on the Pacific coast of North America, is also an example of a keystone species. Its preferred prey is the mussel Mytilus californianus. In the absence of sea stars, these mussels crowd out other competitors in a broad belt of the intertidal zone. By consuming mussels, sea star creates bare spaces that are taken over by a variety of other species.

A study at the University of Washington demonstrated the influence of Pisaster on species richness by removing sea stars from selected parts of the intertidal zone repeatedly over a period of five years. Two major changes occurred in the area from which sea stars were removed. First, the lower edge of the mussel bed extended farther down into the intertidal zone, showing that sea stars are able to eliminate mussels completely where they are covered with water most of the time. Second, and more dramatically, 28 species of animals and algae disappeared from the sea star removal zone. Eventually only Mytilus, the dominant competitor, occupied the entire substratum. Through its effect on competitive relationships, predation by Pisaster largely determines, which species live in these rocky intertidal ecosystems.

Q.1. What is the crux of the passage?

(a)Sea star has a preferred prey.

(b)A preferred prey determines the survival of a keystone species.

(c)Keystone species ensures species diversity.

(d)Sea star is the only keystone species on the Pacific coast of North America.

Q.2. With reference to the passage, consider the following statements:

1.Mussels are generally’ the dominant species in intertidal ecosystems.

2.The survival of sea stars generally determined abundance of mussels.

Which of the statements given above is/are correct?

(a)1 only

(b)2 only

(c)Both 1 and 2

(d)Neither 1 nor 2

Q.3. Which of the following is/are implied by the passage?

1.Mussels are always hard competitors for sea stars.

2.Sea stars of the Pacific coast have reached the climax of their evolution.

3.Sea stars constitute an important component in the energy flow in intertidal ecosystem.

Which of the statements given above is/are correct ?

(a)1 and 2

(b)2 only

(c)1 and 3

(d)3 only

Q.4. Consider the following assumptions:

1.The food chains/food web in an ecosystem are influenced by keystone species.

2.The presence of keystone species is a specific characteristic of aquatic ecosystems.

3.If the keystone species is completely removed from an ecosystem, it will lead to the collapse of the ecosystem:

With reference to the passage, which of the above assumptions is/are valid?

(a)1 only

(b)2 and 3 only

(c)1 and 3 only

(d)1, 2 and 3

Q.5. Which is the most dominant competitor of the Pisaster?

(a)The mussels

(b)the sea oyster

(c)the sea star

(d)the sea urchin

Passage 3

A moral act must be our own act; must spring from our own will. If we act mechanically, there is no moral content in our act. Such action would be moral, if we think it proper to act like a machine and do so. For, in doing so, we use our discrimination. We should bear in mind the distinction between acting mechanically and acting intentionally. It may be moral of a king to pardon a culprit. But the messenger carrying out the order of pardon plays only a mechanical part in the king’s moral act. But if the messenger were to carry out the king’s order considering it to be his duty, his action would be a moral one. How can a man understand morality who does not use his own intelligence and power of thought, but lets himself be swept along like a log of wood by a current? Sometimes a man defies convention and acts on his own with a view to absolute good.

Q.1. Which of the following statements best describe/describes the thought of the writer?

l.A moral act calls for using our discretion.

2.Man should react to a situation immediately.

3.Man must do his duty.

4.Man should be able to defy convention in order to be moral.

Select the correct answer from the codes given below

(a)1 only

(b)1 and 3

(c)2 and 3

(d)1 and 4

Q.2. Which of the following statements is the nearest definition of moral action, according to the writer?

(a)It is a mechanical action based on official orders from superiors.

(b)It is an action based on our sense of discretion.

(c)It is a clever action based on the clarity of purpose.

(d)It is a religious action based on understanding.

Q.3. The passage contains a statement “lets himself be swept along like a log of wood by a current.” Among the following statements, which is/are’ nearest in meaning to this?

1.A person does not use his own reason.

2.He is susceptible to influence /pressure.

3.He cannot withstand difficulties/ challenges.

4.He is like a log of wood.

Select the correct answer using the codes given below:

(a)1 only  (b) 1and 2

(c)2 and 3  (d) 1 and 4

◊ Answer Key

Test 1

Passage 1

1. (b)

2. (c)

3. (d)

Passage 2

1. (b)

2. (b)


Passage 3

1. (b)

2. (a)

3. (b)

Test 2

Passage 1

1. (c)

2. (a)

3. (b)


Passage 2

1 (c)

2. (c)

3. (d)

4. (c)

Passage 3

1. (b)

2. (a)

3. (c)


Test 3

Passage 1

1. (c)

2. (b)

3. (a)

4. (d)

Passage 2

1. (c)

2. (d)

3. (d)

4. (d)

Passage 3

1. (d)

2. (b)

3. (a)


Test 4

Passage 1

1. (d)

2. (a)

3. (b)

4. (c)

Passage 2

1. (b)

2. (b)



Passage 3

1. (c)

2. (c)

3. (b)


Test 5

Passage 1

1. (c)

2. (c)

3. (b)


Passage 2

1. (c)

2. (c)

3. (c)


Passage 3

1. (b)

2. (d)

3. (a)

4. (c)

Test 6

Passage 1

1. (c)

2. (c)

3. (b)



Passage 2

1. (c)

2. (a)

3. (b)



Passage 3

1. (a)

2. (d)

3. (c)

4. (d)

5. (d)

Test 7

Passage 1

1. (d)

2. (b)

3. (a)

4. (c)

5. (d)

Passage 2

1. (c)

2. (b)

3. (d)

4. (a)

5. (a)

Passage 3

1. (a)

2. (b)

3. (b)



Solutions

Test 1

Passage 1

1.Mentioned in the 4th sentence of the 1st paragraph.

2.The answer is evident from the following lines from the passage, ’His sometimes absurd, and sometimes elegant plays demonstrate a great skill in mixing comedy with honest and sincere ideas. Aristophanes’ plays certainly contain many wild and vulgar jokes, yet they also discuss serious concerns of his time, such as politics, art, and education.’

3.Clear from the following sentence of the passage, Aristotle left behind texts detailing his thoughts and questions on physics and astronomy, as well as meteorology, sleep patterns, raising animals, and numerous other subjects.’

Passage 2

1.In the last 5-6 lines of the last paragraph all the 3 points have been clearly mentioned.

2.The following lines clarify the answer choice, ’having said good-night very amicably, took his way home by himself to a western suburb’.

Passage 3

1.The assembly is the most important issue, it is the topic of the passage so the question has to be related to it.

2.This is given in the lines from the passage ’The Conference provides a setting in which citizens can bring problems to the group for discussion and creative problem-solving.’

3.The answer is clearly mentioned in the following lines , ’The Assembly follows certain rules of logic: in order for masses of people to enter into collective decision-making they must be organized prior to any decisions having been made.’

Test 2

Passage 1

1.The tone is clearly ironical as it talks about so many paradoxes that were prevalent in those times, with a tinge of mockery and sadness.

2.The writing style here uses contrasts to show how equal and opposing forces were pitted against each other in the struggle.

3.It is mentioned as a negative remark, a taunt in the case of France, in the last paragraph of the passage.

Passage 2

1.Hobbes espoused psychological egoism, or the belief that human beings are obligated to act in their own self-interest, even to the detriment of others. In all of the above, the protagonist has an option between saving himself/herself or doing good for others. It is only in option (c) that the mother acting to save her own son compromises self-interest. This is the course of action that Hobbes is thus most likely to reject

2.Hobbes held a pessimistic view of the nature of human beings in that he believed them to be egotistical, power-seeking and rational beings. Selfless action and the spirit of sacrifice was alien to this conception. Thus option (c) is not a characteristic oh human beings, as Hobbes saw it.

3.The state of nature is described by Hobbes as an extremely unsafe place where all human beings are equal, but fear for their own lives. It is characterized by a lack of safety, suspicion and each man being against every other. While option c describes a characteristic of the state of nature, it does not particularly indicate a disadvantage. It is (a) and (b) together that describe the nature of life in the state of nature.

4.’Covenant’ here signifies the agreement that members of the state of nature enter into, in order to escape it. It is neither a decree nor an order. A close option is ’undertaking’ but this does not convey the fact that it is a collective decision. Thus (c) is the correct answer.

Passage 3

1.B is the false statement as it is mentioned in the passage that India in the World Bank’s Doing Business Report 2013 has slipped from 131st to 134th position.

2.In the third paragraph of the passage, the author describes the measures that were undertaken to ’restore competitiveness’ to India, which involved industrial licensing and clearances abolishment. This is definitely one of the ways of enhancing competitiveness, which makes option a, by default, the correct answer. We don’t even have to check to the other options, but if we did, we’d find them to be incorrect with respect to given question.

3.The author is opposed to the suggestions in questions a and b, as these are detrimental to the growth of the nation. Option (d) is not correct because it involves an exaggerated suggestion that is not consonant with the theme of the passage. The third passage correctly summarizes what has been said in the entire passage, that key structural changes are necessary for enhancing growth and correcting the present order in the country. Hence, this is the correct answer.

Test 3

Passage 1

1.It is politics or diplomacy based primarily on power and on practical and material factors and considerations, rather than explicit ideological notions or moral or ethical premises.

2.The author mentions in the passage that Iran had thought out its decision to conclude a nuclear deal with the USA, and had a strategy behind doing so. Thus, it was not a coerced, confused or hapless move, as is suggested by options (a), (c) and (d). Thus (b) is the correct answer.

3.The ’offensive realists’ as their name suggests propose more aggressive and spontaneous measures than their counterparts, the ’defensive realists’, who believe in passive defensive techniques. This is best expressed in option (a), while (b), (c) and (d) deal with irrelevant or incomplete.

4.Since the author believes that Iran’s move was a tactical and calculated one, which was based on key political considerations and concerns, he would not agree with the view that USA had triumphed over or defeated Iran as is suggested by options (a), (b) and (c). Option (d) is the best answer.

Passage 2

1.The author here wants to suggest that India’s partition changed the lives of people, without really changing them at all. They were all still the same people, but suddenly circumstances around them had changed, causing Manto to suddenly feel conscious of his religion among Hindus.

2.The author’s suggestion that the bureaucrat was a constant critic of Manto’s work would imply that he deliberately chose to ignore the merit that Manto’s writing deserved, and instead castigated Manto for faults he perhaps didn’t even have. Options (c) and (d) capture this idea the best, and hence (d) is the correct answer.

3.It is said by the author that Manto’s work was a misfit in its time, when if writing didn’t offer a ’communist cure’ or a ’patriotic facelift’, it was considered useless. Thus (d), which refers to political ideologizing, is what Manto’s work did not attempt to provide.

4.The entire passage attempts to delineate Manto’s predicament upon the partition of his nation and his subsequent attempts to deal with the same through his written work. His body of work based on the partition suggest that he was deeply moved by it. Nor was he glad to leave his home for Pakistan. Manto’s work did indeed receive a huge amount of criticism. Thus option b is the correct answer.

Passage 3

1.’The evidence suggested that Neanderthals headed west, toward Europe, and that the Denisovans moved east.’ This line of the passage can make us safely conclude that option d is the answer.

2.The author includes Dr. Arsuaga’s view that both Denisovans and Neanderthals could have possessed the gene. Thus option (b) is the correct answer.

3.The concluding paragraph of the passage provides the clue as to the most important revelation of the new finding, which is that human biological history may extend beyong 100,000 years ago. This is mentioned in option a, which is the answer.

Test 4

Passage 1

1.The first three options can all be regarded as weaknesses of the R2P as it is mentioned in the passage that it is an open, flexible, differently interpreted concept that has been on occasion used by nations to further interests other than human rights. The fourth option is clearly false as the ICISS and UNSC resolutions are mentioned in the passage and there are references to the role of the international community in debate and diplomacy.

2.The Responsibility to Protect involves three components of government action: prevention, reaction and rebuilding efforts in situations that inhibit human rights. Only the first option describes anything that even remotely resembles such government action. The other three options on the other hand involve abuse of human rights.

3.Answer is (b) because; author has clearly mentioned that the results of the Libyan war had compelled the countries like Russia and China to draft vetoes. No justification whatsoever can be provided for the political grasp of super powers on the implementation of UN Policies, US being the spearhead of all. Still it was not able to take any corrective measures.

4.The author is both critical and skeptical of the R2P, but he also mentions its achievements however many they may be. He ends by saying ’R2P as a concept offers much potential but remains far from perfect’ and also ’If it is the end of R2P, one wonders how then intervention will be framed in the future.’ indicating his appreciation of its viability, however limited. The fourth option is a red herring, meant to throw the student because it has no relevance to the author’s tone (’specious’ meaning: circumspect).

Therefore the third option is the best answer.

Passage 2

1.The answer is (b) because his way of handling the situations, ability to bounce back from failures, fresh skills as a politician and a successful administrator had made the Americans to have confidence in his leadership skills, thereby easing their fear.

2.The author mentions that although he has been criticized for his lack of political success, Kennedy’s domestic record is ’far from poor’. This rules out option (a) and (d) which state these exact extremes. Option (c) is also negated because Kennedy’s failure in the Bay of Pigs is mentioned. Therefore, option (b), describing his efforts towards breaking down discriminatory attitudes and practices is the right answer.

Passage 3

1.Rise in the number of children will cause the savings to fall down, increase in consumption which has been clearly mentioned by the author in the passage. Although number of youngsters will increase, but since the “One-Child” policy has been running from quite a long time, it would take another generation to change the average age for China. Author has clearly mentioned that more of social concerns will arise instead of economic impacts. Parents will feel their children as “Safe asset” so will in turn invest more in riskier assets. So the answer is (c).

2.The entire passage focuses on a single theme: that household in China will be encouraged to save less and spend more as a consequence of the relaxation of the old policy. Thus option (c) is clearly not a consequence.

3.Option (b) is the best option of all as it correctly summarizes the entire passage. Option is wrong as the author makes no such judgment and makes an objective outline of the policy and its revocation. Option (c) is wrong as the author makes a wider study of the policy than just its successes and failures. Option (d) is wrong as it throws a net too far that includes other analyses that the author in this passage does not cover.

Test 5

Passage 1

1.Option (c) is the correct answer as the author believes the right to offend (within certain limits) is an essential part of a democratic, vibrant societal set up. Option (c) and (d) are in direct violation of these principles.

2.The meaning of the word is ’lacking in courage’. Timid is the best option in the given context.

3.Option (b) is the best answer as it accurately summarizes the sentiment of the author who is in clear support of absolute freedom of expression. The other options are not clear representations of this sentiment, while (c) is in direct contradiction. Thus, option (b) is appropriate.

Passage 2

1.The author categorically mentions that Huntington was uncomfortable with the Universalist claim of Francis Fukuyama in predicting the end of history with the collapse of the Soviet Union. This is clearly not a characteristic of Huntington or his writing.

2.The question in option (c) is one the author is unlikely to entertain as he is believed to have ignored cross-cultural interactions in his study of civilizations. This makes this question the least favorable one to ask him.

3.As the passage speaks of the divisions between different cultures that create separate identities for each of them, option (c), which most nearly means ’divisions’ is the best answer.

Passage 3

1.This is evident from the passage because it talks about the poor and the suffering and how they are not able to survive in this world unless they become revolutionaries. Here the thought process of the author goes in the manner that these poor people are not able to live a normal comfortable life, due to which the world is not a place where they could live n a manner every normal person would want to. It is out of their reach and hence it is said that the world is not meant for them. In other words it is a place meant for people who have the means and power to survive.

2.The last line of the 1st paragraph clearly talks about subject peoples, hence statement 3 is correct.

3.This is the central theme or basis of the whole passage, since it is written from their view point.

4.Mentioned in the last two lines of the passage.

TEST 6

Passage 2

1.Clearly mentioned in paragraphs 1 and 2 of the passage.

Passage 2

1.Mentioned in the last 3 lines of the passage.

2.Means that the changes have caused harm to some people or affected them negatively.

3.Statement 1 is totally wrong, hence only the second statement is valid.

Passage 3

1.Social movements are the 2nd step, but not the essential part, thus only statement 2 is absolutely correct.

2.Delivery of the basic services to the deprived sections of the society.[last paragraph of the passage.]

3.Mentioned in points 1 and 2 in the passage which talks about the role of the enabling government.

4.Because the government has to be enabling and thereby promote inclusive growth…not mentioned in both the statements.

5.The 1st line of the 1st paragraph hints towards this, option a gets included in option (d), thus option (d) is a superior option.

TEST 7

Passage 1

1.Both the statements are not mentioned in this context in the passage hence both are invalid.

2.The last 4 lines of the passage mention these points.

3.Mentioned at the start of the passage.

4.Clearly one of the key points of the passage, mentioned as an important fact in the last few lines.

5.Since it is an inferential question, thus the answer cannot be directly stated in the passage and has to be inferred, thus option (d) is the correct choice which is easily inferred whereas option (a) is directly stated, hence not the correct answer.

Passage 2

1.This is the central theme of the passage.

2.It is clearly stated in the passage that the quantity of mussels in an area is determined by the presence or absence of its predators, the sea stars. Statement 1 is incorrect as mussels are controlled by sea stars hence cannot be termed as the dominant species, and are rather called the competitive species.

3.Clearly mentioned in the passage, in the 1st paragraph

4.Statement 1 is definitely true, whereas statement 3 is not true because removal of the keystone species does not devastate or collapse the ecosystem, rather it changes it.

5.The mussels is the most dominant competitor of the Pisaster, and is clearly mentioned in the first few lines of the passage.

Passage 3

1.A moral act must be our own act; must spring from our own will. Hence statement 1 is correct, although statement 4 is also mentioned but it says sometimes so it is not the usual but a rarity, thus only 1 is the best option.

2.A moral act must be our own act; must spring from our own will. Hence, option (b) is the right option.

3.It means that he is not using his own mind but is influenced or obeys orders blindly. Thus both statements 1 and 2 are relevant.